Final 2

Pataasin ang iyong marka sa homework at exams ngayon gamit ang Quizwiz!

You are transporting a patient with a past medical history of a meningeal tumor. You would recognize this kind of tumor as being located

In tissue surrounding the brain

Which statement made by a pregnant patient in labor should be of most concern to the EMT?

"My water broke two days ago."

Which class of chemicals would a terrorist likely use in an attack

Class 2 Gases

A patient has been bitten by a nonpoisonous grass snake. Your care would include

Cleaning and dressing the wound

Dead spots represent a failure in which system at an MCI

Communications

Blood in the anterior chamber of the eye is called

Hyphema

As brain herniation occurs, the body responds by:

Increasing systolic blood pressure in an attempt to maintain perfusion to the brain

As soon as the baby is delivered from the vaginal canal, it is critical that the EMT immediately

Initiate drying and warming of the newborn

An extremely high fever in a 2-month-old infant should be considered which medical emergency until otherwise proven?

Meningitis

A 23-year-old female has been involved in a serious motor vehicle collision. Which assessment finding best indicates that she has a flail segment?

Uneven chest wall movement during breathing

You are assessing the pupils of a 5-year-old who fell off a bed while playing. Which of the statement would be most appropriate prior to performing this assessment?

"I am going to use this light to look into your eyes."

You have been called for a 79-year-old female complaining of shortness of breath. Due to the patient's increased respiratory rate, cough, and production of discolored sputum, you are suspicious of pneumonia. A family member asks you how she can have pneumonia, when her temperature is only 97.9° F. Your best response is

"It is not uncommon for the elderly to have pneumonia without the signs of high fever or chills

You have been summoned to a beach for a young male patient who has been stung by jellyfish on both forearms. As the senior EMT on scene, which directions should you provide to the other EMS providers in providing proper care to this patient?

"Let's soak the arms in hot water

A 14-year-old female with a history of congenital heart disease and evidence of reaching puberty is in cardiac arrest. Which instruction would you give other EMTs on scene

"Let's use the adult pads and adult settings for the AED."

You are giving a talk on hunting safety. A hunter asks you when someone should attempt to rewarm a hand or foot that is frostbitten. What is your best response

"Rewarming should take place only when there is no chance that the tissue will refreeze."

What would the expected preductal SpO2 reading be at 5 minutes after birth?

At least 80%

A female patient tells you that she is 38 weeks pregnant. Where would you expect to palpate the top of the uterus during your exam?

At the level of the xiphoid process

What is the risk of operating an ambulance in a fashion that creates a negative cabin pressure

Buildup of CO2 inside the unit

A patient has been assaulted and sustained blunt trauma to the abdomen. As you start the primary assessment, you note that he is vomiting blood and you begin to suction him immediately. After the airway has been suctioned, you should

Check the adequacy of the patient's breathing

After restraining a violent patient on the stretcher, which action is it essential that the EMT perform as soon as reasonably possible?

Check the tightness of the chest strap

Assessment of an 88-year-old female who fell reveals bruising lo her right buttock. The skin is intact and the patient complains of tenderness to the area when you palpate. As a knowledgeable EMT, you would recognize this patient has which type of injury?

Contusion

Vision disturbances caused by glaucoma occur secondary to

Damage to the optic nerve

What is your first priority in managing a patient with a psychiatric (behavioral) emergency

Ensuring the safety of you and your partner

A patient has swelling and deformity to the wrist. After splinting, in which position should the hand be placed

Fingers curled inward

A terminally ill female with ovarian cancer is receiving palliative care when she:

Has a morphine pump implanted in her abdomen

A worker has been exposed to beta rays given off by a radioactive material, but is not contaminated. He presents to you when the fire department carries him out from the industrial plant in which there was an explosion and partial building collapse. What should you do first?

Initiate your primary assessment

You have been called for a 2-week-old baby who is sick. Assessment reveals him to have a fever and rhonchi that is audible with a stethoscope over the lungs. Which of the additional assessment findings would be most concerning to you given the age of this patient

Nasal passages occluded by mucus

Physical signs and symptoms such as severe neck pain, clumsiness, and trouble swallowing can be indicative of

TBI

What best describes the purpose of determining a Glasgow Coma Scale (GCS) score in a patient with a head injury?

The GCS score helps determine whether a patient's mental status is improving or deteriorating

An example of a quinary effect of an explosion is:

Toxic contaminants

In a patient with post-traumatic stress disorder (PTSD), the EMT would recognize that a patient is reliving an event when he states

"I cannot sleep well because of nightmares that I am back in the desert."

While transporting a patient with schizophrenia, he suddenly screams, "Do you see that snake? It is right next to you! Kill it! How should you respond to this patient?

"I do not see a snake; I see a stethoscope. I do not doubt that you believe you see a snake, though

A construction worker complains of pain to both eyes after an extended period spent welding on a gas well. The light is bothering him, and he rates the pain as a 10/10. He states that this problem occurred once before and was caused by the intense light of his welding tool. The primary assessment reveals no deficits. The patient's vital signs are pulse, 80 beats/min; respirations, 14 breaths/min; blood pressure, 132/68 mmHg; and SpO2, 98%. Although the injury is very painful, his vision is not compromised. Which set of instructions indicates proper care of this patient

"I do not think that we will need to provide supplemental oxygen."

A patient with postural hypotension would most likely make which statement

"I get so dizzy when I get up off of the couch."

When assessing a 27-year-old patient who is eight months pregnant, which statement made by the patient should the EMT be most concerned about

"I have not been able to see the doctor this pregnancy at all."

You are presenting information on emergency care to a class of pregnant women. A participant asks you what the most frequent cause of injury is to a woman during pregnancy. What is the correct response?

"Motor vehicle collisions account for about half of all injuries sustained during pregnancy."

A patient with blunt chest wall trauma has a flail section to her chest. She is conscious and confused, her breathing is rapid and shallow, and breath sounds are diminished bilaterally. After manually stabilizing the flail section of the chest wall, the EMT best treats this condition by:

Providing positive pressure ventilation with oxygen attached

You have been called for a 77-year-old female with altered mental status. On scene, you note the patient's inability to speak and vomitus in her airway. Her breathing is labored and inadequate, and radial pulse rapid and weak. She is not moving her right arm or right leg. What should be your first priority

Suction the airway and determine adequacy of the breathing

A male patient has sustained gunshot wounds to the neck and abdomen. He is unresponsive and has gurgling respirations. Your immediate action would be to:

Suction the airway while providing manual spine motion restriction

Which assessment finding would best indicate that a patient has suffered a full-thickness burn to his leg?

Dry skin with little pain

Which observations would cause the EMT to classify a patient's soft tissue injury as an abrasion?

Skin is scraped and red; blood is oozing from the injury site

A 21-year-old-female has been struck in the left eye by a softball during a game. Which finding is most concerning for an eye injury

Unusual sensitivity to light

Which type of muscle tissue is found in the forearm

Voluntary

Which medical problem commonly leads to vision problems

Diabetes

You have been asked by your medical director to put together an emergency kit specifically for patients who have been stung or bitten by small marine life animals. Which of these items should you place in that kit?

Vinegar

Asherman, Halo, and Bolin devices are commercially available devices to use for which injury

Open chest wound

You have been dispatched to a Veterans Administration (VA) clinic for a male patient with a behavioral emergency. On scene, the VA physician tells you that the patient is a returning veteran with newly diagnosed post-traumatic stress disorder (PTSD). Based on this information, you would surmise

the veteran may have been in the military but not necessarily involved in combat

At the scene of a multiple-casualty incident, you note that another EMT has become overwhelmed and emotionally incapacitated after unsuccessfully trying

to save a child's life. You should

As you assess a very sick 3-year-old patient with a fever, rash, and sore neck, you suddenly suspect he might have meningitis. Your immediate action would be to:

Don additional BSI precautions of a mask and gown

It has been 30 minutes and the placenta has yet to deliver. The EMT should:

Continue to monitor the mother and baby

An 81-year-old woman with severe kyphosis and osteoporosis fell and now complains of neck pain, back pain, and nausea. You can palpate a lateral displacement of a cervical vertebrae. Given the patient's past medical history, what is most appropriate

Place her in a supine position on the long spine board and pad all voids

Assessment of a 4-year-old reveals him to be unresponsive with no spontaneous respirations or pulse. Your immediate action would be to

Start chest compressions

You are suspicious that a child is being physically abused and neglected. What would be your best course of action in such a situation?

Document the situation

You have been called to a residence for a spider bite. On scene, the homeowner states that he was moving some boxes in his garage when he accidentally grabbed a spider and it bit him. He killed the spider and shows you a brown-colored spider with a "violin shape" on its back. He then states that the bite does not hurt and he sees no need to go to the hospital. As a knowledgeable EMT, you should state:

"You really need to go because the bite of that spider does not heal very well and will put you at risk for infection."

Which statement made by the EMT shows an understanding of how an open wound should be bandaged?

"I use bandaging material to secure the sterile dressing in place."

After taking a young, depressed female to the hospital, your partner says, •1 am tired of running all of these psych patient calls. There are real patients with real emergencies and pain out there. It is just a waste of our time!" Given his thoughts, what is your best response

"Patients with mental problems are suffering from emotional pain and injuries, which can be just as real as physical pain.

You have been called for a male patient who began complaining of chest discomfort while receiving hemodialysis. Assessment shows no threats to the airway, breathing, or circulation. His vital signs and blood sugar are normal. Which question should you ask first?

"Was the dialysis session completed?"

In which area of the spinal column do the ribs originate

Thoracic spine

You suspect that a patient has a fracture of the left orbit. Which statement made by the patient would reinforce this suspicion?

"I am seeing two of everything."

The fundamental reason a patient uses BiPAP when sleeping is to:

Keep the small airways open

A 1-year-old male is in respiratory distress and wheezing. His pulse is 156 beats/min, respirations are 32 breaths/min, and SpO2 is 95% on 4 lpm of blow-by oxygen provided by on-scene EMRs. His mother denies a medical history for the patient, but states that she has asthma and uses an albuterol inhaler. Given this information, what would be appropriate in the care of this child

Add humidification to the oxygen

The EMT would best describe a hazardous material as

Any material that poses a threat to life

A patient with hypothermia is alert and confused, but complains of left arm pain. His friends found him outside, where the patient spent the night after he got drunk and passed out in the cold. It appears as though he has a localized freezing cold injury to the arm in question. In caring for this patient, which of these treatments would be acceptable

Board splint the arm to keep it from moving

An unresponsive patient is in a car with the doors locked and the windows up. Which action is most appropriate when attempting to gain access to the patient

Break the window opposite the patient with a spring-loaded punch

When assessing a 3-year-old child for possible injuries after the child fell down a flight of stairs, which finding would raise the EMT's suspicion that the child may be a victim of physical abuse

Bruises are found on his chest and abdomen

When using an air splint, the EMT must remember that air splints

Can decrease circulation in the extremity

Five minutes after delivering a baby, the mother feels the sudden urge to push and a gush of blood comes from the vagina. Given that the mother is not delivering twins, which stage of labor should the EMT recognize?

Placental

Damage to which thoracic tissues can cause impairment of ventilation from a developing pneumothorax

Pleural membranes

A central intravenous catheter is typically placed for which reason?

Repeated administration of medications

A landscaper who has been working in 105°F weather is found collapsed in a client's yard. He is responsive to painful stimuli and has an open airway. His breathing is shallow, alveolar breath sounds are absent, and radial pulse is weak. The patient's skin is hot and dry and appears to be sunburned. The EMT should immediately:

Start positive pressure ventilation

Prior to the revisions in prehospital spinal assessment and care, the primary criterion for determining the need for spine motion restriction was:

The mechanism of injury

What is considered to be a normal finding when assessing a patient with a possible head injury

The right pupil constricts when light is shined into the left pupil

A 3-year-old female has been accidentally struck in the chest with a baseball bat during a domestic altercation. Assessment indicates redness to the right lateral chest with no deformity, crepitus, or broken skin. Vital signs are pulse, 124 beats/min; respirations, 22 breaths/min; blood pressure, 108/62 mmHg; and SpO2, 99% on room air. Given these assessment findings, what would your best action be?

Treat the child for suspected internal chest injury

Which statement shows that an EMT understands field care of a patient with a possible joint dislocation

"The care for a patient with a joint injury is identical to that for a patient with a fracture.

Which one of these statements about drowning is true?

Actual drownings are responsible for only a small number of water-related deaths

While responding to an MCI drill involving an explosion, you pass seven ambulances sitting in a parking lot three blocks away from the incident. You should recognize this area as the

Staging unit

The EMT would recognize a patient as having a phobia when the patient makes which statement?

"I cannot eat meat because they put dangerous bacteria in it that can hurt me."

What are the three main layers of the skin?

Subcutaneous, epidermis, dermis

When providing care to the pediatric patient who has been injured or ill, the most important aspect of care normally revolves around:

Airway and respiratory support

The components of the Apgar scoring system include

Heart rate, spontaneous activity, and respirations

You have arrived on the scene for a patient assaulted with a baseball bat. As you approach, which position would seemingly indicate that he has suffered an injury to the abdomen

On his side with knees drawn to the chest

At the scene of a multiple-casualty incident, you are triaging patients using the Simple Triage and Rapid Transport (START) system. You are presented with an unresponsive patient who is not breathing. What should you do next

Open the airway

When performing the primary assessment, which sign or symptom best suggests that the patient is in shock?

Skin that is cool and diaphoretic

You arrive on scene to find that the police have subdued a violent man with a history of psychiatric problems. The police state they will put the patient on the stretcher. Which action should cause the EMT to take immediate and corrective action

The patient is placed and restrained in a prone position on the stretcher

The increased pliability of the child's ribs makes him more prone to:

Bruising or other injury to the lungs

In a nuclear explosion, the EMT should expect the majority of patients to complain of

Burns

The EMT's initial concern when treating a patient with a gunshot wound to the chest is:

Ensuring an open airway

To best communicate with a geriatric patient with diminished eyesight, the EMT should:

Stand in front of the patient and talk normally

When assessing a 91-year-old female who fell, which assessment finding should concern the EMT most

Sudden onset of confusion

A ventilator-dependent patient is being ventilated with a bag-valve mask as he is transported to the hospital. Which finding is the best indication that the patient is being adequately ventilated during transport?

Sustained SpO2 reading of 98%

Pediatric patients are patients who range in age from

birth to 18 years

Which patient would the EMT classify as having a psychiatric (behavioral) emergency

A 36-year-old male with no known history who is hearing voices

A trauma surgeon informs you that the patient whom you transported to the emergency department earlier was diagnosed with an epidural hematoma. The patient was taken into surgery and the hematoma was removed, and now the patient is in critical condition. Based on this information, the EMT should recognize that in this patient

A pocket of arterial blood collected between the skull and dura matter

When using the Glasgow Coma Scale (GCS), the EMT understands that

The higher score, the better the patient's neurologic status

Who determines the order in which patients are transported to various hospitals in the area of the MCI

The transport unit leader

An unrestrained female driving a small car is involved in a rollover-type collision. Why is her risk for serious injury or death significantly increased?

There are more impacts in a rollover causing injury

In addition to brain injury from the high voltage and amperage, the patient who experiences a lightning strike also typically develops:

Trauma from air rushing out of and back in to the strike location

You are called to a home for a 7-year-old female with a suspected upper respiratory infection. As you assess the patient, you note a bruise to her right upper arm. As you continue to assess and care for this child, you should:

Treat the respiratory infection and perform a head-to-toe assessment

The EMT is correctly assessing a 2-year-old male patient who is unresponsive after being assaulted by the mother's live-in boyfriend when he

Checks capillary refill time to help determine perfusion

An alert and oriented patient has had a portion of his hand amputated by a table saw. Assessment reveals the patient to be in great pain. with a moderate amount of dark red blood still coming from the injury site. His airway is open and his breathing is fast. but adequate. The radial pulse is rapid and strong, and his skin is warm and dry. The patient's amputated hand has been placed in a bag of ice by coworkers. What should the EMT do immediately?

Control the bleeding with direct pressure

A 76-year-old female fell while walking to her bathroom. She is now on the floor and reports severe pain to her right hip. Her airway is patent and her breathing adequate. Family members tell you that she takes medication for high blood pressure, high cholesterol, and depression. When assessing the patient, which assessment finding is most concerning?

Heart rate of 128 beats per minute

The EMT's care for an open chest wound and for an abdominal evisceration are similar in that:

Occlusive dressings should be placed over both wounds

A 52-year-old male is unresponsive and has bruises and lacerations to the head, chest, and abdomen. Bystanders state that the patient was intoxicated and got into a fight with several patrons of a bar. They state that he was beaten with his opponents' fists, but not other objects or weapons. The patient has snoring respirations, shallow breathing, and a strong radial pulse. His skin is warm and dry. After providing manual spine motion restriction, what should be your next action

Perform the jaw-thrust maneuver

A child has been pulled from the pool after being submerged for less than 30 seconds. The child is crying and hyperventilating, as well as coughing up water and vomiting. The EMT should:

Place the child on his side and initiate a primary assessment

An 8-year-old boy is unresponsive following the accidental ingestion of his father's high blood pressure medications. His airway is open and his breathing is labored and inadequate. The pulse rate is 48 beats per minute and his skin is cool and diaphoretic. Your partner states that the patient's lungs have crackles in them and the blood pressure is 60/40 mmHg. Which intervention represents the most important care that you will provide to the patient next?

Positive pressure ventilation

Your EMS system uses the Simple Triage and Rapid Transport (START) system for triaging patients at a multiple-casualty incident. Which item(s) should you include in your triage assessment to determine the order in which patients will receive emergency care?

Respiratory and perfusion status

You have been called to a home for a female in labor. On scene, the family tells you that the 37-year-old patient is 42 weeks pregnant and wanted to have her baby at home naturally. The patient has been in labor for more than 24 hours, but suddenly began complaining of severe and tearing abdominal pain. Which condition should the EMT suspect?

Ruptured uterus

The best way to detect contact lenses for removal is to:

Shine a penlight into each eye at a slight angle and look for a shadow

A patient with a history of arteriosclerosis has

Stiffened arteries

You have been called by family members for their mother, who is "not acting right." At the scene, the family informs you that they are concerned because their 68-year-old mother has been complaining of a headache for two days and is now very confused. When asking about a bruise on the right side of her forehead, the family states she fell in church a week ago and hit her head. Since all other aspects of the assessment are unremarkable, you suspect a head injury. Which type of injury would you suspect given the findings and history

Subdural hematoma

A patient has had part of his right thumb amputated in an industrial accident. Coworkers have retrieved the thumb and wrapped it in a towel. The EMT demonstrates appropriate handling of the amputated part when he:

Wraps it in a dry sterile dressing

Which statement by the EMT indicates proper application of the term drowning?

"A drowning occurs anytime a person is submerged in a liquid and suffers some sort of respiratory impairment."

What of these patients is most accurately described as suffering an environmental emergency?

An intoxicated male patient who collapsed in the snow and is complaining that he feels cold

At a long-term care facility, a nurse informs you that the patient whom you will be transporting has a peripherally inserted central catheter (PICC) in place. To find this catheter, you should look at the patient's:

Antecubital fossa

An elderly patient fell down a flight of basement stairs and is found at the bottom by family members about 20 minutes later. Your assessment reveals that the patient cannot feel painful stimuli to his hips and legs, nor can he move his legs, but he can feel you lightly touching the skin of his legs. Which type of injury may this be?

Anterior cord syndrome

While evaluating a patient who experienced a fall, you note crepitus in the left forearm in the area of the wrist. Crepitus in the wrist is typically caused by

Bone ends grinding together

As a diver rises too quickly from a deep-sea dive, the nitrogen in his bloods forms ever-enlarging bubbles. The bubbles then cause damage to internal organs and tissues. The EMT would recognize this outcome as demonstrating:

Boyle's law

You are by the side of an 11-month-old female with a decreased level of consciousness. Caregivers will not provide an accurate history of how the child came to this condition, only that she was difficult to wake up this morning, about 30 minutes ago. When assessing the child, which finding would be most immediately concerning

Bradycardic heart rate

Which class of hazardous materials includes radioactive sources that may be exploited by terrorists in an attack

Class 7

The EMT should recognize a Texas catheter when he observes a:

Condom-like device over the patient's penis

An intoxicated 28-year-old female has suffered burns to both legs after passing out next to a space heater, which ignited her pants. Emergency Medical Responders have removed her clothing and cooled the burns. Which action is most appropriate in the continued care of this patient

Cover the burns with a clean, white, dry sheet

A new EMT asks you why the elderly are at a higher risk for developing pneumonia, as compared to younger individuals. You inform him that one reason the elderly are more susceptible to respiratory infections because of a(n):

Decreased cough reflex

The son of an 88-year-old patient states that he told his mother to double the dose of her antibiotic so she would "get better faster." This represents a problem given that the elderly have a(n)

Decreased liver and kidney function

What would be a primary concern for an 11-year-old female with a prolonged asthmatic episode lasting several days and a low-grade temperature

Dehydration

The secondary radiation exposure from a nuclear blast is the result of

Fallout

Upon arrival at the home of a woman in labor, a midwife on scene tells you the patient is 7 cm dilated and is complaining of painful contractions. Which stage of labor should the EMT identify

First stage

You have arrived on the scene to help a child who is short of breath. The mother is screaming hysterically that her 3-year-old boy cannot breathe, making assessment of the patient very difficult. Your initial action in the management of this situation would be to:

Have your partner talk to the mother while you assess the child

A patient has referred pain to her left shoulder. What should the EMT suspect?

Hemorrhage from the spleen

When treating a confused patient who has been exposed to high temperatures, which of these findings is most concerning

Hot and dry skin

Which layer of the skin contains the larger blood vessels?

Hypodermis

A confused 62-year-old female fell at home, hitting the side of a table with her chest. Assessment reveals instability to the left lateral chest, accompanied by minor bruising in the same area. The patient complains of intense pain every time she breathes. Her airway is patent, breathing is rapid and shallow, and skin warm and non-diaphoretic. Vital signs are pulse, 112 beats/min; respirations, 24 breaths/min; blood pressure, 132/64 mmHg; and SpO2, 90%. Breath sounds are somewhat shallow but still equal to both lungs. Based on those findings, you should treat this patient for which life-threatening condition

Hypoxia

A 49-year-old male was climbing on a truck at a construction site when he fell backward to the ground. He presents with a 2-inch linear wound to the top of his head. Bleeding has been controlled and the skull can be seen through the wound. How should you document this injury on the prehospital care report

Laceration

Which triage system is intended to be adopted as the national standard for MCI triage

SALT

Which action would reduce the chance of being involved in an emergency response-related collision with the ambulance

White strobe lights should be used in conjunction with the red flashing lights

The head of the water rescue team has just informed you that the divers have retrieved a young woman who fell through ice on a pond while walking her dog. The estimated submersion time is 22 minutes. Another EMT asks you if it will be necessary to start resuscitation, given the length of submersion. What is your most appropriate reply

"Absolutely! Given that she has been in cold water, she may still have a chance of survival."

The mother of a 3-year-old boy has called 911 because her son has a low-grade fever and difficulty breathing. On scene, the mother tells you that she is a licensed practical nurse (LPN), and that she suspects her son is suffering from croup. Which statement made by the mother would reinforce this suspicion?

"At night, he seems to get a barking-like cough."

The EMT is properly assessing for a spinal cord injury when she

Asks the patient to spread his fingers apart on both hands

An example of an injury that results from the primary exposure in a nuclear explosion is

Multisystem trauma

The primary assessment of a teenage male patient who is unresponsive with hot, moist, and flushed skin reveals his airway to be open, breathing shallow, and radial pulse weak. You have initiated positive pressure ventilation with supplemental oxygen. What should you consider doing next?

Transfer the patient into the ambulance

You are treating a 6-month-old patient who was accidentally dropped down a flight of steps, when her mother stumbled at the top of the stairway. The infant will only open her eyes and moan to deep painful stimuli, and tries to withdraw from the pain. Given these findings, you calculate her Pediatric Glasgow Coma Scale score to be

8

Two cars have collided head-on. One car was traveling at a speed of 55 mph and the other at 35 mph. The total speed of impact would be

90 mph

7. Direct pressure has failed to control an arterial bleed on a patient's lower leg caused by an industrial accident. what the EMT next action be

Apply a tourniquet

A patient with a past history of quadriplegia from a four-wheeler accident would most likely be unable to:

Hold utensils for eating

A 25-year-old male jail inmate was pushed over the railing of a walkway 20 feet above the ground. He is unresponsive and has an open fracture of the left upper arm. How would you determine if the patient has sensation in his legs

Pinch his foot and look for movement on the leg

When an EMT is driving an ambulance in emergency mode (lights and sirens), most states do not permit the EMT to:

Proceed through a red light without stopping

A middle-aged male sustained blunt force trauma to the abdomen, resulting in an isolated tear and perforation of the large intestine. When following up on the patient, what would the EMT suspect occurred, given the organ that was injured

The patient developed an infection

The EMT shows she understands the regulation of temperature in the human body when she states:

"If the body becomes too hot or too cold, the brain will send instructions out to the body to either retain or lose heat.

The EMT would recognize the patient has a sensory impairment when he encounters:

A 49-year-old male who is hearing impaired in one ear

For which patient would the palm method be best to estimate the body surface area affected by a burn?

A 49-year-old patient with a partial-thickness burn to the inner thigh

When an elderly individual is injured, deterioration of compensatory mechanisms in the patient can result in:

A rapid onset of shock

You observe a traumatically injured 3-year-old female patient being secured to a long spine board with no padding behind the neck, shoulders, or back during a spine motion restriction intervention. What is the main concern with this oversight? A1 Pain and discomfort

Airway compromise

A patient complains of lower back pain after falling down five stairs. Assessment of the patient's cervical spine reveals no displacement, tenderness, or instability. The patient does state that each leg has feelings of "electrical shocks" shooting through them. How should the EMT care for this patient?

Apply a properly sized cervical collar after initiating manual spine motion restriction

Pediatric injury prevention programs are an important aspect of child health care because:

Children inherently lack mature decision-making skills

The parameters of the Glasgow Coma Scale are

Eyes, motor, and verbal response

You are transporting a female patient who is seven months pregnant. She has been diagnosed with preeclampsia and is currently hypertensive. Which

I feel like I am going to have a seizure.

A patient with a history of cataracts would most likely complain of:

Inability to see clearly

An industrial worker has sustained a laceration to his abdomen. On closer inspection, you note what appears to be fat tissue and a portion of intestine protruding through the wound. Which intervention is most appropriate for treating this injury?

Occlusive dressing applied over saline-soaked gauze

A patient with kyphosis has fallen and now complains of new-onset neck and back pain with numbness in both arms. Regarding spine motion restriction, you should

Place pillows In voids between the patient and the long board during the spine motion restriction process

You are preparing a patient with an indwelling urinary catheter for transport. Pertaining to the catheter, which action must the EMT take?

Place the catheter bag below the patient's bladder

Why would an EMT be concerned about a partial-thickness burn circumferentially to the chest

Restriction of breathing

A football player injured his knee during practice and is in pain. His left knee is swollen, ecchymotic, and flexed in an upward position. Your partner reports that the distal skin is warm, and he has located a weak pedal pulse. Which action should you perform immediately?

Splint the knee in the position found prior to providing transport to the hospital

Which statement shows that the EMT understands the palm method of estimating the BSA burned?

"I use the patient's palm to estimate the size of the burn area."

Which statement regarding limb presentation is true during childbirth?

A cesarean section birth will likely be required

Which bodily activity is a function of a voluntary muscle?

Chewing food

What is the LEAST reliable way to determine the presence of hazardous materials

Your senses

A 25-year-old female presents with abdominal pain. She appears thin and healthy, but in obvious distress. Of the many questions the EMT may ask, which one should the EMT ask early during the patient interview?

"Is there any possibility you are pregnant?"

The EMT correctly defines a multiple-casualty incident (MCI) when he states

"It is any event that places an excessive demand on rescue personnel and equipment."

When should the EMT instruct his partner to cut the umbilical cord?

Following delivery and after the baby has been dried off

Following a direct lightning strike, which symptoms should the EMT expect the patient to display?

Apnea

Which piece of information would you provide to a group of young teenage drivers to decrease their chance of suffering a "whiplash"-type injury if involved in a motor vehicle collision?

Ensure that the head rest is properly positioned

Which statement demonstrates that the EMT has a proper understanding of basic operations at the scene of a hazardous materials emergency

"Life or health of the EMS providers should never be risked if the threat is only to the environment

A patient has an injury to the elbow and forearm. While you are conducting the reassessment, which statement made by the patient should concern you most

"My hand feels like it is going to sleep."

Which statement made by an EMT shows a proper understanding of the EMT's role at the scene of a multiple-casualty incident

"The role of the EMT will vary depending on the exact situation."

A young female, after being rescued from a burning apartment, is found to have partial- and full-thickness burns from her chest down to her feet. As such, the EMT would recognize the potential for the onset of

Hypothermia

Which statement by the patient best indicates that the patient is in the second stage of labor

I feel like I have to move my bowels.

A patient who is conscious and breathing has been pulled from a stream of cold water. To decrease her loss of heat via the mechanism of conduction, the EMT should immediately

Remove her wet clothing

An EMT is presenting a continuing education class on the care of the pregnant female. When discussing the amniotic sac, he is correct to emphasize which point

The amniotic sac protects and insulates the baby during gestation

While delivering a baby in the field, the EMT notices that the baby is covered with a green-brown liquid. The EMT should immediately recognize that

The baby was distressed and may be hypoxic

The EMT shows that he understands the danger of heat stroke when he states:

"In heat stroke, the body loses its ability to rid itself of excess heat, causing the core temperature to rise

You are providing a "stand-by" service at the county fair. At midmorning, a 50-year-old male patient approaches your tent and states that he was bitten in the right shoulder by some sort of insect and has tremendous pain. Assessment reveals his shoulder to have a large reddened area with a sting mark in the middle. The skin is warm to the touch. Your care of this patient should include:

Applying a cold pack to the site

Which sign or symptom of shock will be observed last?

Decreased blood pressure

You have arrived at the scene of a shooting where a middle-aged male sustained a single gunshot wound to the abdomen. As you approach the patient, you note that he is sitting up and attempting to talk with the police officers while holding a blood-soaked towel over the left upper quadrant of his abdomen. Once you get beside the patient, you realize he is speaking in a different language that neither you nor the officers understand. Your initial action in caring for this patient is to:

Initiate spine motion restriction precautions

A 57-year-old male has been involved in a motor vehicle collision and is entrapped in his car. He is unresponsive, with agonal gurgling respirations and blood coming from his nose. His respirations show poor effort and are inadequate. The radial pulse is rapid and weak, and the skin cool and diaphoretic. An Emergency Medical Responder is holding manual spinal motion restriction. Which action would you perform first

Suction the airway with a catheter

Which of these statements made by an EMT shows that he understands the care of a patient with a freezing cold injury to the foot?

"While it is recommended to remove wet clothing from the patient, it is best to leave clothing that is frozen to the skin in place."

The EMT should recognize a full-term pregnancy has occurred in which patient?

A 21-year-old woman who is 39 weeks pregnant and feeling dizzy

The EMT would recognize which neonate as premature

A baby weighing 7 pounds 4 ounces born at 36 weeks

Which of these patients would the EMT recognize as suffering from an evisceration injury?

A male patient with a loop of intestine protruding from an open surgical wound

You have been dispatched for a 2-year-old female who is not breathing. When responding, you realize that your primary goal is to

Arrive at the scene safely

A patient with advanced dementia who is living with family members is unable to provide an accurate medical history. In this situation, what would the EMT's best course of action be

Ask family members about the patient's past medical history

A problem with the Simple Triage and Rapid Transport (START) system of triage is that it

Is difficult to apply to young children

Which organization developed and published "Strategy for a National EMS Culture of Safety

NHTSA

The EMT's primary concern with any open injury to the chest is:

Stopping air entry

Which statement is true regarding driving in inclement weather

Stopping on wet pavement takes approximately twice the distance as stopping on dry pavement

After positive pressure ventilation, a newborn's heart rate increases from 80 to 120 beats per minute. The EMT should:

Stop positive pressure ventilation and provide blow-by oxygen

Which statement about different musculoskeletal injuries is true

A sprain is an injury to a joint with possible damage to or tearing of ligaments

Which injury is most serious, warranting immediate and rapid transport to the local trauma center?

Instability and crepitus to the left lateral chest wall

A young boy was running with a pencil and tripped. The pencil impaled the boy's left eye and remains lodged in place. He is in his mother's arms and both are crying hysterically. Appropriate care for the eye would include:

Stabilizing the impaled object and providing immediate transport

A patient with hypothermia is in cardiac arrest. The automated external defibrillator (AED) is applied and delivers one shock. Following the shock, what should the EMT do next?

Start cardiopulmonary resuscitation

An elderly female fell down three steps and now complains of back pain. After performing the primary assessment, which question would be most important to ask first

"Did you become dizzy and fall or slip and then fall?"

You have been called to a nursing home for a confused 91-year-old male patient who became dizzy and fell, striking his head on the floor. Which statement made by the nurse should be of most concern to you?

"He has a heart valve problem and takes Coumadin to help prevent his blood from clotting.

The EMT would recognize an era veteran when a family member states:

"He trained for combat but was never deployed."

You are assessing a 6-year-old girl with possible pneumonia. She has labored breathing and a fever of 102° F. When you are assessing and classifying her respiratory status, which finding would provide the strongest evidence that she is in respiratory failure?

Altered mental status

You respond to a single-car accident in which an unrestrained 3-year-old child was thrown from the back seat into the windshield, striking his head. As your partner applies spine motion restriction precautions, you note that the child has a patent airway, is breathing well, and is alert and oriented; however, you also notice that blood and fluid are draining from his right ear. How would you treat the bleeding

Apply a dressing loosely over the ear to absorb the blood and fluid

You arrive at a meat packing plant to find a 30-year-old male patient who was cut in the wrist with a sharp knife. The patient appears pale, and blood is spurting from the laceration. Your immediate action would be to

Apply direct pressure to the laceration

You are transporting an elderly female who fell in a nursing home and displays outward rotation of her left leg. She has a lengthy medical history, including dementia. Currently, she is very quiet and receiving oxygen through a nasal cannula at 6 liters per minute. She also is secured to a long spine board. During your reassessment, you note her to be alert but confused, with slightly accelerated respirations and a strong radial pulse. You also note that her Sp02 has dropped from 97% on scene to 91%. You should

Apply supplemental oxygen

A patient complains of back pain and numbness to both legs after being thrown from a bicycle. When should the EMT first check the motor function, sensory function, and pulses in the legs of this patient?

As the secondary assessment is performed

You have just applied a pressure dressing to the leg of a woman who suffered a deep laceration to her calf from a piece of broken glass. What should you do next

Assess for a pedal pulse

As you approach a young male lying on the sidewalk who was called into 911 as an "unresponsive male," you observe an area of dark blood on his pants. He appears lethargic, pale, and diaphoretic. Your first action in caring for this patient would be to:

Assess the airway

A patient with Cushing reflex is most likely experiencing which pathophysiology

Brain herniation

You have been called for an intoxicated male who sustained a full-thickness burn to his leg after passing out against a kerosene heater. After ensuring that his airway, breathing, and circulation are intact, you cool the burn with water and then

Carefully wrap the burn with a dry sterile dressing

An 87-year-old female has called 911 for generalized weakness. When assessing her pupils, you note the right eye to be cloudy, making the assessment difficult. Based on this finding, you should be suspicious of:

Cataracts

A bullet fired from a gun at close range passes through the patient's liver. However, on autopsy, the coroner discovers that the man's pancreas, stomach, and gallbladder were also injured, even though not in direct contact with the bullet. As a knowledgeable EMT, you would recognize that which mechanism is responsible for injuries to these additional organs

Cavitation

A worker in a factory complex has a sliver of metal lodged in the colored portion of his eye. The EMT would recognize the foreign body as lying in the

Iris

A localized terrorist attack involves pepper spray being released at a high school dance. While you are en route to the scene, the incident commander reports approximately 50-plus people have been exposed and calls for activation of the county disaster plan. Given the nature of the weapon, what should you anticipate doing on scene

Irrigating eyes with saline

During a continuing education session on OB/GYN topics, the EMT would recognize which statement about the umbilical cord as true

It contains one vein and two arteries

A 3-year-old child has stuck a crayon in his nose. Assessment reveals the crayon to be deeply embedded in the right nostril, with some irritation and swelling noted. His vital signs are pulse, 124 beats/min; respiration, 20 breaths/min; and SpO2, 99%. What would be most appropriate action to take when caring for this child

Nonemergent transport to the hospital

Which bone injury would be said to occur in the appendicular skeletal system?

Right-hip fracture

The EMT is caring for a patient who has a problem with the autonomic component of his nervous system. Which sign or symptom would most likely be caused by this condition?

Slowing of the heart rate

On the scene of a motor vehicle collision, which EMT is engaged in simple extrication of the patient?

The EMT removing the patient from the car through an open door

Which assessment findings related to the mental status of a 1-year-old would the EMT consider normal?

The patient is curious about your penlight

A patient who was injured in a motor vehicle collision has a separation of the rib and its associated spinal vertebrae. Based on the anatomy of the spine, where has this injury occurred?

Thoracic spine

You are transporting a 44-year-old male patient who was stung multiple times by fire ants. Which of these statements made by the patient should concern you most?

"I am beginning to feel itchy all over and I'm developing hives."

A partial thickness burn covering 8 percent of the body would be considered moderate for which patient?

A 25-year-old patient with sickle cell disease

Which patient has the best chance for an optimal outcome after drowning?

A 34-year-old female who was submerged for 9 minutes in 39° F water

Your service has acquired the antidote for a nerve agent attack. By which route will this antidote most likely be given

Injection

The EMT would use the sterile scissors found in the OB kit to cut:

The umbilical cord

You have been called to a residence for a complaint of chest pain. On scene, you are met by the patient's husband, who informs you that his wife suffers from anxiety attacks and that she received bad news tonight concerning her mother. This news increased her anxiety, and now she just needs to calm down. According to the husband, her doctor said the chest pain the patient gets is just a temporary response to the anxiety. Assessment of the 46-year-old woman reveals her to be extremely anxious and complaining of chest pain. In treating this patient, the EMT should

Check vital signs

You have been called for a deeply disturbed psychiatric patient who has cut off the tip of his penis with a steak knife. Assessment reveals the 31-year-old male to be crying, with blood noted to his hands and pants. The primary assessment reveals no acute life threats. At this time, your priority is to

Control any active bleeding that may be occurring

When cutting the umbilical cord, how should the baby be positioned

Level with the mother's uterus

A roofer on top of a two-story house called 911 after experiencing chest pain and dizziness. You are providing care to the patient on top of the roof, and the fire department has arrived to help remove the patient from his present location. As the patient is brought down the ladder, your highest priority as an EMT is:

Your personal safety

The EMT shows that he understands the seriousness of a prolapsed umbilical cord when he states:

"A prolapsed cord that is pinched can stop the flow of oxygen to the baby and must be addressed immediately."

The EMT shows that he can accurately differentiate placenta previa from abruptio placentae when he states:

"Bleeding associated with abruptio placentae is typically associated with abdominal pain; bleeding associated with placenta previa is painless.

The EMT is properly assessing for sensory function in the hands when he tells or asks the patient:

"Can you tell me which finger I am touching

A male patient in his thirties fell 1 O feet off a loading dock, landing on his head and back. He has deformity and depression to the back of his head and is unresponsive with snoring respirations. Vital signs are pulse, 132 beats/min; respirations, agonal; blood pressure, 136/64 mmHg, and SpO2, 89%. The patient responds to painful stimuli with purposeful motion. Which instruction would you provide other caregivers at the scene?

"I need someone to start positive pressure ventilation at 12 breaths per minute

After a 29-year-old male with chest pain is removed from the house by stretcher, the man's 7-year-old son throws a temper tantrum. His mother is embarrassed and states that he has not done this since he was 2 years old. Given the situation, your response would be:

"I would not be concerned. This is a stressful event he doesn't understand, and he may just be having trouble coping with it.

You have been called to a residence for an unknown medical problem. As you enter the house, which scene clue would most likely suggest the use of home medical oxygen

"No smoking sign" posted on the front door

You are caring for a patient with an extensive freezing cold injury to his right hand and arm. Which of these instructions would be appropriate to give to your EMT partner

"Why don't we get that ring off his right ring finger."

Assessment of a newborn indicates that she has slow and irregular respirations with a weak cry. You should award her how many points according to the Apgar scoring system?

1

When asked by a concerned newborn parent, the EMT should indicate that an acceptable respiratory rate for the newborn is:

52 breaths per minute

You are determining the PGCS score for a 28-month-old female who was stung by a bee while playing in the backyard. You note that the patient is alert. When asked, she tells you that she was stung on the bottom of her foot. At this point in lime her GCS score would be:

15

Starting at which age should the EMT consider transporting any injured patient to a trauma center

55 years of age

What would the minimum expected preductal SpO2 reading be at one minute after birth?

60%

What would be the calculated minimally acceptable systolic blood pressure for a 9-year-old girl who has a significant bleed following an injury involving broken glass

88 mmHg

When assessing a geriatric patient for pneumonia, the EMT must remember that:

A normal body temperature may still be observed in this patient

A patient exhibits retrograde amnesia when she

Cannot remember falling and hitting her head

Which type of injury is most likely to result in capillary bleeding only

Abrasion

Which chemical agent will generally kill a person most quickly

Aerosolized vesicant agent that is inhaled into the lungs

Since supplies and materials used to treat the sick or injured may be used on every call, it is good practice to replace or restock the ambulance

After every call

You are driving an ambulance with antilock brakes. A deer darts out in front of the ambulance. Your most appropriate action would be to

Apply constant and firm pressure to the brakes

When relying on your senses to determine whether a hazardous spill has occurred, you should

Assume that the area around any leak is hazardous

While on standby at a semi-professional baseball game, you are summoned onto the field at home plate for a player who complains of severe leg pain after colliding with the catcher of the opposing team. The primary assessment shows no threats to the airway, breathing, or circulation. The secondary assessment reveals a severely deformed knee that is swollen and ecchymotic. The leg is pale and cool, and the patient cannot move his leg when asked to do so. In addition, you cannot palpate a pedal pulse. What should the EMT's priority action be at this time?

Attempt to straighten the leg until a pulse returns

You are transporting a very sick 3-year-old with suspected septic shock. His airway is patent, respirations tachypneic, and radial pulse weak, with skin that is hot to the touch. Vital signs are pulse, 152 beats/min; respirations, 32 breaths/min; blood pressure, 94/64 mmHg; and SpO2, 94% on a nonrebreather face mask. The patient has a rectal temperature of 103.9° F. Tylenol has been administered. During transport to the hospital, you should be most concerned with which clinical finding?

Blood pressure of 84/32 mm Hg

Which statement concerning a flash burn is accurate?

Clothing will typically protect the skin from a flash burn

The EMT would suspect that a patient has a traumatic brain injury (TBI), as opposed to post-traumatic stress disorder (PTSD), when he observes:

Confusion and disorientation

You are assessing an elderly male patient who was trapped in a burning trailer. Which finding would be most indicative that he has an inhalation injury?

Coughing of black sputum

Which option best illustrates a basic medical supply that must be carried on an ambulance

Exam gloves

You are called by a husband who states that his wife is bipolar and in need of help at the hospital. Which presentation would make the most sense related to this disorder?

Depressed and not wanting to talk

A young boy riding an ATV did not see a chain stretched between two poles and hit it with his neck while traveling forward. Subsequently, he was thrown backward from the vehicle with significant force. He was not wearing a helmet. Emergency Medical Responders are with the patient and are holding manual spine motion restriction as well as administering oxygen. When you are assessing this patient, which sign would indicate the injury that must be addressed and managed first

Difficulty speaking

Emergency Medical Responders have removed an intoxicated and confused patient from a car that struck another vehicle at a high rate of speed. The patient was unrestrained and his car did not have air bags. Assessment reveals deformity bilaterally to the hips and femurs and bruising to the knees. Based on this injury pattern, the EMT would recognize which type of injury mechanism?

Down and under

An intraventricular shunt benefits a patient by:

Draining cerebrospinal fluid from the brain

Your partner believes the 90-year-old male you are caring for is dehydrated. Which finding best reinforces his suspicion?

Dry mouth and tongue

A young female complains of sharp pain to the left lower quadrant of her abdomen. She states that her last period was seven weeks ago. With this information, the EMT should be suspicious of:

Ectopic pregnancy

A hearing-impaired patient with a hearing aid is having a difficult time understanding your questions. Your initial action would be to:

Ensure that the patient's hearing aid is properly placed and turned on

A fair-skinned patient was exposed to the sun and has a painful superficial burn to her upper back. As a knowledgeable EMT, you would recognize that which layer(s) of the skin has been injured

Epidermal

A patient has sustained an injury to his mediastinum. Based on the anatomy of his chest, which structure has been injured?

Esophagus

You have arrived on the scene of a stabbing. As you approach the patient, you note that his shirt is off and a single stab wound to the right lower quadrant of the abdomen is evident. The patient appears unresponsive as you approach. At the patient's side, your first action should be to

Examine the patient's airway

Rescue workers are preparing to remove the windshield and roof from a car to extricate a patient who is trapped. Before the rescue commences, what should the EMT do first, prior to the other actions?

Explain to the patient what is about to take place

Which of the following findings in a patient with a traumatic mechanism of injury is sufficient to prevent the EMT from "clearing the spine" and necessitate the implementation of spine motion restriction precautions?

GCS score of 12

You are transporting a female patient who was sexually assaulted by having a foreign body inserted deep into her vagina. You are attempting to control hemorrhage by the appropriate means. As you reassess the patient, which finding is of most concern

Heart rate increases from 116 to 140 beats/min

In a normal fertilization, the egg and the sperm meet in which structure?

Fallopian tube

What is the best description of the goal of a trauma system?

Getting the right patient to the right facility in the right amount of time

You are assessing a 29-year-old pregnant female who has signs of imminent delivery. Her past medical history includes HIV infection. Compared to a patient with no infectious disease, how will standard precautions differ for this patient?

Gloves, a gown, and eye protection should be worn for both the patient with HIV and the noninfectious patient

You are called for an 8-year-old boy who has fallen. At the scene, the boy's mother states that her son was running while flying a kite and tripped, striking his face on a rock. Assessment reveals a large reddish-blue lump with intact skin under his left eye. When alerting the hospital emergency department of your arrival, you should inform them that the patient has which type of injury?

Hematoma

A concerned parent asks you which vaccine will decrease the risk of her child getting epiglottitis. Which vaccine would you tell her?

Hib vaccine

In reviewing prehospital care reports, you note that more than 75 percent of the patients transported from a local geriatric extended care facility have been diagnosed with hypertension. As a knowledgeable EMT, you would recognize that:

Hypertension is common among the elderly

Consumption of large amounts of water during prolonged periods of exertion can result in which disorder

Hyponatremia

How can an EMT best protect herself, as well as get the most appropriate resources, at the scene of a multiple-casualty incident involving a weapon of mass destruction

Identify the specific agent or mechanism used

When a second-story deck collapsed, several patients fell from the deck. They are now lying on the ground below. As the first emergency personnel to arrive on the scene, the initial priority for you and your partner is to determine

If the scene is safe to enter

At a multiple-casualty incident, a crew brings you a patient with a red tag tied to his wrist. What does this tag indicate

Immediate care and transport are necessary

The signature wound of recent combat missions in the Middle East is typically caused by

Improvised explosive devices

What is the best location for assessing breath sounds in the pediatric patient

Midaxillary region of the lungs

A patient with hypertension and diabetes weighs 190 pounds. His ideal weight is 160 pounds. This patient would best be described as

Obese

The EMT is correctly assessing a child's BRUE when he

Obtains a thorough history

After delivery, you note blood still escaping from the mother's vagina. The sanitary pads that you placed earlier between her legs continue to become soaked. Your next action would be to:

Perform a uterine massage

A young adult male has a gunshot wound to the left side of his head. He is unresponsive with snoring respirations. He is breathing 6 times per minute with a pulse rate of 52 beats/min and a blood pressure of 192/104 mmHg. His radial pulse is strong and his skin is cool, but not diaphoretic. In addition to spine motion restriction precautions, the first intervention for this patient would be:

Performing a jaw-thrust maneuver

Which mechanism of injury produces the conditions for traumatic asphyxia?

Person pinned between a truck and wall of a building

The EMT is by the side of a young female who was cut while washing dishes. She has a jagged laceration to her left forearm that is bleeding steadily and heavily. When attempting to control the bleeding, the EMT would first:

Place a large sterile dressing over the site, with hand pressure applied over the top of the dressing

You have been dispatched for a young female in labor. On arrival, you find an arm protruding from the vagina. What should you do next

Place the mother in a knee-chest position

A 31-year-old female is in labor. After ensuring her airway, breathing, and circulation, you assess her perineum and observe the umbilical cord protruding from the vagina. However, the baby is not visible. Your next action would be to

Place the patient in a knee-chest position

On scene, a young male experiencing a behavioral emergency talks very rapidly and constantly interrupts you while you attempt to ask him questions. As such you would recognize that the patient is exhibiting

Pressured speech

Protection of the EMT from chemical agent exposure requires

Proper training and equipment

A young man was riding a motorcycle when he was hit head on by a car and thrown from the bike. Your scene size-up reveals him to be lying supine in the roadway with obvious deformity to his right thigh and left ankle, with blood noted on his pants in these areas. The patient assessment reveals he is unresponsive with snoring respirations and is breathing irregularly at 16 times per minute. His radial pulse is moderate in strength, and his skin is warm and dry. Which action should be performed first by the EMT?

Provide manual cervical spine motion restriction

You have been called for an unknown medical emergency at a residence. On scene, you are presented with an 84-year-old patient who is confused and does not obey commands. The best means of determining the patient's normal mental status would be to

Question family members

You arrive at the scene where a young male patient fell out of a tree he was climbing. Although he fell from a lower branch, he was still reportedly 10-12 feet in the air when he fell. The patient landed on his side on the ground. Which element of the kinematics of trauma would explain the mechanism of any injuries he may have sustained?

Rapid deceleration

41. If you are first on the scene of an MCI, it is imperative that you:

Recognize whether it is a terrorist attack early on

When assessing a patient undergoing dialysis, including his AV graft, which finding would be of greatest concern to the EMT?

Red and hot skin at the graft site

In a patient with conjunctivitis, the EMT would expect to observe a/an:

Reddened membrane over the sclera

Nerve agents typically cause death by triggering which type of organ or system failure

Respiratory

An 8-year-old boy fell forward off of a swing onto outstretched arms. He did not hit his head or lose consciousness and is complaining of left forearm pain. There is deformity to the left wrist, but no other injuries or life threats. His vital signs are pulse 132 beats/min, respirations 20 breaths/min, blood pressure 108/62 mmHg, and SpO2 100%. The proper care of this patient would be to

Secure the forearm to a padded board splint and apply a cold pack to the injury site before ransporting the patient nonemergently

At the scene of a motor vehicle collision. a driver is entrapped in his car after striking the back of a semi tractor-trailer at a high rate of speed. Which action would be performed first when providing stabilization of the car?

Shut off the engine

A young boy complains of pain to his fingers after spending several hours outside riding a sled in cold winter temperatures. After ensuring he has no life-threatening conditions, you turn your attention to his hands and note that the fingers are cold to the touch. You suspect that he is suffering from a superficial freezing cold injury. Which sign or symptom would help confirm this suspicion

Skin on the fingers that is soft and numb

A female patient with a maxillary fracture has sustained injury to her

Upper jaw

What would cause you to start transporting a critically injured patient to a local hospital (not equipped for major trauma) instead of waiting on a helicopter for rapid transport to a trauma center 50 miles away

The beginning of lightning and poor weather conditions

A patient complaining of eye pain informs you that he was diagnosed with a corneal abrasion three days ago. He called 911 today because the discomfort has not yet resolved and he desires medical attention. Based on this information, which statement is most accurate?

The clear structure that covers the pupil has been damaged

he young pediatric patient is more prone to head injury because:

The head is proportionately heavier and the neck muscles are weaker

Which statement shows that the EMTs are correctly using a vest-type device while removing a patient from an entrapped vehicle location?

The head is secured to the device after the torso has been secured

When applying the rule of nines to a 10-month-old child, the EMT should recognize which statement to be accurate?

The head of a child is considered to be 18 percent of the total body

When faced with the possibility of delivering twins, the EMT must remember that

The second infant may be born breech

A 31-year-old female in her third trimester of pregnancy lost her balance and fell down a flight of stairs. Treatment included spinal motion precautions and oxygen therapy. When transporting her, the EMT should

Tilt the long spine board to the left

What is a vagus nerve stimulator used for

To prevent seizure activity

Bystanders have called 911 for a female patient who lives on the street. She has been outside in the cold all night and is muttering incomprehensible words. Her airway is open and her breathing is adequate. Her skin is cold, and a radial pulse is present but weak. What should the EMT do next

Transfer the patient to the ambulance

When managing a patient with a soft tissue injury, it is essential that the EMT performs which action?

Use the appropriate personal protective equipment

A 24-year-old male has cut his arm with a table saw. Assessment reveals dark red blood flowing steadily from the laceration. As an EMT, you would identify this bleeding as

Venous

A heavy dresser fell onto a 76-year-old male's foot as he attempted to move it across his bedroom floor. His right foot and ankle are bruised, but the patient is refusing treatment or transport since it does not hurt. Which statement made by the patient during your assessment would be most concerning and warrant contact with medical command for further advisement

"I have been told I have peripheral neuropathy."

When moving a patient with severe generalized hypothermia, which of these instructions is most appropriate to relay to your fellow EMS providers?

"I want everyone to take extra care in moving the patient very gently to the stretcher. We do not want her to go into cardiac arrest."

You are preparing to transport a patient with a behavioral emergency to the hospital. As he is being loaded into the ambulance, he smiles and says, "What if I decide to just go crazy and hurt you during the ride to the hospital?" What is your best response?

"If you touch or hurt me, you will be physically restrained immediately. No exceptions."

Which statement about the use of an escort, such as a police cruiser, to drive a critical patient to the hospital is true

"It should be used as a last resort, because it increases the risk of an accident."

A 24-year-old man was shocked while working on his house's electrical system. Assessment reveals a minor burn to the fingers on his left hand. Aside from being slightly shaky, the patient states he feels fine and does not see the need to be treated further or transported to the hospital. Which statement would you make before allowing him to refuse additional care and transport

"It would be best to let the emergency physician check you out. Electrical shocks can damage the heart or other internal structures."

You arrived on scene at a local restaurant, where you encounter a highly agitated combat veteran who is yelling at patrons. Given the public nature of the scene and the patient's known history of post-traumatic stress disorder (PTSD), which statement indicates appropriate initial management of the situation?

"Let me see if I can establish a rapport and calm him."

When caring for a 34-year-old male with a history of post-traumatic stress disorder (PTSD), which instruction should you provide to your partner

"We need to determine whether the patient feels suicidal or homicidal."

Which instructions would you provide to the Emergency Medical Responders when treating a patient who was punched in the eye and has blood visible in the anterior chamber

"We will need to cover both eyes with patches before we transport the patient."

The EMT shows that she understands the concept of driving with due regard when she states

"When approaching an intersection, I wait for all traffic to stop before proceeding through."

When asked, a young female with abdominal pain replies that she is not sure if she is pregnant. Given this response, which question would provide the best information to determine if the patient may be pregnant?

"When was your last normal menstrual period?"

You are assessing a military veteran who complains of a headache, visual disturbances, and generalized anxiety. You suspect that he may have post-traumatic stress disorder (PTSD). Which question would be most appropriate to obtain additional information regarding the possibility of PTSD

"Where did you see combat?"

Which of these patients with heat emergency would be the highest priority for transport?

A 19-year-old responsive to verbal stimuli with hot dry skin and tachycardia

Given the research related to suicide, which patient should the EMT recognize as the highest risk for a suicidal action?

A 72-year-old widow who was just informed of a diagnosis of Alzheimer's disease

Dementia is typically observed as

A chronic and irreversible condition

What is one of the most common criminal uses of hazardous materials that the EMT is likely to encounter

A methamphetamine lab

You have been summoned to a grocery store for a 6-year-old girl who has collapsed. On scene, you find the girt lying unresponsive on the floor. You quickly open the patient's airway and determine that she is not breathing. When you try to ventilate her with the bag-valve mask, you are unable to do so. Repositioning the head and reattempting ventilation is not successful in passing air into the lungs. What would your next action be?

Administer 30 chest compressions

A 51-year-old male was burned in the face after a steam valve was accidentally opened at his industrial job. His face is red and swollen, and he is screaming in pain. The patient also states that he is having a hard time seeing. What is the EMT's priority concern in caring for this patient?

Airway compromise

You are positioning an obese patient who weighs 360 pounds on a stretcher for transport to the hospital. No other life threats exist and the vital signs are stable. Given that the patient is complaining of a headache, which transport position would be most appropriate?

Allow the patient to assume a position of comfort

You have been called to a residence of a patient with diabetes who exhibits an altered mental status. A family member states she could not reach the patient by telephone, so she came over and found the patient awake but confused. The patient can remember his name and address, but cannot remember the day or year. He refuses to go to the hospital, but consents to an assessment. Of these physical exam findings discerned on assessment, which one is most concerning

Bruise to the left temple

A 68-year-old female patient complains of a headache and generalized weakness. Her husband informs you that she was with a friend yesterday and was involved in a motor vehicle collision. She did strike her face on the dashboard, but refused treatment on scene. Her past medical history includes a stroke with right arm weakness and high blood pressure, for which she takes medications. When assessing this patient, which finding should concern the EMT most?

Bruising behind her left ear

You suspect that your patient-a lethargic 6-month-old baby-is the victim of child abuse. When performing the primary assessment, which assessment Parameter should you use to quickly establish his perfusion

Capillary refill

You have responded to a bar for a fight involving approximately 8 to 10 patrons. Weapons were used in the fight, and all the patrons have varying degrees and types of injuries. You are in charge of triage and must use the Simple Triage and Rapid Transport (START) system. You find a young person lying on the floor with blood covering his shirt. Quick assessment reveals him to be breathing at 24 times per minute. What should you do next

Check for a radial pulse

A restrained driver was in a car that struck another car from behind at a low rate of speed. The driver has a bruise to her forehead and complains of tingling in her left leg. She is lying on the side of the road, and another EMT has established manual cervical spine motion restriction precautions. A quick scan of her as you approach indicates that the patient is in little to no distress. What should you do first

Check her breathing and radial pulse

A patient has terminal lung cancer and a valid Do Not Resuscitate (DNR) order. His wife has called 911 because her husband's pulse oximeter reading fell below 85% and an alarm sounded. The patient is currently receiving home oxygen through a nasal cannula. Assessment reveals him to be alert and oriented and slightly tachypneic with no abnormal lung sounds. At this time, your next action would be to:

Check the SpO2 using your pulse oximeter

Your partner is assessing a patient who has a history of antisocial disorder and just got into a heated verbal exchange with his neighbor. He threatened to kill his neighbor, and 911 was called. As you observe the patient, what is the strongest indication that the patient may be ready to have a violent outburst

Clenching of both fists

A patient has experienced partial- and full-thickness thermal burns to the anterior chest, abdomen, and arms in an industrial accident. Which action would be appropriate in the care of this patient?

Cool the burns with water and cover the burns with a sterile, dry burn sheet

You are by the side of a 2-year-old male whose mother states he has a cough that has slowly worsened over the past three days. The patient's airway is patent and respirations labored. lntercostal retractions are accompanied by stridorous respirations. His skin is very warm to the touch. Vital signs are as follows: pulse, 152 beats/min; respirations, 28 breaths/min; SpO2, 97%; and temperature, 100.3°F. Breath sounds are clear and equal. Based on this presentation, you would suspect and treat the patient for

Croup

A patient was trapped in a burning apartment before being rescued by firefighters. As you approach him, you note that his legs and arms are burned. The patient is screaming in pain and coughing, but no longer actively burning. Given this information, which intervention would you perform first?

Decide whether to administer oxygen with a nasal cannula or nonrebreather mask

You have been called to a lower-income housing complex for a 1 ½-year-old child who is short of breath and has a fever. When assessing this patient, which technique would be appropriate?

Determining the respiratory rate before laying hands on the child

You are by the side of a young male patient with a history of behavioral problems. Friends called 911 after the patient experienced several violent outbursts this evening. Which finding would cause you to suspect a physical cause for this behavior, as opposed to a mental one?

Dilated pupils that are sluggish to react

What are the stages of labor in the order in which they occur?

Dilation, expulsion, and placental delivery

At the scene of a multiple-casualty incident caused by a weapon of mass destruction, what is your first priority as an EMT

Ensuring the safety of yourself and your partner

How can cyberterrorism impact medical equipment

Equipment can be exploited electronically to cause patient harm

You have been dispatched to a pool party for a 19-year-old male with shoulder pain. He states that he dove off the diving board and hit the bottom of the pool with his right shoulder and back. Although there is alcohol at the party, he denies drinking. The patient is alert and oriented and has obvious redness and abrasions to his right shoulder, neck, and back. He complains of neck pain and tingling in his right arm, but denies any other discomfort to any other part of the body. Your next action would be to

Establish manual spine motion restriction

You and your partner are the first emergency personnel on the scene of an overturned tanker truck. The driver appears motionless in the cab of the vehicle. You also note liquid leaking from the rear-valve assembly of the tanker trailer. After properly positioning your ambulance, your immediate priority is to

Evacuate people from cars near the tanker

A 49-year-old male has summoned EMS because of a problem with his tracheostomy tube. At the patient's side, you note the tracheostomy tube to be protruding abnormally from the stoma in his neck. Your first action in caring for this patient would be to

Evaluate the adequacy of the patient's airway and breathing

On the scene of a hazardous materials emergency, which individual is best suited to provide command and coordination of assets at the incident?

Hazmat specialist

You walk into the station at the beginning of your shift and see another EMT laundering sheets and pillowcases used from the previous shift. Which observation should cause you to stop and correct the EMT in relation to this task

He is using an EPA-registered sterilant as the detergent

You are treating a 9-year-old male who is short of breath and has a history of asthma. At the scene, he presents in severe respiratory distress, with intercostal retractions, lethargy, and expiratory wheezing. His pulse is 136 beats/min, respirations are 32 breaths/min, and SpO2 is 95%. You have assisted the patient with his albuterol MDI and are transporting him emergently to the hospital. As you reassess the patient, which finding is most concerning?

Heart rate of 68 beats/min

You are on the scene of a motor vehicle collision involving two cars. To remove the driver of the first car, rescuers must break the windshield and roll back the roof. While assisting in this operation, which pieces of personal protective equipment will afford you the best protection?

Helmet, goggles, turnout gear, leather gloves, and steel-toed boots

You have been called for an alert and oriented male patient. Upon your arrival, his roommate states that the patient spent several hours locked out of his house in the cold temperatures after an evening of drinking since no one was home and the patient lost his house keys. Since the primary and secondary assessments have ruled out immediate life-threatening conditions, you have decided to rewarm the patient. Which of these warming techniques would be most appropriate for this patient

Hot packs to the patient's chest, groin, and armpits

A 23-year-old diver has been brought to the surface by fellow divers after behaving erratically while in the water. Your assessment reveals him to be confused and uncoordinated, with an open airway and adequate breathing. He has a strong radial pulse and SpO2 of 98%. The patient has diabetes, and your partner reports that his blood sugar is 91 mg/dl. The remainder of your assessment is unremarkable. Based on this presentation and history, what would be the most likely cause of his condition

Increased nitrogen levels in the blood

What is the most common reason for an EMS response to a patient with an implanted VAD?

Infection

You have been called for a 4-year-old female who is short of breath. Her mother informs you that she has had a fever and runny nose for two days and today began to have difficulty breathing. Furthermore, the mother is having a difficult time waking her daughter up. You find the girl lying in bed with snoring respirations despite use of the head-tilt, chin-lift airway maneuver. Your assessment reveals her to be responsive to painful stimuli and breathing at 12 times per minute with minimal chest and abdominal rise and fall. Her skin is warm to the touch with cyanosis around the lips. Your partner informs you that her heart rate is 124 beats per minute. What is your immediate action in caring for this child

Insertion of a nasal airway

If a patient has severe retroperitoneal bleeding from an organ located within this cavity, she has most likely injured her

Kidney

A traumatic brain injury (TBI) would most likely be caused by

Nearby explosion of a concussion grenade

The high pressure alarm is sounding on a home ventilator. As your partner prepares the bag-valve mask, you quickly determine that no problem exists with the machine itself. As positive pressure ventilation is initiated, you note high resistance to the ventilations and have a difficult lime adequately squeezing the bag. Your next action would be to:

Not change the ventilator or settings, but rather summon ALS for on-scene backup or en route intercept

A patient was burned when he accidentally ignited gasoline while working on his car. When performing your assessment of the injury, which finding would indicate that the patient has suffered a partial-thickness burn?

Painful skin on the abdomen that is moist and contains blisters

A combat veteran patient with a history of severe PTSD states that he has a headache that feels worse than any other he has ever had. The patient is alert and oriented, with no threats to the airway, breathing, or circulation. His vital signs are pulse, 78 beats/min; respirations, 16 breaths/min; blood pressure, 168/100 mmHg; and SpO2, 95% on room air. What would your next action be

Perform a neurologic assessment

A patient has been struck by lightning at a picnic. A doctor on scene states that the patient was in cardiac arrest after the strike, but with 1 minute of CPR, has a heartbeat and weak sonorous respirations. The patient remains unresponsive. Which priority care should the EMT provide

Perform the jaw-thrust maneuver

While cleaning a gun, a 44-year-old man accidentally shot himself in the abdomen. On your arrival at the scene, the patient is responsive to painful stimuli

Perform the primary assessment, administer supplemental oxygen, perform the secondary assessment, transfer the patient to the stretcher, and provide rapid transport

A paramedic has asked that you apply a dressing over a sucking chest wound. Which option is the best choice to cover this injury?

Petroleum gauze

You are in charge of triage at a scene involving 32 patients exposed to carbon monoxide in a church basement. You find a 6-year-old boy who is not breathing. You open his airway and find that breathing does not return. Furthermore, you cannot locate a radial pulse. You should

Place a black tag on this patient and move to the next patient

You have been called to assist another crew with the birth of a baby. On scene, another EMT informs you that the mother's perineum tore and is bleeding heavily. You would:

Place a sterile dressing between the mother's vagina and rectum

A vehicle has been struck in its front end by another car. The driver is entrapped, and the roof must be rolled back to allow extrication. The car has been appropriately stabilized and you are inside with the patient. The primary assessment is complete and a cervical collar applied. Rescue personnel are ready to begin cutting the vehicle. At this time, the appropriate action is to

Place a tarp over the patient

A pregnant 31-year-old female has been shot in the neck. Currently, she is responsive to verbal stimuli, with an open airway and rapid, but adequate respirations. Her radial pulse is weak and fast. Vital signs are as follows: pulse 152 beats/min, respirations 22 breaths/min, blood pressure 92/76 mmHg, and SpO2 100% on high-concentration oxygen as initiated by the fire department EM Rs. Which other intervention by the EMT will most benefit this patien

Place an occlusive dressing on the neck and keep the patient warm during transport

You are securing a 4-year-old-boy on a long spine board during a spine motion restriction process. Which action would be appropriate when performing this intervention?

Place padding between his shoulders/back and the spine board

You have arrived at the side of a 35-year-old female who is 37 weeks pregnant and in labor. Your exam reveals crowning with contractions 60 seconds apart. Your immediate action would be to

Place the patient on her back, and then open and prepare the OB kit

A patient is in cardiac arrest after being submerged in a pond for 15 minutes. You note that his abdomen is greatly distended and it is very difficult to ventilate him. What should you do next?

Place the patient on his side and firmly press on the abdomen

Severe external bleeding should be controlled during which phase of the patient assessment

Primary assessment

You have been called for a 27-year-old female who is 32 weeks pregnant. She states that she is experiencing uterine contractions but feels no pain with them, just a tightening in her abdomen. The contractions began three days ago and seem to be increasing in frequency today. She denies discharge or rupture of her membranes. Given this presentation, you should recognize:

Probable false labor, but have the patient transported to the hospital for evaluation

You have been called to an apartment for a child who is sick. As you approach the 4-year-old boy, you note that he is sitting upright in his mother's arms with his chin thrust forward. He has a sickly appearance and is drooling. His airway appears open and his breathing is adequate. His radial pulse is strong and his skin is hot to the touch. Which intervention would be most appropriate in the care of this patient first?

Provision of supplemental oxygen via a nonrebreather mask

The EMT is correctly using the pediatric assessment triangle on a 3-year-old male trauma patient when he:

Quickly determines the circulation based on skin color

You have been called for a 91-year-old female who fell. Since your assessment reveals bruises to her body in various stages of healing and inconsistencies in information provided by her family, you are suspicious of geriatric abuse. What would your best action be

Quietly transport the patient and inform the emergency department physician

What is the most significant mechanism by which the body can lose heat?

Radiation

A frantic mother dialed 911 for her 3-year-old daughter, who stuck a small, dried bean into her ear. Assessment shows the bean to be lodged firmly in the ear canal. When treating this patient, the EMT would

Reassure the mother and transport the patient to the emergency department

You have been trained to the OSHA level of "First Responder Awareness." With this level of training, which activities are you qualified to perform

Recognize a potential hazardous materials emergency and call for the appropriate resources

An 84-year-old female fell down six steps, landing on carpet. She now complains of abdominal and pelvic pain. Her airway is open, breathing adequate, and radial pulse intact. Skin is cool and dry, with a noted skin tear to the left elbow with minor bleeding. EMRs report the following vital signs: pulse, 84 beats/min; respirations, 20 breaths/min; blood pressure, 126/68 mmHg; and SpO2, 97% on room air. Her family states that the patient is acting normally. The hips are easily moved and do not appear broken. The patient does not want to go to the hospital and is asking to refuse transport. Your next action would be to:

See which medications she is taking

A female patient is described as primigravida. Consequently, the EMT would recognize that:

She is pregnant for the first time

On the scene of a medical emergency, you are directed to a 5-month-old baby in his crib. He responds to painful stimuli by whimpering and is breathing at a rate of 50 breaths per minute. His airway is open, and his brachia! pulse is fast, weak, and difficult to locate. Assessment of his skin reveals it to be cool and pale in color. According to the mother, her child has had vomiting and diarrhea for four days. She also reports that the father of the baby has diabetes. Based on this information, the EMT would recognize which condition?

Shock

A patient has been critically hurt in a nighttime motor vehicle collision. She was wearing a seat belt and hit a tree at 70 mph. The car she was driving weighed 2 tons and was equipped with air bags that did deploy. Which factor had the greatest impact on the injuries she sustained?

Speed of the vehicle at impact

A 49-year-old male has been stabbed in the lower right chest. Police tell you that the patient got into an argument with an unidentified man, who then stabbed him before fleeing the scene. After assessing the patient, you are suspicious that the knife punctured the lung and is causing internal bleeding. In this situation, the mechanism of injury would be:

Stab wound to the right chest

Which one of these injuries is most likely to cause peritonitis?

Stab wound to the small intestine

Despite positive pressure ventilation, a term newborn's heart rate has decreased from 80 beats per minute to 40 beats per minute. He is breathing spontaneously but shallowly at 40 breaths per minute and has a mottled appearance. Your next action would be to:

Start external chest compressions

A 6-year-old male is in severe respiratory distress with inspiratory strider. He has pale, moist skin, with cyanosis noted around his mouth. The patient is very lethargic and cannot hold his head up. His mother is panicked and reports that he has a history of asthma. Breath sounds are diminished and his vital signs are pulse, 162 beats/min; respirations, 40 breaths/min; blood pressure, 122/66 mmHg; and SpO2, 81%. What would your immediate action be?

Start positive pressure ventilation

While assessing a patient with altered mental status, you note a catheter-like device projecting from the center of the patient's pelvic area. This is most likely a(n)

Supra pubic catheter

An 82-year-old female with nausea and vomiting has called 911. When obtaining her past medical history, she informs you that she suffered a transient ischemic attack (TIA) two months ago. Based on this information, what should the EMT expect to find?

The absence of any neurologic deficits

While in an apartment to provide care for a 52-year-old female with shortness of breath, you note a baby sleeping in a nearby crib. Which observation related to the baby would prompt you to intervene and speak to the family?

The baby is sleeping on her abdomen

Which characteristic do hypergolic substances display when they are combined in a WMD?

They spontaneously ignite when combined.

Which device can be used to replace the long spine board when providing full spine motion restriction precautions to an adult

full vacuum mattress

Which one of the statements shows that the EMT has a good understanding of dealing with a behavioral emergency patient?

"It is okay to spend an extended amount of time on scene before transporting patients with behavioral problems

Which instructions would you provide to a new EMT who is preparing to assess a stable 9-month-old boy who has a rash

"Allow the mother hold him as you do the assessment."

When obtaining a history of a 5-year-old boy with generalized hypothermia, which question should be asked before all others

"How long was he out in the cold?"

The EMT shows an understanding of newborn resuscitation when she states:

"Interventions in a newborn should be performed for 30 seconds followed by reassessment."

A patient has been bitten by an unidentified snake. Which of these statements made by the patient would lead the EMT to believe that the snake was poisonous

"Its head was triangular."

When preparing to provide positive pressure ventilation to a child, what will be an acceptable tidal volume if the child weighs 22 kg?

154 mL

While examining a pregnant patient who has not received prenatal medical care, you note that you can palpate the uterine fund us at the level of the umbilicus. What is the approximate gestational age of the fetus?

20 weeks

Which statement about peritoneal dialysis is true?

A catheter is inserted into the abdomen during peritoneal dialysis

You are maintaining manual cervical spine motion restriction for a patient who is being log rolled, transferred, and secured to a long backboard. At which point will you release the manual spine motion restriction hold

After the patient has been properly secured with straps to the long board

You have been called for an 87-year-old male who fell in his kitchen while making breakfast. The patient states that he hit his forehead but managed to catch himself on the way down and did not hit the ground "too hard." Your assessment reveals him to have kyphosis of the thoracic and cervical spine, but no neurologic deficits. His legs, however, "feel weaker." Which action is appropriate given these assessment findings?

Apply a cervical collar and prepare the patient for any discomfort that the collar may cause

A construction worker has a metal rod impaled in his right forearm. Assessment of the injury indicates heavy bleeding from around the impaled object. What should you do first?

Apply direct pressure around the rod

In planning for a response to an attack with a weapon of mass destruction involving an incendiary device, the EMT should be prepared to handle which types of injuries?

Burns

A patient with quadriplegia most likely experienced an injury to his

Cervical spine

While a factory employee is moving a container of a radioactive liquid, some of the liquid splashes onto his arm. This situation is best be described as

Contamination

The EMT is properly assessing the breathing of a 3-year-old when he:

Counts the respiratory rate for 30 seconds and multiplies by 2

The EMT is correctly assessing a child's apical pulse when he:

Listens with a stethoscope to the child's left side of the chest

A malnourished child would be an example of

Neglect

Assessment of a pregnant patient in labor reveals what appears to be the baby's buttocks presenting at the vaginal opening. The EMT would immediately:

Place the patient in supine, head-down position, with hips elevated, and administer

A small metal rod was thrown from a metal lathe while it was in operation and is now impaled in a man's inner thigh. Assessment indicates that there is no active bleeding from the site, but the surrounding tissues are ecchymotic. The paramedic asks you to stabilize the rod with bulky dressings. You recognize this action is important because it will:

Prevent motion of the rod and further internal injury

A patient has received burns to his arm from a beta particle exposure. This type of damage would be classified as A- Radiation poisoning B- Radiation sickness C- Radiation injury D- Radiation exposure

Radiation injury

After the unsuccessful resuscitation of a 3-year-old boy who was shot by his mother's boyfriend during a domestic dispute, you find yourself extremely stressed and anxious over the call. You are having trouble sleeping and the resultant anxiety is affecting your home life. What should be your initial action, given this situation?

Talk to a close friend or fellow EMS provider about your feelings

When you form an initial impression for a pediatric patient based on consciousness, breathing, and color, you are following:

The Pediatric Advanced Life Support (PALS) method

The release of myoglobin into the blood, caused by an electrical burn, can result in failure of:

The kidneys

Which sign or symptom best indicates that the patient with cold skin is experiencing generalized hypothermia?

Heart rate of 48 beats/min

The EMT should recognize heat cramps as the probable cause of a patient's problem when the patient states

"I have pain in my belly and legs."

What is the most important initial information you receive from dispatch?

Call location

A young female patient was walking on a 7-foot-high retaining wall when she lost her balance, stumbled, and fell from the wall. She impacted a concrete sidewalk surface with her left foot and left hand. Your assessment reveals ecchymosis, deformity, and edema to both the left foot and the left hand. Based on the concept of injury caused by indirect force, where else should the EMT look for injury

Left hip

A new EMT who just started working with your EMS system asks you to quickly describe a flail segment. Which statement best describes this injury

Two or more adjacent ribs are broken in two or more places."

A patient has suffered a small but jagged laceration to her left hand. When cleaning the wound, it is important that the EMT:

Use sterile gauze and wipe in a direction away from the site of injury

At a community meeting about preparations for a biological terror attack, an attendee asks you how anthrax acquired naturally differs from anthrax acquired via terrorism. What is your best response?

"Anthrax is the same regardless of whether it is intentionally inflicted or acquired naturally."

You are at a community crisis center with a male patient who is depressed. What is the most important question you must ask first in the care of this patient

"Do you feel suicidal or homicidal?"

A 51-year-old female patient tripped in the kitchen and fell against a deep fryer, splashing hot oil onto her right arm. Assessment reveals partial- and deep partial-thickness burns to her arm. You estimate that the burn covers 4 to 5 percent of her body. Which statement, made by the patient, provides the most important information for determining the overall severity of the burn from a medical standpoint?

"I have had diabetes for several years."

You are on the scene of a car that hit a tree head-on. The patient has a laceration to his forehead and a deformity to his left forearm. The car is a hybrid that runs on traditional fuel as well as hydrogen gas. The vehicle has moderate front-end damage, but the air bags did not deploy. The patient was wearing a seat belt. Which instruction is most appropriate in this situation to give to the arriving fire department personnel?

"I need to have the battery cables disconnected."

Which response shows that the EMT is correctly assessing motor function in the arms of a patient with a potential spine injury

"I need you to flex both arms across your chest.

You are caring for an emotionally distraught female who just suffered a miscarriage. During transport, which statement would be the most appropriate one for the EMT to make to the patient?

"I really do not know what to say, but let me know how I can help you.

You are having a difficult time controlling bleeding from a small skin avulsion on a patient's ankle. Which statement made by the patient best explains why control has been difficult to achieve?

"I take Coumadin for my irregular heartbeat."

A patient complaining of a headache states that he fell and struck the back of his head. Which statement by the patient should be of most concern to the EMT?

"I take a blood thinner for my heart.

Which statement made by the EMT is most appropriate regarding dealing with caregivers and children during a medical emergency?

"I try to include the caregiver in all that I do with his or her child so that the child and caregiver are more comfortable

An EMT asks you how he can decrease the chances of being involved in a collision when driving with lights and siren at night. Your advice would be:

"If blinded by the lights of an oncoming car, look to the right side of the road and not at the car

Another EMT tells you that he had a call the previous shift in which he thought a 5-year-old boy might be the victim of abuse, since he had bruises on both upper legs and chest. However, the EMT did not communicate this suspicion to anyone or transport the patient. What would your best response be

"If you do not take steps to report this suspicion, you can be punished in a court of law."

A 23-year-old roofer has fallen 20 feet from the roof of a house into shrubs below. Aside from superficial abrasions and complaint of soreness, you detect no obvious injuries. Consequently, the patient states that as long as his legs are not broken, he does not see the need to go to the hospital. As a knowledgeable EMT, your best response would be:

"In a fall from this height, you may have hurt some internal organs. You really need to be examined

You are transporting a young female who just had a miscarriage at home. During the transport the woman weeps continually. Which statement is most appropriate given the situation?

"Is there anything that I can do to help you right now?"

You have been called to an extended care facility for a 39-year-old male with shortness of breath. On scene, you discover that the patient attempted suicide several years ago by shooting himself in the head. The patient is now belligerent and swears at you as you assess him. Which question should be asked of the nursing staff first

"Is this his normal mental status?"

A 5-year-old male is drooling with stridorous respirations and has a fever of 104 °F. Which statement made by the patient would concern the EMT most?

"It hurts to swallow."

Which statement made by another EMT during a continuing education session, regarding treatment of the pediatric patient in the prehospital setting, is correct?

"It is critical that the EMT be able to determine the adequacy of the airway and respiratory status of the patient, as these are commonly the reason for acute deterioration.

A male patient was injured when a steam valve opened, causing hot steam to contact his face. Which statement made by the patient would be of most concern to the EMT and require immediate assessment or treatment

"It is sort of hard to breathe.

The EMT shows she understands the concept of the mammalian diving reflex by stating:

"It may provide a benefit to the submerged patient by diverting oxygen-rich blood to the brain and heart

You are teaching some Emergency Medical Responders (EMRs) to use the JumpSTART system of triage. Which comment made by an EMR requires you to intervene and provide corrective instruction?

"The JumpSTART system is very nice because it allows me to take 2 minutes to assess each patient."

Which statement demonstrates an accurate understanding of vital signs in relation to the geriatric patient

"The resting respiratory rate is typically higher in elderly patients."

A group of EMTs is preparing to restrain a large and violent patient. Which instruction from the team leader is most appropriate?

"You distract him by coming at him from the front and the rest of us will come at him from different directions, each taking a separate limb

The EMT should recognize which of these patients is experiencing generalized hypothermia?

A 21-year-old female who was trapped in cold water for 10 minutes before being rescued

Which patient has an isolated brain injury that should be considered the most serious?

A 21-year-old male wearing a deformed motorcycle helmet who does not respond to verbal or painful stimuli after crashing

When triaging multiple burn patients, the EMT would recognize which patient as having the most critical burn

A 22-year-old patient with a partial-thickness burn to the right hand

Which action shows that the EMT is properly caring for the umbilical cord after delivery?

A 6-inch segment of cord is still connected to the baby

After assessing these patients, which would the EMT identify as suffering from multisystem trauma?

A 61-year-old male who became dizzy and fell down five stairs, and who has a laceration to he head and open fracture of the left lower leg

A patient has a dialysis AV graft in his right forearm. Which statement regarding the EMT's assessment and interaction with this technology is correct?

A blood pressure cannot be taken in the right arm

You have been asked to present information on water safety and drowning to a community civic group. In regard to prevention, which point would you emphasize

A significant number of drownings are preventable due to the involvement of alcohol

Which patient requires the EMT to remove an impaled object in the field?

A young female with a broken pencil through her cheek and into the oropharynx; blood from he injury is flowing into her throat

When assessing a patient who sustained blunt trauma to the chest, which assessment finding is most indicative of an injury to the lung, impairing adequate ventilations?

Ability to speak only a few words. following by a gasp

A patient was assaulted and sustained multiple injuries of various types. Which injury does the EMT recognize as presenting the greatest risk for infection?

Abrasion to the right side of the face

When treating an injured geriatric patient, it is very important to:

Achieve an oxygen saturation of at least 95%

An alert and oriented patient has been exposed to the biological toxin ricin. He presents with a fever, cough, mild dyspnea, and generalized weakness. His airway is patent and his breathing is adequate. His skin is warm and dry, and his radial pulse is weak. Which intervention is most appropriate when caring for this patient

Administer oxygen to the patient

A patient has inhaled cyanide gas. He is alert and complaining of shortness of breath. His airway is patent, his breathing is adequate, and his radial pulse s strong. His skin is warm and dry, and his breath sounds are clear. Given these assessment findings, what should the EMT do first?

Administer oxygen via a nonrebreather mask

A young man fell down a flight of stairs and is now restless and confused. His airway is open, and he is breathing adequately at 18 times per minute. He has a radial pulse of 102 beats per minute, a blood pressure of 96/78 mmHg, and his skin is cool and diaphoretic. Your next action would be to:

Administer supplemental oxygen

The mother of a 4-month-old boy awoke to find him in his crib and not breathing, so she summoned EMS. On scene, you are providing resuscitative care while the parents watch your actions in disbelief. Which decision made by the EMT would be most appropriate regarding their presence in the room

Allow them to observe the resuscitation if they wish

You have been called for a 7-year-old girl who has vomited once and complains of abdominal pain. When you are performing the secondary assessment and obtaining a medical history, which action by you would be considered most appropriate?

Allowing the child to play with the stethoscope before listening to her lungs

You have applied a nonporous dressing to a stab wound on a patient's chest. The Emergency Medical Responder assisting you asks you why the dressing was taped on three sides and not all four. Your best response is that taping the dressing on three sides

Allows trapped air to escape on exhalation

Which action is most appropriate when using a helicopter on the scene of an emergency

Always approach the helicopter from the front, never from behind

When treating a pregnant female who was involved in a traumatic event during her third trimester, the EMT must remember:

An elevated heart rate seen in shock may not be discemable from the elevated heart rate from the patient's increased blood volume

While a 43-year-old man was cleaning a window, the glass broke and a sharp piece sliced his groin. Assessment shows him to have a large laceration to his left scrotum, with a testicle exposed and protruding through the sac. Blood is pouring from the laceration, and the patient rates the pain as an 8/1 O. What should the EMT do first?

Apply direct pressure to the injury to stop the hemorrhage

You arrive at a meat packing plant to find a 30-year-old male patient who was cut in the wrist with a sharp knife. The patient appears pale, and blood is spurting from the laceration. Your immediate action would be to:

Apply direct pressure to the laceration

A developmentally challenged male is biting himself and has a bleeding bite to his right hand. As you apply direct pressure to the hand, the patient continually tries to bite you, pulls his arm away, and screams. It seems only his mother can calm him down. At this time, you would

Ask the patient's mother to hold direct pressure to the bite

A high school football player was hit from behind and now complains of severe back pain and numbness to his right leg. Manual spine motion restriction is being maintained by an assistant coach. The trainer has already removed the face mask from the helmet. Once at the patient's side, what should you do next?

Assess the airway and breathing

The stinger of a bee is evident in the left thigh of a female patient who states that she is allergic to bees and has an EpiPen for such occasions. According to the patient, the sting occurred two hours ago and she just now noticed that the stinger remains in place. She did not take her epinephrine and appears asymptomatic for an allergic reaction. At this time, you would

Attempt removal of the stinger

You are transporting a child with a small laceration to his leg, sustained after cutting himself of a piece of broken glass. During transport, you observe a pulse oximeter reading of 87% on the frantic, crying, and actively moving child. The child is alert and oriented with a normal pulse and respiratory rate with pink skin that is warm to touch. You should consider:

Attempting to calm the child and obtain another Sp02 reading

Alzheimer's disease is diagnosed by:

Autopsy

Which statement about dressings and bandages is true?

Bandages should not be applied until bleeding is controlled

When do most women experience a decrease in the nausea and vomiting associated with hyperemesis gravidarum

Between 14 and 20 weeks

Regarding a terrorist attack involving the use of biological toxins, which statement is true?

Biological toxins cannot be passed from one person to another

Which patient triage tag indicates the lowest priority for care and transport on the scene of a multiple-casualty incident

Black

On scene, a paramedic directs you to help another EMS crew who is caring for a patient who was hit and trapped under a passenger van that ran into a crowd. As you approach the patient, which scene size-up clue seemingly indicates that the patient is suffering from traumatic asphyxia?

Bluish discoloration to the neck and face

An elderly patient has fallen in her home and activated the 911 system by pressing a medical alarm worn around her neck. On arrival, you can see the patient on the floor, but she cannot come to the door to open it and let you in. In this situation, your best course of action is to:

Break a window in a room away from where the patient is located

Which finding, when considered individually, is most consistent with a basilar skull fracture

Bruising behind the ear

You have arrived on the scene where a depressed patient is crying in an upstairs bedroom When assessing her which action should be done prior to the others

Check a radial pulse

You are securing a patient to the long backboard. Of the options listed, which has the elements of securing the patient in the correct order

Chest and pelvis, head, legs

You have been called for a male worker complaining of arm pain. At the scene, the 31-year-old patient states that his arm was crushed between two heavy boxes that shifted while being stacked by a forklift. He had minimal pain at that time, but now his right forearm is swollen and painful, especially with movement. The patient also states that the arm and fingers feel as if they are "asleep." You note decreased strength to the arm and a radial pulse that is weak when compared to the left arm. There is no deformity to the arm and the skin is intact, although hard on palpation. Vital signs are pulse 88 beats/min, respiration 18 breaths/min, and blood pressure 134/76 mmHg. Given these assessment findings, the EMT should be suspicious of

Compartment syndrome

An elderly patient fell down a flight of stairs and now complains of neck and back pain as well as weakness to both legs. The primary assessment reveals no life threats to the airway, breathing, or circulation. Manual cervical spine motion restriction is being maintained. What should the EMT do next?

Complete the secondary assessment, looking for additional injuries

A conscious and alert 32-year-old male was assaulted and struck in the jaw with a baseball bat. He complains of tremendous jaw pain and exhibits an unstable mandible. His airway is patent, breathing adequate, and radial pulse normal. In caring for this patient and his injury, you would:

Consider the need to implement spine motion restriction precautions

A patient who requires ongoing home medical oxygen complains of generalized weakness. Assessment indicates a patent airway and adequate respirations. His radial pulse is strong and regular, and his skin is warm and dry. Vital signs are pulse, 92 beats/min; respirations, 18 breaths/min; blood pressure, 156/88 mmHg; and SpO2, 95%. The patient is currently on 2 1pm of oxygen via an oxygen concentrator positioned bedside. What would be appropriate care in the management of this patient?

Continue oxygen administration at 2 lpm via nasal cannula during transport

You are on the scene of a shooting. Your assessment reveals a 23-year-old male who has been shot twice. The first wound is to the left lower quadrant of the abdomen and is actively bleeding. The second wound is to the left lateral chest and bubbles every time the patient exhales. What would your immediate and best action be

Cover the chest wound with a gloved hand

A man grinding metal took his safety goggles off and was struck in the face with a shower of tiny metal fragments. He now complains of pain and visual disturbance in his right eye. When assessing the eye, you note that the globe is red, with several pieces of metal embedded in the tissue. Proper care of this patient would include

Covering both eyes with a bandage

A patient complains of pain and discomfort to the left eye after being hit in the face with a cloud of dust while working at a manufacturing complex. As you specifically assess the eye, you note some redness to the globe but do not see any obvious foreign object. The most appropriate care you can provide to this patient would be:

Covering both eyes with a bandage

The process of effectively using all resources available to reduce errors, increase safety, and improve performance is called

Crew Resource Management (CRM)

A young adult female was struck by a vehicle while she was riding her bicycle. She was not wearing a helmet. She is now unresponsive and has a deformity with a deep depression to the temporal area of the head. You do not see any bleeding, and the patient's scalp is intact. What is the greatest threat associated with this injury?

Damage to the brain

A patient fell and sustained an open fracture to the left humerus. However, assessment reveals the bone to have pulled back into the arm. Bleeding from the site is controlled. How will splinting the left arm benefit this patient?

Decrease the opportunity for further injury to nerves and blood vessels

You have been called to a New Year's Eve party for a male patient who was stabbed in the anterior chest. According to partygoers, the patient went outside with another male and was stabbed. Fifteen minutes later, he was found in the snow, and 911 was called. When assessing this patient, which finding would you consider to be the most serious and warranting immediate attention?

Decreased breath sounds to the left lung

You have been dispatched to transfer a patient with schizophrenia from a hospital emergency department to a behavioral health unit at another hospital. While en route, your partner states that he cannot remember what schizophrenia is. You should tell him that schizophrenia is a psychiatric disorder commonly characterized by

Delusions and hallucinations

A young female wears her lap belt too low, over her upper thighs, because the belt is uncomfortable when worn properly. If she is involved in a head-on "up and over" type of collision, to which injury is she most prone, given the position of her lap belt?

Dislocated hips

It is common for victims of human trafficking to

Display feelings that the trafficker is a loving partner

At the scene of a multiple-casualty drill, you have been assigned to the role of transport unit leader. In this role, one of your responsibilities is to

Distribute patients to each medical facility

You have been called for a 2-year-old girl who has been sick for two days. In preparing to assess the patient, it is important that you recognize that patients in this age group generally:

Do not like having clothing removed

Ageism would occur when the EMT:

Does not attempt to assess combative elderly patients with dementia because they "always act that way

Which behavior would the EMT recognize as uncharacteristic of a conscious and stable 2-year-old boy who fell and hurt his hand?

Does not cry or protest when taken from his mother

An 83-year-old female fell down the basement stairs. Once at her side, she tells you that her left hip hurts and rates it as 10/10. Since the patient is downstairs, you recognize the need for assistance in extricating her from the residence. As you start the primary assessment, you note a dog at the top of the stairs starts growling at you. At this time, what is your primary concern?

Dog

You have arrived on the scene of an explosion in a warehouse used to store fertilizers. A paramedic, who has already triaged the nine patients involved, directs you to a young man sitting under a tree. The man states that he is having excruciating ear pain. The initial assessment and rapid trauma assessment reveal no obvious injuries. Given this presentation, the EMT would recognize the injury was likely sustained:

During the primary blast phase

Croup is recognized as an illness that involves:

Edema beneath the glottis

A construction foreman was stabbed with a screwdriver in the right anterior chest by an angry employee. Prior to your arrival, he removed the screwdriver. He is alert and oriented and complains of pain to the injury site. Your primary assessment reveals an open airway, adequate breathing, and strong radial pulse. Based on these findings, which intervention should be performed next?

Evaluate the patient's SpO2 level

A female patient who is 35 weeks pregnant tripped and fell down 12 stairs. She now complains of back pain, numbness to her legs, and nausea. She is secured to a long spine board with spinal motion restriction precautions placed and is being non-emergently transported to the hospital for evaluation. En route, the patient exclaims that she is having contractions and thinks that her water may have broken. Which intervention should you perform immediately?

Examine her genitalia for crowning or bleeding

How can you help prevent the entrance of carbon monoxide from the ambulance exhaust into the unit

Extend the exhaust and leave the rear windows closed

As part of your service to the community, you are certified as a car seat specialist and provide monthly classes on the safe transport of infants and children. Over the course of one day, several cars have pulled into your EMS station and asked you to look at their car seats. Which car seat position would indicate that the parent is safely transporting his or her infant (younger than 1 year old)?

Facing backward in the back seat in a reclined position (45 degrees)

The need for a comprehensive understanding of normal growth and development, which enables the EMS provider to better anticipate the physiologic and emotional needs of the child who is affected by illness or injury, is a principle of

Family-centered care

hen assessing a patient at an extended care facility, you note the presence of an ostomy bag. Based on this finding, you realize that the patient has impaired function of

Fecal elimination

A 21-year-old male has a gunshot wound to the head. As you assess him, which finding would you recognize as a nonpurposeful response to pain?

Flexing his arms across his chest when you pinch his shoulder

A male patient has just been burned with hot water and has blisters on his left arm. The EMT would recognize that:

Fluid has collected between the layers of skin

A student in a science lab has had an unknown chemical splashed into her eye. After assessing the airway, breathing, and circulation, the EMT would immediately

Flush the eye with copious amounts of tap water

A patient has an injury to his arm caused by an unknown vesicant. After assessing the patient's airway, breathing, and circulation, you should

Flush the injured area with water

A 77-year-old male, who is blind and has a guide dog, called 911 for chest discomfort. As you are preparing to transport the patient, he indicates that he wants to take his dog. How should you respond?

Follow local protocol

Which type of radiation will travel the farthest and penetrate the human body with the greatest ease

Gamma

Medical direction has asked you to remove a tick from behind the left ear of a 5-year-old girl. You would:

Grab the tick close to its head and pull directly outward

You are assessing a female patient who informs you that she has been pregnant four limes before but delivered only once. What would be a correct way to document that information on the prehospital care report?

Gravida IV; para I

A helicopter has been requested at the scene of a motor vehicle collision at night where two people have been critically injured. As safety officer overseeing the incident, which action would cause you to immediately intervene for the safety of all rescuers, bystanders, and patients?

Guiding the helicopter to the landing site using a bright spotlight

During the summer months, you work as an EMT at a first aid station that helps people who hike trails in the high mountains of the western United States. One afternoon, you are presented with a 13-year-old male patient who presents as alert and oriented, but complains of chest pain, dyspnea, and a headache. He is tachypneic and tachycardic and has slight inspiratory crackles. Given this presentation, what would be the most likely differential diagnosis

High-altitude pulmonary edema

The primary center for controlling the temperature in the human body is located in the

Hypothalamus

A 45-year-old female was at a family reunion and fell off a trampoline. She hit a picnic table with the left side of her lower rib cage. Now, she is very anxious and confused. Her airway is open and her breathing adequate. Her radial pulse is rapid and weak, and her skin Is cool and diaphoretlc. Vital signs are pulse, 132 beats/min; respirations, 20 breaths/min; blood pressure, 102/78 mmHg; and SpO2, 97% on ambient air. The patient has ecchymosis to the left lower rib area as well as tenderness and instability to the rib cage. She also has a superficial abrasion to her forehead. Given these findings, you would suspect and treat her for which condition?

Hypovolemic shock

You are managing a patient with a large flail segment to the right lateral chest. With this condition, you should recognize that the immediate threat to life is:

Hypoxia

Which complication from a pulmonary embolism is of greatest concern to the EMT

Hypoxia from poor lung perfusion

You are assessing a 21/,-year-old female who was involved in a minor car accident. She is currently alert and oriented. While you are assessing her for possible injuries, which assessment procedure would be considered appropriate?

If possible, allow the child to hold a favorite toy during the assessment

A paramedic has asked you to sterilize a laryngoscope blade that was placed in the mouth of a patient during an endotracheal intubation procedure. To sterilize this piece of equipment, you should:

Immerse and leave the blade in an EPA-registered and approved chemical sterilant

You are called to treat a homeless person who complains of an inability to walk due to sores and numbness of his feet. The EMR tells you that the patient is well oriented with good pulses and respiration but that his feet are edematous with open sores and broken-down skin. This finding is the typical result of which condition

Immersion foot

Which of these statements about immersion hypothermia is false?

Immersion hypothermia typically does not occur until the water temperature is less than 35° F

A newborn's first Apgar score was 6. His second Apgar score is 9. What does this information mean to the EMT?

Improvement

You have been called to the side of a confused geriatric female who complains of abdominal pain. Before palpating the patient's abdomen, you would first:

Inform the patient that you will be pressing on her abdomen

A 74-year-old female complains of abdominal pain after falling down five stairs off her front porch and striking her abdomen and left hand on a mailbox at the bottom. The patient also reports pain in her left wrist and right ankle, both of which show obvious deformity. She is alert and oriented, and her abdomen is free of bruising but tender in the left upper and lower quadrants. Her vital signs are pulse, 132 beats/min; respirations, 22 breaths/min; blood pressure, 106/86 mm Hg; and initial SpO2, 93% on room air. Oxygen has been applied, and the patient has full spine motion restriction precautions in place. Advanced life support has been requested and is 12 minutes away from your current location. What is your next action?

Initiate immediate transport and intercept with the ALS unit

A 22-year-old male attempted to hang himself in a garage using chains suspended from the roof trusses. He was quickly found by family members and taken down to the ground. Currently he is complaining of neck pain as well as mild difficulty breathing. Assessment shows his airway to be patent, breathing adequate, and skin warm and dry. His neck is red, but free of any subcutaneous air or swelling. Your partner has identified an abnormal bony protrusion to a couple of his cervical vertebrae. What should you do first when caring for this patient?

Initiate manual spine motion restriction

The father of a developmentally challenged 45-year-old male summoned EMS because his son was "not acting right." The patient is awake, but is nonverbal, which is not typical according the father. Your assessment reveals no abnormalities, and vital signs are normal. When calling the chief complaint into the hospital, you would state that the patient

Is not acting normally per his father."

With the current trend of providing medical technologies in the home, the EMT:

Is often the first resource called if a failure occurs

Which statement about removing a helmet in the prehospital setting is true?

It is acceptable to leave the helmet on a patient if the patient has no airway or breathing problems

Which statement concerning the biological agents used in terrorist attacks is true

It may take several days to weeks to realize or determine that an attack involving biological agents occurred

A patient had an unknown chemical splashed into her left eye at a factory. You have been irrigating the eye on scene and are now getting ready to transfer the patient to the stretcher. Knowing that you will need to continue irrigation throughout transport, how will you position this patient on the stretcher?

Left lateral recumbent

You have been called to an assisted living facility to transport a patient who has been vomiting continuously for 6 hours. She is responsive to verbal stimuli. The patient has both arthritis and severe kyphosis. How should you transport this patient on the stretcher

Left lateral recumbent position

Which statement is true regarding linear skull fractures?

Linear skull fractures cannot typically be identified with palpation

An Emergency Medical Responder reports that a male patient, who was injured while playing football, has bruising to the lumbar region of the body. Based on this statement, the EMT should expect to find bruising in which area

Lower back

A female lifeguard fell 10 feet from her chair and now has deformity, swelling, and ecchymosis to her right lower leg. The leg also is rotated medially. Another lifeguard has taken and is maintaining manual cervical spine motion restriction. As your partner conducts the primary assessment and finds no life threats or disturbances in her mental status, you should perform which action first

Manually stabilize the lower right leg

Which statement accurately describes multisystem trauma?

Multisystem trauma has a noted higher incidence of morbidity and mortality

A patient whom you transported several days ago stops by your station to thank you. He reports that he was diagnosed with a strain to the lower right leg As such, you recognize which structure as being affected by that injury?

Muscle

Which organization(s) published the recommended guidelines for securing and tracking vehicle access in the ambulance industry

NAEMT

You have been called to a home by neighbors because they are concerned for the welfare of the children inside. On scene, you find two children left by themselves. They are playing in a filthy room with no light or heat. They are dirty and have insect bites all over their bodies. One is in a soiled diaper and has hard stool caked to the buttocks. The EMT would best recognize this situation as one of

Neglect

A young female pedestrian was hit by a car and thrown 15 feet, striking her head against a metal guardrail. You start your secondary assessment and note a deep depression to the right parietal area of her head, with intact skin overlying the area. You should:

Note the injury and continue assessing the patient

You have been called to a local emergency department to transfer a male patient to a specialty facility for further care. The man was involved in a deep-sea dive emergency and has been diagnosed with barotrauma. Which finding should be expected with this condition?

Pain and deafness to the ears

While at a public pool obtaining a refusal of care from a patient who was stung by a bee, you hear screaming. Patrons are waving you over to a 13-year-old boy who hit the diving board with his head while diving. He is now in the water and is motionless. The lifeguard is in the water and informs you that the patient's eyes are open and he is breathing with a pulse, but not talking. What should you do next?

Pass a cervical collar and long board into the water for providing spine motion restriction precautions to the patient prior to his extrication from the water

A 42-year-old man has called 911 because of severe back pain. He informs you that his back pain is related to a recent fracture of his coccyx. Based on this information, where should the EMT expect the patient to be complaining of pain

Pelvic area

When assessing a 65-year-old male patient, you note a tube-like device inserted into the lower portion of his trachea. You recognize that the device has most likely been placed to:

Permit the exchange of 02 and CO2

When working the scene of a water rescue, anytime that the EMT is within 10 feet of the water's edge, it is essential that she don which equipment for personal safety

Personal flotation devices

You have been called to an extended care facility for a 48-year-old male patient with a history of schizophrenia. On scene, you find the patient to be highly agitated and clutching his left hand while complaining that there are bugs in it. After performing the primary assessment and finding no life threats, you should:

Physically assess his left hand for any abnormalities

A female patient suffered a straddle injury (genitalia trauma) while trying to balance on top of a chain-link fence, after she fell with a leg on each side of the fence. At this time, you would:

Place a moist compress, such as a moist sanitary pad, over the injury to control the bleeding

A natural gas explosion has destroyed an apartment building. At least five occupants of the building were killed, and another 12 are injured. You are in charge of triage. You are presented with a young woman who is breathing at a rate of 40 breaths per minute after fire department personnel carried her out from the collapse site. Using the Simple Triage and Rapid Transport (START) system for triaging, you should immediately:

Place a red tag on this patient and move on to the next patient

You are en route to the hospital with a patient who is unresponsive and has hot, dry skin. His friends state that the patient had been drinking and passed out in the hot sun for several hours before 911 was called. An Emergency Medical Responder is ventilating the patient with a bag-valve mask attached to supplemental oxygen. Which of these actions would be most beneficial to the patient next?

Place cold packs to the patient's groin and armpits

The EMT is correctly assessing the skin temperature of a patient who has been exposed to cold temperatures for an extended period of time when he

Places the back of his hand on the patient's abdomen

A 7-year-old with a fever of 101.3° F and shortness of breath has a patent airway, but tachypneic respirations. His heart rate is 132 beats/min, respirations are 28 breaths/min, and blood pressure is 94/60 mmHg, with an SpO2 of 96% on room air. Crackles and wheezes are noted in the base of the right lung only. Given this presentation, you would suspect and treat the patient for

Pneumonia

An industrial worker sustained partial- and full-thickness burns to his right hand after accidentally touching an electric furnace used to incinerate medical waste. Assessment reveals no threats to the airway, breathing, or circulation. Which action would the EMT perform next?

Pour cool saline water on the hand

Firefighters have just pulled a young woman from a house fire and have laid her on the lawn. The patient is conscious, oriented, and badly burned on the chest, abdomen, and legs. Her clothes are still smoldering. Which action should the EMT take immediately?

Pour water on the smoldering clothes

After successfully delivering a baby, the EMT notes that the protruding umbilical cord is lengthening and a small gush of blood came out of the vagina. The appropriate action would be to:

Prepare for delivery of the placenta

A 36-year-old male patient was involved in an altercation and now has a 2-inch laceration from a knife to the right side of his neck. Bleeding was controlled prior to your arrival by Emergency Medical Responders. Your paramedic partner has asked you to apply an occlusive dressing to the wound. You recognize that the primary benefit of the action is to:

Prevent air entry into the circulatory system

Approximately 15 vehicles have been involved in a "chain-reaction" collision on a freeway one foggy morning. Your unit is the first ambulance on the scene, and you begin going from vehicle to vehicle to determine the type and severity of each injury experienced by the occupants. This is an example of which kind of triage?

Primary

You arrive on the scene of a motorcycle accident and find an unresponsive male patient lying in the middle of the road with blood coming from his ears and nose. He was not wearing a helmet and has snoring respirations. Which sequence of events would be most appropriate in the care of this patient?

Primary assessment, airway and ventilatory management, oxygen administration, spine motion restriction precautions, transport within 1 O minutes of arriving on scene

A 78-year-old male pedestrian was struck by a car at a convenience store. The patient is conscious but confused, and has sustained multisystem trauma. What would represent the best sequence of care for this patient?

Primary assessment, oxygen, secondary assessment, spine motion restriction precautions and all other care en route to the hospital

What determines the order in which patients are relocated from triage to treatment

Priority

During a motor vehicle collision, a confused unrestrained male struck the steering wheel with his abdomen. Currently, he is confused and complains of generalized abdominal pain, which he rates as 6/10. Assessment indicates red marks across his lower abdomen caused by the steering wheel. His pulse is 140 beats/min, respirations are 24 breaths/min, blood pressure is 108/88 mmHg, and SpO2 is 86%. The patient looks pale and has moist skin. What is most appropriate when providing oxygen to this patient

Provide 15 lpm through a nonrebreather mask

You have just arrived on the scene of a motor vehicle collision where a car rolled several times at a high rate of speed. The car is on its wheels, with the restrained driver still in his seat. After ensuring that the scene is safe and gaining access to the patient, what should you do immediately if the patient is found unresponsive

Provide manual spine motion restriction

A patient has been pulled from a house fire. Assessment reveals him to be lethargic with stridorous respirations, minimal chest wall motion, and a rapid but strong radial pulse. Firefighters are dousing his body with water to cool him and ensure he is extinguished. Which action would the EMT perform immediately once the fire on the patient is adequately extinguished with water?

Provide positive pressure ventilation with a bag-valve mask

In which of these settings will sweating be less effective as a means to cool the body?

Relative humidity of 98%

What is a general guideline to use when deciding whether to remove a patient's contact lenses?

Remove contact lenses if the eye has been injured by a chemical burn

You are responding to a possible pediatric overdose. It is raining and suddenly the ambulance starts to hydroplane. Your immediate action should be to

Remove pressure from the accelerator pedal

A 24-year-old female who was hiking in shorts informs you that she has been bitten by an insect on her leg. She states no other complaints. Assessment reveals a tick embedded in her leg. As an EMT, you should provide care to this patient by:

Removing the tick with tweezers

You have been dispatched to a residence for a female patient who is dizzy and "passing out." An Emergency Medical Responder meets you at the door and reports that the patient is lying in bed and is nine months pregnant. Her pulse rate is 112 beats per minute and her blood pressure is 84/50 mmHg. Respirations are 24 breaths per minute and she has a room air pulse oximetry reading of 97%. What should you do as soon as you reach the patient's side?

Roll her onto her left side

When assessing a male patient who was punched in the face by another man, you detect blood in the white portion of his eye. Which optical structure has been injured?

Sclera

You are transporting a critically ill 4-year-old patient. Regarding the panicked and upset mother, you would:

Secure the mother in the "captain's chair" of the ambulance, behind the patient, during transport to the hospital

A patient is being extricated from a car using a short vest-type device. After the patient has been extricated, the EMT should

Secure the patient with the vest-type device to a long backboard

An adult female is found unresponsive after being shot in the chest. Which statement made by your partner is of most concern and indicative of patient deterioration?

She is getting more difficult to ventilate."

Which statement would most likely be accurate regarding a 42-year-old female with traumatic paraplegia?

She requires a wheelchair

Which of these findings would present earliest in a patient with hypothermia?

Shivering

You are assessing a conscious but confused hunter who became lost in the woods on a cold day. Your assessment shows him to have an open airway, adequate breathing, and a weak radial pulse. His skin is cold to the touch and he is shivering. The EMT should recognize that the

Shivering is a protective means by which the body is attempting to warm itself

What is the primary concern with cyberterrorism

Shutting down the infrastructure

The EMT would recognize that a hypothermic patient's condition is deteriorating when he observes

Slurred speech

What is an emergency care measure for a patient with a possible bone fracture that can be an effective means to reduce pain

Splinting the fracture

During the secondary assessment, the EMT notes instability and a flail segment to a patient's lower left lateral chest. The EMT would immediately

Stabilize the segment with his hand

You are called by the owner of a restaurant for a female patron who spilled hot tea onto her right thigh. Your assessment reveals the burn area to be soft, red, and painful. The skin is intact with no blister formation noted, although some edema is present. Based on these assessment findings, you would recognize this injury as belonging to which category of burn?

Superficial

A patient has sustained a large abdominal evisceration, such that a large portion of intestines are now exposed. After properly managing this injury, you are preparing the patient for transport. You ascertain that there is no evidence of spinal injury. How should you place the patient on the stretcher for transport?

Supine position, with knees and hips flexed

When faced with danger, which component of the nervous system is immediately activated for basic survival

Sympathetic

A patient who received peritoneal dialysis has called 911 for generalized weakness. Upon arrival of EMS, which finding should be of greatest concern in this patient

Temperature of 101 ° F

Hospice care indicates that a patient is: A. Terminally ill and foregoing any additional care until death B. Terminally ill and but receiving care to reverse the disease process so as to avoid death C. Terminally ill and receiving measures to increase comfort until death D. Terminally ill but without a Do Not Resuscitate order

Terminally ill and receiving measures to increase comfort until death

An ambulance, while responding to a call using lights and sirens, is involved in a collision at an intersection. Witnesses state that the ambulance slowly went through the red light without first stopping, resulting in a collision with a car. No one is injured, and the driver of the car is insured. Based on the witnesses' accounts of the collision, which outcome is likely to occur

The EMT driving the ambulance may be liable for the collision

You are called for a 2-year-old boy who has a nosebleed. While you are en route to the call, the dispatcher informs you that the patient has hemophilia. As an EMT, you should recognize that:

The bleeding may be significant

A female patient with osteoporosis stepped sideways on her foot and has suffered an open tibial fracture to the distal portion of her right leg. Which intervention indicates that appropriate care is being given for this injury

The bone end and soft tissue wounds are covered with a sterile dressing

Which statement is most accurate regarding the processes needed for a patient to clench the hand into a fist?

The central nervous system must send a message through the spinal cord and then to the peripheral nerves

When treating a pediatric trauma patient, what is important for the EMT to remember?

The flexibility of the chest wall in a child can allow for internal injury without obvious external signs

You have been called for a patient with severe back pain. When you are obtaining a medical history, the patient tells you that he has a history of damaged disks in his vertebrae. As a knowledgeable EMT, you should recognize that:

The fluid-filled structures between the vertebrae have been injured

You are reassessing a patient whom you are treating for heat exhaustion. Which of these findings indicates that the patient is improving?

The mental status is now alert and oriented

You are assessing a 9-month-old boy who has had a fever for the past two days. What would be a likely finding if the fever has caused dehydration?

The mother informs you that the baby is wetting very few diapers daily

In terms of high-visibility ANSI vests, which statement is true?

They make EMS personnel more visible to passing motorists

You are transporting a restrained 23-year-old female patient with a past history of sexual abuse and multiple behavioral problems. She is very upset with being restrained as well as with being transported to the hospital. She informs you that she will take you to court for your actions. Consequently, you are concerned about false accusations. What is your best protection should she follow through with legal action

Thoroughly document everything that happens on the call

You are at the scene where a male patient threatened to take pills to end his life. In talking with the patient, he informs you that he just got a new job that requires more of his time and his wife has been giving him a hard time about it. Tonight, he had enough and just made the comment because he was mad at her. He says that although he is still upset, he does not plan to do anything "drastic." Before taking further action, which fact about the patient should raise the EMT's suspicion that he is indeed at a higher risk for suicide

Unsuccessful attempt three years ago

What is the earliest point at which you can begin considering if there are obstacles to patient extrication

Upon receiving the call from dispatch

When providing needed oxygen to a patient with a core temperature of 95°F and adequate breathing, which strategy is the best option?

Warmed and humidified oxygen delivered through a nonrebreather mask

A car has rolled several times and is on its roof. Closer inspection reveals the unrestrained driver lying in the back with the back portion of the roof collapsed. Which instruction should you give first

We need to take the weight off of the roof posts."

According to the recommended guidelines for securing and tracking vehicle access in the ambulance industry, how often should security briefings be held

Whenever there is a change in the Homeland Security threat level

You are teaching a group of EMTs about multiple-casualty incidents and the Incident Command System (ICS). Which statement about ICS is true

Plain English is used for all communications in the United States

Which one of these patients is the EMT justified in restraining?

A patient who threatens to kill himself and his wife and is trying to leave the scene

A male patient who works in a steel fabricating plant has suffered a flash burn to his right arm and hand. Which assessment finding would assist the EMT in determining that the burn is superficial and not partial thickness?

Absence of blisters

A patient was stabbed in the right anterior chest and is in obvious respiratory distress. As you perform the secondary assessment, which signs and/or symptoms would suggest that she is developing a tension pneumothorax

Absent breath sounds on the right, distended neck veins, tracheal deviation to the left

You are by the side of an 88-year-old male who fell. He has a hematoma to his forehead as well as deformity to his left forearm. His daughter informs you that her father has severe dementia and she cares for him at home. As you assess him, he continually screams out, "Mary, help me. I need help!" At this time, which question is most appropriate to ask the daughter

"Is this how he normally is?"

You are hosting a discussion on injury prevention when a participant asks you if you believe that helmets for motorcyclists truly make a difference. You would respond by saying

"Motorcycle helmets decrease the risk of death when the rider is involved in a collision."

You are assessing a young male patient who was assaulted with a baseball bat and struck once on the left upper quadrant of the abdomen. You would recognize that this patient has Kehr's sign when he makes which statement

"My left shoulder is killing me!"

Which statement made by an EMT shows a proper understanding of what is required when restraining a patient?

"One person should be assigned the task of explaining everything to the patient before, during, and after he has been restrained

Which statement indicates an understanding of the EMT's pre hospital role in caring for the patient in shock secondary to significant soft tissue trauma?

"Since trauma is best treated in the hospital, the EMT should provide care to maintain perfusion to the vital organs and transport."

An elderly female patient fell and now complains of right hip and right knee pain. When assessing her, which finding would you find most concerning

Absent pedal pulse

You have been called for a 35-year-old female who is experiencing vaginal spotting. The patient states that she has noted the discharge of blood from her vagina and is concerned because she is 36 weeks pregnant. She has also experienced some abdominal pain, which she describes as "sharp." Your assessment reveals bleeding as noted and tenderness on palpation just right to the midline of her abdomen. Her pulse is 122 beats/min, respirations are 22 breaths/min, blood pressure is 118/82 mmHg, SpO2 is 96%, and oral temperature is 97.3° F. Based on this presentation, you would:

Administer high-concentration oxygen and immediately transport the patient

An Emergency Medical Responder asks you to explain a pneumothorax. You would explain that a pneumothorax occurs when

Air accumulates around the lung, causing a portion of ii to collapse

A 66-year-old male has summoned your assistance because he is short of breath and his home pulse oximeter is reading 89%, despite 3 lpm of home medical oxygen. Assessment reveals an intact airway and adequate breathing. The patient's radial pulse is normal, and his skin is hot and dry. Vital signs are pulse, 84 beats/min; respirations, 18 breaths/min; and blood pressure, 166/86 mmHg. Breath sounds are clear bilaterally. The patient's pulse oximeter reading is fluctuating between 88% and 90%. In this situation, you would first:

Check the patient's SpO2 with your pulse oximeter

As an EMT working at a mountain resort, you are presented with a 50-year-old female who states that she does not feel well. From your assessment, which of these findings would seemingly indicate that the patient is being adversely affected by the high altitude

Complaint of dyspnea on exertion

To gain access to a patient entrapped in a car, the rescuers must use the "jaws of life" to open the door. The EMT should recognize this situation as:

Complex access

A young intoxicated male patient cannot move his left arm and leg after diving into the shallow end of a pool and hitting the bottom head first. The EMT would recognize which mechanism as most likely responsible for this injury

Compression

A male soccer player was struck in the head with a soccer ball. Players state that he was dazed for several seconds following the impact and then asked the same questions over and over. Presently, he is conscious and oriented to person, but confused about place and time. He also has a reddened area to the side of his head and face. As you proceed with your assessment, his memory continues to improve. Based on these findings, the EMT should suspect which type of injury?

Concussion

A patient who was involved in an altercation was struck on the side of the head with a baseball bat, in addition to suffering several blows to the arms and legs. When assessing this patient, which sign or symptom best indicates the patient has suffered a brain injury?

Confusion and combativeness

The normal response of the pupils while examined with a penlight is described as

Consensual

After successfully rewarming a patient's frozen foot and frozen toes, the EMT should:

Cover the foot and toes with dry, sterile dressings

You have arrived at the emergency department with a young child whom you believe is the victim of child abuse. To which person would it be best to report your suspicion

Emergency physician

What is the first golden principle for prehospital multisystem trauma care

Ensure safety of the rescue personnel and the patient

BRUE evaluations are typically performed on infants who:

Had a medical episode that concerned their caregiver

When providing prehospital care to a female patient with a soft-tissue injury, which statement is true

Hands should be washed even if gloves were worn during care

What is the mnemonic for the standard process used during on-scene management of a disaster or MCI?

ICS

Which clinical finding is most suggestive of a patient who is decompensating?

Narrowing pulse pressure

A 39-year-old male has a single stab wound to the anterior chest. When notifying the emergency department, you inform them that the patient has a/an

Open chest injury

A 14-year-old male fell 10 feet from a retaining wall and hit his head on a metal post. He is responsive to verbal stimuli with incomprehensible speech. The secondary assessment indicates blood and fluid coming from inside the patient's left ear. Given these assessment findings, the EMT should

Place a piece of sterile gauze over the ear to catch the fluid

You are completing the delivery of a near term baby boy when you notice that the fluid expelled with the baby is greenish-brown. At this point you should

Quickly suction the nose and mouth only if the newborn's condition is depressed

You are called to a residence for a 27-year-old woman who fell and hurt her arm. As you begin your assessment, you begin to realize that there may be abuse involved. Suddenly the husband decides that he will transport the woman and you are no longer needed. You are unable to change the husband's mind and he is becoming more agitated. What should you do next

Retreat to the ambulance and notify law enforcement

You are responding to a low-income apartment complex for a 16-year-old female who is five weeks pregnant with vaginal bleeding. When mentally reviewing possible causes of hemorrhage for this patient, which condition are you most likely to encounter given the information provided thus far

Spontaneous abortion

The EMT should recognize a possible spinal column injury with no spinal cord involvement when he discovers which assessment finding

Tenderness to the thoracic spine with intact motor and sensory abilities to each extremity

Assessment findings for a 33-year-old female who was shot in the right side of the chest indicate that she has a pneumothorax. As you care for this patient, your primary concern is monitoring the injury for

Tension pneumothorax

Ensuring that all children and adolescents, no matter where they live, attend school, or travel, have access to and receive appropriate care in a health emergency is the purpose of:

The federal Emergency Medical Services for Children (EMSC) program

Which definition correctly identifies a stoma?

The opening in the neck for a tracheostomy tube

You are responding to a patient with a tracheostomy tube who is weak and has a fever. In this situation, you can assume:

The tracheostomy tube is below the larynx

Which statement is true regarding post-traumatic stress disorder

There is no known or universal cure for post-traumatic stress disorder

Regarding the risk of suicide for combat veterans, which statement is true

There is often no preventing suicide once this course of action is decided

An intoxicated driver drove his car into the side of a slow-moving train, hitting a tanker car. You recognize the need to call CHEMTREC to get information on what is contained within the tanker. Which piece of information is it critical to provide to this agency so it can respond with helpful information

UN identification number

How long can a person who has radiation sickness expect to have symptoms

Up to 7 or 8 weeks

The EMT shows that he understands the difference between a pneumothorax and a tension pneumothorax when he makes which statement?

"A tension pneumothorax causes cardiac output to decrease; a simple pneumothorax does not affect cardiac output.

The EMT identifies the primary benefit of triage when she states

"Triage is used to determine the order in which patients will receive medical care and transport

A middle-aged male's arm became caught in a grinding machine, and the arm was eventually extricated from the machine by coworkers. As you approach the patient, you note him to be holding a bloody towel over a severely deformed right arm. The patient is alert and anxious, and he has a patent airway. His breathing is adequate and his left radial pulse is fast and strong. You note his skin to be cool and diaphoretic. Given this information, which care would be provided first for this patient?

Administer supplemental oxygen

Public warnings are best carried out by

Any means you have available

A patient has accidentally removed his peripherally inserted central venous catheter (PICC). What would appropriate care for this patient include

Apply a bulky dressing to the insertion site to control any bleeding

A construction worker was accidentally shot with a nail gun. The nail penetrated the skull approximately ½ inch through the temporal region. Before your arrival, coworkers pulled the nail free. Based on the mechanism of injury, which sign provides the strongest evidence that the meningeal layers have been breached

Cerebrospinal fluid oozing from the puncture wound

What is one of the best ways lo help reduce the public's stress associated with an evacuation

Communicate-assign several rescue workers to gather information and disseminate it to local radio and television stations

A small truck has gone over an embankment. The driver has a head injury and is confused, but is able to tell you that he is hauling N-acetyl-para-aminophenol. Which resource would provide you with the best initial information about this chemical?

Emergency Response Guidebook

Which statement regarding pediatric injury prevention is most accurate?

Injury prevention and accident prevention are not the same

67. When should the EMT use the Apgar scoring system on a newborn?

One and five minutes after the baby has been delivered

A leading cause of death in children younger than 14 years could be prevented by:

Safer driving of vehicles

You arrive at the residence of a physician who informs you that his pregnant wife requires immediate transport to the hospital because she is bleeding and has a history of abruptio placentae. As a knowledgeable EMT, you should recognize that the greatest threat to the baby is

Hypoxia

Which statement is most accurate concerning spine motion restriction of an ambulatory patient?

If the patient is reliable, he can self-restrict while the cervical collar is applied

Which statement made by the EMT indicates an understanding of the prehospital calculation of body surface area (BSA) burned

"Determining the BSA is an estimation and should never delay transport of the burn patient to the hospital

On scene at a college football game, a wide receiver is unresponsive after colliding with another player and forcefully taking a helmet to the chest. The athletic trainer reports that the patient is pulseless and apneic. Given the mechanism of injury and patient's presentation, which instruction would be appropriate?

"Let's apply the AED and follow all prompts."

A 5-year-old boy who is drooling has suspected epiglottitis. His pulse rate is 144 beats/min, respirations are 46 breaths/min, and blood pressure is 110/52 mmHg. The patient is noncompliant with the pulse oximeter, but his skin is normal colored. Which statement made by the EMT indicates proper care of this patient?

"Let's give him some supplemental oxygen."

A patient has been pulled from a pool in which he was submerged for several minutes. He is pulseless and apneic. An Emergency Medical Responder asks you if you want to press on the man's abdomen to remove the water from his stomach. What is the most appropriate reply?

"Let's hold off on that until we see how well we can ventilate the patient."

You are on scene of a serious motor vehicle collision involving multiple cars on a two-lane county road. As the EMT in charge, what is the most appropriate instruction to give other rescuers on scene

"Let's stop all traffic and reroute it to alternative roadways."

You have just arrived by the side of a 23-year-old female who is 37 weeks pregnant and apneic and pulseless after being shot in the abdomen. Regarding care of this woman, which statement would you provide to other rescuers on scene?

"Waiting for ALS backup is not beneficial; we need to provide CPR and get her to the hospital now

You are participating in a training exercise for an attack with a weapon of mass destruction involving the release of a nerve agent. In the triage area, you are presented with a patient who responds to painful stimuli and has an open airway. He is breathing 8 times per minute and has a weak radial pulse. He is wheezing and has pinpoint pupils. In his pocket, you see a metered-dose inhaler containing albuterol. When caring for this patient, you should first:

Administer positive pressure ventilation

While responding to an emergency call, what should you do if you encounter a school bus unloading students with its red lights flashing

Stop and wait for all children to cross the road safely

You have been summoned to a field where a 32-year-old male patient has been struck by lightning. Emergency Medical Responders are with the patient, providing manual spine motion restriction. Once you arrive at the patient's side, what should you do first?

Assess the airway and breathing

Firefighters have pulled a middle-aged male from a burning bedroom. The patient is unresponsive and has obvious deformity of his left ankle. His respirations are rapid, stridorous, and shallow. Your partner inserts an oropharyngeal airway and begins ventilating the patient with a bag-valve mask appropriately. Which action would you perform next

Assess the patient's carotid pulse

The Apgar scoring system is useful in determining:

The newborn's overall condition

According to family members, a 24-year-old female suddenly began to act in a hostile manner. After assessing her, you determine that the patient cannot remember her name or present location and is drooling. Based on these findings, you should suspect a:

Behavioral emergency resulting from a physical condition

A 2-month-old presents with labored breathing, fever, and coughing. Auscultation of the lungs reveals significant wheezing bilaterally. Based on this presentation, an EMT would recognize that the patient most likely has:

Bronchiolitis

An injury to a hemisection of the spinal cord that disrupts the spinal tracts on only one side of the cord is:

Brown-Sequard syndrome

At the scene of a fall in a mall parking lot, you begin to suspect that the victim, who received only minor injuries from the fall and refuses transport, is a victim of human trafficking. What should you do next

Call the Human Trafficking Hotline

Why would iodine tablets be a benefit to administer to the general public after a nuclear bomb detonation

Iodine decreases the risk for thyroid cancer

A train carrying radioactive materials has derailed. The radiation safety officer informs you that you will be transporting a 31-year-old male who was exposed to the radioactive waves emitted by the material. As such, you should recognize that

The patient represents no contamination threat to you

33. Which sign or symptom best indicates the patient has experienced a spinal cord injury to the thoracic spine?

Tingling in the legs

A patient has a history of tendonitis to the right elbow. Based on this history, the EMT would understand that the

Tissue connecting the muscle to the bones of the elbow is inflamed

Your partner is off from work for an extended period because he is having surgery for a torn ligament. Which structure is in need of that surgery

Tissue that connects one bone to other bones

You have been called to care for a 21-month-old female who has been bitten by a dog. Given the patient's age, you would appropriately classify the patient as a(n):

Toddler

After being assaulted by her boyfriend, an 18-year-old female who is 33 weeks pregnant is found unresponsive in her bedroom. She appears to have been beaten in the head and abdomen, as evidenced by multiple hematomas and lacerations to these areas. Which assessment finding should concern the EMT most and should be addressed first?

VA. SpO2 of 93% on room air

If the red diamond on a National Fire Protection Association placard contains a "4," the EMT should recognize that the material within this container is:

Very flammable

How important is it to make accommodations such as advocating for the impoverished when providing medical care

Very important, because many Americans will live below the poverty level for at least a year

Which statement regarding voluntary muscles is true

Voluntary muscles generally connect to the bones of the skeletal system

An Emergency Medical Responder (EMR) quickly checks the breathing and pulse of a patient with Alzheimer's disease who has been sleeping outside in the cold for several hours. The EMR informs you that the patient grimaces to painful stimuli but is not breathing and does not have a pulse. What is your best response

"Let's perform the primary assessment again, and this time take longer to assess for breathing and a pulse."

A young male patient who was angry with friends wandered away from a party and spent the night outside uncovered in 40-50° F temperatures. He is confused, has decreased but adequate breathing, and has a weak radial pulse. His skin is cool and capillary refill delayed. You note that he is not shivering. Based on this presentation, what can you safely conclude?

The patient's body temperature is most likely dangerously low

As an instructor for safe ambulance operations, which statement should you make to new EMTs about responding with lights and sirens through curves in the road

"At night, turn your high beams off prior to entering the curve."

You have been dispatched to a residence where a woman lacerated her arm after falling while holding a drinking glass. She informs you that when she initially cut herself, blood spurted from the wound; then, within a second or two, it subsided to a mere trickle. She asks why this would happen. What is the correct response

"Blood vessels tend to constrict quickly once they are injured."

The EMT has fundamental understanding of abdominal trauma when he states:

"Blunt trauma is especially lethal because of the large number of organs that can be affected

Which question is best asked when attempting to determine the degree of sensation in the hand of a patient who has an elbow injury?

"Can you tell me which finger I am touching?"

Which statement made by an EMT demonstrates an understanding of chest trauma?

"Closed chest injuries are caused by blunt trauma, and can be just as serious as open chest

Which statement indicates an accurate understanding of administering positive pressure ventilation (PPV) to a geriatric patient?

"Gentle ventilations that make the chest rise are adequate to provide proper airflow to the lungs

A bleacher at a college has collapsed, and your crew is the first EMS unit to arrive on the scene. As you enter the gym, several patients are walking around with visible injuries. Which instruction is most appropriate for these ambulatory patients

"Go outside and wait by the red fire truck in the parking lot and stay there."

While performing the reassessment of a 2-year-old male who is severely dyspneic, you note that he is now unresponsive and not breathing. He has a heart rate of 32 beats per minute and his skin is cool and diaphoretic. Which instruction would you provide to another EMT who is helping you transport the patient?

"Let's start chest compressions and positive pressure ventilations."

Which statement made by the EMT indicates an understanding of the relationship between seizures and pregnancy

"Prolonged seizures can easily cause the mother and baby to become hypoxic."

While running in a parking lot, a young boy tripped and fell. He hit his face and mouth on a curb, knocking one of his top front teeth from its socket. Bleeding has been controlled and the tooth found. Which instructions would you give your partner when caring for the tooth

"Rinse the tooth first, then put it in a cup of saline for transport."

You have been called for an 84-year-old female who fell and hit her head. The patient is confused and has a laceration on her forehead. When getting the history from family members, which statement suggests that the patient is suffering from delirium?

"She could not remember her name yesterday."

The EMT understands the importance of evaluating the mechanism of injury when he states:

"The mechanism of injury can be used to guide the EMT's injury assessment and treatment of the patient."

The friend of an impoverished SO-year-old male called 911 because his friend had chest discomfort. On scene, the patient informs you that he does have a cardiac history and has not seen his cardiologist in more than a year because he lost his job. The patient does not want treatment or transport since he knows that both are expensive and he will be unable to pay for such care. In this situation, what would your best response be?

"You may be eligible for free or reduced-cost medical care."

A 22-year-old male was fishing when he was submerged for approximately 30 seconds after he moved into deeper water to untangle his fishing line. He states that he did take some water into his lungs but feels better now and does not see the need to be transported to the hospital. Which response is most appropriate

"You seem okay now, but it may take a day or two for complications to become apparent. ou really should be checked out in the hospital."

You are at the scene of a multiple-casualty incident where a church van carrying 15 parishioners overturned. You and your partner are in charge of the treatment unit and are presented with four patients. Which one will you treat first

A confused 69-year-old male with a respiratory rate of 42 breaths/min and a rapid and weak radial pulse

You are using the Simple Triage and Rapid Transport (START) system. Which patient should have a yellow tag applied prior to being moved to the treatment area

A female with a respiratory rate of 22 breaths/min, a palpable radial pulse, and who squeezes your fingers when instructed

Which rescuer would be the most appropriate choice to rescue an injured man clinging to an overturned boat in a shallow lake?

A firefighter who is wearing a personal flotation device, does not have any medical training and is a good swimmer who is trained in water rescue

At a construction site, an 18-year-old male was struck in the head by a pallet of concrete blocks as it was being moved by a crane. He has a large laceration to the left side of his head, with noted instability to the skull beneath the wound. Moderate bleeding from the injury site is also noted. The patient's airway is open, his breathing is adequate, and his pulse is strong and regular. Proper care for this patient should include:

A loose dressing over the top of the laceration

A 61-year-old male who became dizzy and fell down five stairs, and who has a laceration to the head and open fracture of the left lower leg

A patient, who was constructing a bomb in his basement, has sustained a secondary-phase blast injury. Which presentation would the EMT most likely observe from this blast phase

A female patient who is 36 weeks pregnant with twins tripped in a parking lot, hitting her head, her wrist, and her abdomen. She is alert and oriented and complains of a headache and abdominal and wrist pain. She denies shortness of breath and has an SpO2 of 96% on ambient air. Assessment reveals a large abrasion and hematoma to her forehead and abdomen as well as deformity to her left wrist. Which intervention would best treat the injuries the fetuses may have suffered

Administering high-concentration oxygen through a nonrebreather mask

A patient has been splashed in the face with battery acid. He now complains of being unable to see anything and of severe burning to his face. You observe that he is holding both eyes tightly shut and has significant burns on his cheeks and eyelids. In this situation, the EMT would assess the eyes:

After completing the primary assessment

A 67-year-old female complains of severe left lower leg pain. She states that she was walking across the room when she suddenly heard a "crack" and felt excruciating pain in her leg. Her left tibia appears to be deformed and ecchymotic, as well as tender on palpation. Her vital signs are pulse 96 beats/min, respirations 28 breaths/min, blood pressure 168/74 mmHg, and SpO2 98%. Additionally, the patient states that her doctor told her that the steroids she takes can weaken her bones, placing her at greater risk for fractures. Appropriate care for this patient would include:

Application of a splint to the leg

You are called to a nursing home for an elderly male patient who is unresponsive. On arrival, you find the patient in a wheelchair with a noticeable hematoma to his forehead and a skin tear to his left wrist. Staff states that the patient had an episode of chest pain just before suddenly passing out and falling to the ground. What should you do immediately?

Apply spine motion restriction precautions

You have been put in charge of creating a protocol that will direct care provided by EMS. In regard to an ice-water drowning patient who is severely hypothermic and in cardiac arrest, what treatment would you propose?

Apply the AED and give one shock if indicated. Contact medical direction if additional shocks are indicated by the AED

A 9-year-old boy has fallen from a swing. Assessment findings reveal no threats to the airway, breathing, or circulation, but do indicate deformity, pain, and swelling to his right wrist. The right radial pulse is strong and is accompanied by skin that is pink and warm to the touch. Which action indicates that the EMT is properly caring for this patient?

Applying cold packs to the wrist to reduce swelling

An elderly female fell and is now very confused. You are having a problem getting her past medical history. Her daughter states that she is typically alert and very well oriented. What is your next best action?

Ask the patient's daughter for the history

A confused patient with a tracheostomy tube is breathing shallowly at 8 times per minute. The family states they accidentally overdosed him on some of his medications when two different family members were caring for him at two different times today. The patient is nonverbal because of the tracheostomy and has an SpO2 of 81% on room air. What would your immediate action be?

Attach a bag-valve device to the tracheostomy tube and initiate ventilation

A 31-year-old groundskeeper has been stung by bees three times in his right arm and twice to his left ankle. After performing the primary and secondary assessments, which reveal no acute disturbances, what should you do next

Attempt removal of any embedded stingers

As the EMT responsible for triage at a motor vehicle collision, you find a patient who cannot walk due to his injuries; the patient is breathing 18 times per minute and has a radial pulse. What should you do next

Check the patient's mental status

Which situation represents simple access to a patient

Climbing stairs to gain access to a patient on an upstairs patio floor

Which type of hallucination is most dangerous

Command

On scene, you are by the side of a 3-week-old male who was born prematurely and has an apnea monitor on. His mother states that she was awakened by the alarm and when she went to check on the baby, he was awake and crying. Assessment reveals him to be resting quietly in his mother's arms. The airway is patent and respirations adequate. Vital signs are pulse, 136 beats/min; respirations, 28 breaths/min; and SpO2, 99% on room air. Since the local emergency department was busy when you were there dropping off a patient about an hour ago, it would be appropriate to:

Continue assessment of the baby, treat any problems found, and initiate transport

A patient was building a bomb in his garage when it accidentally detonated. Assessment findings from the rapid secondary assessment reveal part of the intestines and a portion of the liver protruding from the right side of the abdomen. Proper care of this injury would include:

Covering the exposed organs with a large sterile dressing soaked with sterile water

Which item best describes use of a weapon of mass destruction

Death of 49 people in a crowded mall following intentional dispersal of an unknown bacterial agent

When classifying a burn as either superficial, partial thickness, or full thickness, the EMT would need to determine the:

Depth of the burn

Which abdominal organ or structure, if seriously injured, would cause the most rapid death?

Descending aorta

An 85-year-old male with a history of an irregular heartbeat, diabetes, renal failure, and stroke fell while gelling out of bed, striking his head on the nightstand. He is alert and oriented and denies any complaint. Vital signs are all within normal limits. There is a minor abrasion on his forehead, but he denies any associated pain or headache. What should be your next priority?

Determine whether he takes blood thinners for the irregular heartbeat

A young male was riding a motorcycle when he lost control and was thrown from the vehicle. The patient is unresponsive and receiving positive pressure ventilation. The secondary assessment reveals bright blood flowing from an unrecognized femoral bleed. Initial management of this bleeding would include:

Direct pressure with a dressing applied

A teenage female patient was ejected from a car as it rolled down a hill after she ran off the road. She is responsive, but with signs of early shock and respiratory distress. She has bruising to her anterior abdomen and chest, a laceration to her left temple, and deformity to her left wrist. At which point should the wrist injury be splinted?

En route to the hospital

You are caring for a conscious 81-year-old male who complains of weakness. The patient has a hearing impairment and is wearing a hearing aid in his right ear. As you attempt to get a history, he shakes his head, claiming that he cannot hear you. What should you do immediately?

Ensure that the hearing device is turned on

Which disease or illness would you suspect when confronted with a pediatric patient exhibiting an acute onset of respiratory distress and an audible inspiratory strider sound, but no cough

Epiglottitis

A combat veteran's wife called 911 after her husband suddenly became confused and agitated. According to the wife, the patient takes several mood-stabilizing drugs and has a history of traumatic brain injury (TBI) from an improvised explosive device (JED) while on tour in the Middle East. There is no other medical history. On scene, in the patient's bedroom, the patient is staring at you with a face that appears angry. What is your initial priority in the care of this patient

Establish a rapport with the patient and ensure personal safety

Which of these is a high-velocity wound that carries the greatest risk for death when considering the concept of cavitation?

Gunshot wound to the spleen

A 17-year-old male with cerebral palsy and severe developmental challenges fell down five stairs and is crying. To best determine if the patient is in pain, the EMT should

Have the father ask the patient

A male patient experienced an injury that completely severed his spinal cord in the thoracic spine. Which presentation would be associated with this injury

He will experience paralysis to the legs

On the National Fire Protection Association hazardous materials classification placard, the blue diamond represents

Health hazard

You have been called to a high school science lab, where a male student was splashed in the eye with an alkaline solution. The patient complains of burning to the eye as well as pain. After performing the primary assessment and finding no life-threatening conditions, you would:

Hold the patient's eye open and begin flushing with large amounts of tap water, and continue this intervention throughout transport

A patient sustained a crush injury after being pinned by a car that fell off jack-stands while the patient was underneath the car working on it. The injury occurred to the left upper quadrant of the abdomen. Assessment shows bruising to that area, with instability noted to the lowest rib. The EMT's highest concern regarding this injury should be

Injury to the spleen

When suctioning a 46-year-old male with a tracheostomy tube, which procedural step is appropriate

Insert the soft suction cannula until mild resistance is met and then initiate suction

The driver of a truck carrying a radioactive substance accidentally came in contact with the material after getting into a crash on the freeway. You are first on the scene, and a quick observation indicates that the driver still has the powder on his arm. Your immediate action would be to:

Instruct the patient to remain still and wait for the radiation safety officer

You have completed the assessment of a 32-year-old patient who is starting her seventh month of pregnancy. During your assessment, which finding should you be most concerned about?

Intermittent vaginal bleeding

The rapid secondary assessment of a male patient who was thrown off a motorcycle indicates instability and pain on palpation to the pelvic area. What is he EMT's primary concern related to this injury?

Internal blood loss

A patient who experienced blunt trauma to the abdomen has a lacerated liver. Given this information, what would represent the most immediate threat to life

Internal hemorrhage

You are instructing a group of new EMTs on the basics of driving an emergency vehicle. What would you emphasize as the most frequent situation or location in which collisions involving ambulances occur

Intersections

An intoxicated 24-year-old female patient has been struck by lightning and is conscious but confused. Which of these assessment findings should the EMT investigate further first?

Irregular heartbeat

Why is the uterus essential to a healthy pregnancy and childbirth?

It powerfully contracts to force the fetus from the mother's body following gestation

You have had to restrain a patient who is being involuntarily committed to a psychiatric institution. During the restraint process, the patient vomited. How should you position him on the stretcher?

Left lateral recumbent position

A car has been hit head-on by another vehicle at a moderate rate of speed. Seat belts were in place and the air bags deployed. The patient complains chest pain. Quick observation reveals a bruise to his sternum. The EMT should immediately:

Lift the air bag and check for steering wheel deformity

You arrive on the scene of a two-car motor vehicle collision. The patient was the unrestrained driver of a car that struck another car from behind while moving at a speed of 25 mph. During the collision, the patient flew forward and struck the windshield with his head. No air bags were deployed. The patient extricated himself and denies head, neck, or back pain; however, you do note a small cut on his hand, which occurred as he was getting out of the car. When asked about preexisting medical problems, he tells you that he did have herniated disks in his lumbar spine that were surgically repaired several years ago. Based on this information, your strongest reason to initiate spine motion restriction precautions for this patient would be:

Mechanism of injury

During scene size-up on an unknown medical call, which clue would best indicate that the patient may be a veteran of the military

Military-style haircut

While returning from a call, you are flagged down by a frantic family member of a 12-year-old male patient. She is standing in the yard, and reports that the child was shot in the abdomen and is now inside the house. She states that she has already called the police, who have not arrived yet, and that the shooter is another family member who has since left the scene. What is your best course of action

Move the ambulance and wait for law enforcement to arrive

You have been called for a geriatric patient with advanced dementia. The staff at the geriatric extended care facility believes that the patient has a urinary tract infection. At bedside, you find the patient to be confused and aggressive. She is on constant oxygen at 3 1pm for COPD. During transport, she repeatedly yells at you and strikes out at you. You would respond by

Moving away from the patient so she cannot strike you

A 57-year-old diver has been brought to the surface by fellow divers. He complains of difficulty breathing, dizziness, and severe chest pain. He states that he has a cardiac history and takes nitroglycerin, which is in his belongings nearby. Emergency Medical Responders report the following vital signs: pulse 128 beats/min, respirations 18 breaths/min, blood pressure 88/56 mmHg, and SpO2 93%. After applying supplemental oxygen to the patient, your next action should be to

Perform a secondary assessment

When performing the secondary assessment, which sign is most suggestive of a spinal cord injury?

Persistent penile erection

Beta radiation represents the greatest threat to a person's well-being when the

Person ingests it by eating contaminated food

A 22-year-old female, who is eight months pregnant, calls 911 for vaginal bleeding. Your primary assessment reveals no acute life threats, but the patient does have some evidence of blood in her underwear. The patient denies abdominal pain. Her vital signs are normal. Given these findings, the EMT should be suspicious of:

Placenta previa

Prior to delivery, the EMT prepares to create a sterile field around the patient's vaginal opening. This is best accomplished by:

Placing a sheet from the OB kit under the patient's hips and another over her abdomen and legs

You are assessing a patient who had a previous brain injury in which a portion of the cerebellum was destroyed. In relation to this specific injury, which finding would the EMT expect?

Poor muscle coordination when signing his name to the prehospital care report

A patient who was found face-down in a pond is unresponsive, apneic, and pulseless. Friends state that the patient was hot and wanted to take a swim in the pond. Which intervention should be included in your care of this patient?

Positive pressure ventilations at 10 ventilations per minute with supplemental oxygen

During which abnormal birth presentation is it permissible for the EMT to place a gloved hand into the vaginal canal?

Prolapsed umbilical cord

Which of these conditions, in addition to cool temperatures, is necessary for trench foot to develop?

Prolonged exposure to moisture

Family members report that their 62-year-old mother complained of dizziness and right leg weakness just before falling down a flight of stairs. Your assessment indicates that the patient is responsive, but confused and talking with garbled speech. Her breathing is adequate and her radial pulse is strong. Vital signs are pulse 84 beats/min, respirations 18 breaths/min, blood pressure 188/110 mmHg, and SpO2 96%. There is an open fracture to her right forearm. When instructed to do so, the patient does not move the fingers of her right hand. When caring for this patient, which sequence of actions seems most appropriate

Provide spine motion restriction precautions to include a cervical collar, long spine board, and straps before extricating her to the ambulance for immediate transport

An elderly woman experienced a closed head injury when she fell down five steps. She is responsive to painful stimuli and her breathing is irregular and shallow. When evaluating her airway, you note that she has both upper and lower dentures. Her upper dentures are secure, but her lower dentures are loose. Prior to providing positive pressure ventilation, you would:

Remove the lower dentures

You have been called to a residence for a 63-year-old female with a history of COPD who is short of breath. The patient is moderately dyspneic with the following vital signs: pulse, 124 beats/min; respirations, 20 breaths/min; blood pressure, 162/56 mmHg; and Sp02, 86%. She is using an oxygen concentrator set at 4 1pm. What is your best course of action

Remove the oxygen concentrator and administer high-concentration oxygen to the patient

You are at a scene where a patient was calm and compliant until a few seconds ago. Now he is brandishing a knife and threatening to kill you and your partner if you do not immediately leave. The man is intoxicated and police have been notified but have not yet arrived. What is your best course of action

Remove yourself, your partner, and family from the house

A 61-year-old male fell off a roof. Your primary assessment findings include unresponsiveness, agonal breathing, and a slow and weak radial pulse. The patient's skin is cool and dry. Emergency Medical Responders are maintaining manual spine motion restriction procedures. What should you do immediately

Start positive pressure ventilation

You are assessing a male patient who was stabbed three times in the chest and abdomen. As you begin your primary assessment, you note the patient to have a decreased level of consciousness and gurgling respirations. Your next action would be to

Suction the airway

You have been called to a preschool for a seizing child. On arrival, staff directs you to a bed in which a 4-year-old boy is lying. Staff state that the child began making "funny" noises in his sleep, then began to seize for approximately 60 seconds. Your assessment indicates that the boy is responsive to painful stimuli and is breathing, but has gurgling sounds coming from the upper airway. His radial pulse is rapid and strong, and his skin is cool and diaphoretic. What is your immediate action in caring for this child?

Suction the airway

A 23-year-old male struck his face on the steering wheel when his vehicle hit another car from behind. The patient's nose is deformed and actively bleeding. In his care, which intervention would be a priority

Suction the airway to keep it free of bloo

When assessing a patient, which finding should raise the EMT's suspicion that the patient may have a pulmonary embolism?

Sudden onset of shortness of breath with clear breath sounds

A 24-year-old intoxicated female patient was pulled from a pool by hotel personnel. The primary assessment reveals her airway to be open and her breathing adequate. Her radial pulse is strong but fast, and she is persistently coughing. Vital signs are pulse 144 beats/min, respirations 24 breaths/min. blood pressure 166/100 mmHg, and SpO2 92%. What would be considered appropriate care for this patient

Supplemental oxygen through a nonrebreather mask

An alteration in brain function or other brain pathology caused by an external force that causes structural brain damage is described as

TBI

According to the neonatal resuscitation pyramid, which level of care will most newborns require

Tactile stimulation and drying

When teaching Emergency Medical Responders about air bags, which point would you emphasize

The benefit of air bags is lessened in a multiple-vehicle collision impact

Assessment of a patient involved in a motor vehicle accident reveals him to have crepitus to the left humerus, instability to the left lateral chest wall and flank, and pain on palpation to left hip region. The patient also complains of pain to the right side of the neck. Based on this injury pattern, the EMT would recognize the patient was

The driver of a car hit on the driver's side

Which statement best characterizes the capabilities of a Level Ill trauma center?

The hospital has some surgical capabilities to help trauma patients, but will generally stabilize and them and still arrange transfer

You arrive on the scene to help another crew with the extrication from an attic of a female who fell while retrieving holiday decorations. As a result of the fall, the patient has an open fracture to her left lower leg. The patient has been fully immobilized to a long board and has a rigid splint in place to her left leg. What would indicate the extremity has been improperly splinted

The patient is able to flex her left ankle when directed to do so

The EMT is correctly assessing breath sounds on a 3-year-old male when he:

Understands that sounds from the left lung can be heard on the right side of the chest

On scene, a 6-year-old male who is short of breath will not allow EMRs to place a nonrebreather face mask on his face. Assessment indicates moderate respiratory distress with the following vital signs: pulse, 124 beats/min; respirations, 24 breaths/min; blood pressure, 98/56 mmHg; and SpO2, g2%. At this time, you try:

Using a nasal cannula with 2 lpm oxygen

A vacuum splint has just been applied to the injured arm of a patient who fell backward from a chair while hanging drapes. Which statement or question should the EMT make or ask next after placing the splint

"I am going to feel your wrist for a pulse now."

A car swerved to miss a deer in the road and hit a utility pole at a low rate of speed. Damage to the car is minimal, but the 82-year-old male passenger was not wearing a seat belt and hit his head on the dashboard. He presents as confused and diaphoretic. His daughter, the driver, states that he has a history of dementia, high blood pressure, and diabetes, as well as coronary artery disease. She adds that he took his insulin about 2 hours ago. Which question to the daughter would be most important at this time

"Did he eat anything today?"

Family has called you for a 78-year-old female with a history of dementia, because she appears more confused than normal. The primary assessment reveals no life-threatening conditions. Her vital signs are pulse, 88 beats/min; respirations, 22 breaths/min; blood pressure, 134/66 mmHg; and SpO2, 95% on room air. Aside from the dementia, the patient has a history of hypertension, COPD, and breast cancer, which resulted in the removal of her right breast 11 years ago. Lately, she has had an upper respiratory infection and is telling family that she has a headache. Your first question to family should be:

"Do you know if she fell recently and hit her head?"

Which instruction is appropriate when helping a new EMT care for a male patient with a laceration to his left arm

"Don't worry about removing embedded dirt from the wound, just the dirt around the laceration

A patient has full-thickness burns to the fronts of both legs and the entire left arm. Using the rule of nines, the EMT should estimate what percentage of the patient's body has been burned?

27 percent

A male patient has been shot in the chest with a small-caliber gun. Which presentation indicates that he is in an early stage of shock?

Alert and anxious, pulse 96 beats/min, BP 134/88 mmHg, pale and cool skin

A 13-year-old boy flipped his bicycle and landed on outstretched arms, which caused a fracture to his collar bone. This is an example of: "'A. An indirect force injury

An indirect force injury

As an EMT, you realize that post-traumatic stress disorder (PTSD) occurs when

An individual is exposed to an abnormal or dangerous condition that is difficult to mentally handle

Despite the increase in maternal blood volume, the pregnant patient is typically:

Anemic

You have arrived at a scene where a young girl tripped while running, and pushed her arms through a plate glass window while falling. She is responsive to painful stimuli and has blood spurting from a large laceration on her right upper arm. Your partner yells for you to immediately apply direct pressure to the laceration to prevent further blood loss, while he works to control the airway. You have yet to apply gloves, but your uniform shirt has long sleeves, and you normally wear prescription glasses. What would you do next?

Apply gloves prior to applying direct pressure over the injury site

While working to build a house, a construction worker was hit in the head by a 2-inch x 4-inch plank that slid from the roof and fell to the ground below. The patient has a sizable laceration to the left side of his face and is spitting and coughing blood. He also lost several teeth, two of which he is holding. Your initial action when caring for this patient would be to:

Apply supplemental oxygen

You are assessing a patient who was burned when she dropped a hot iron on her bare foot. Her foot is red and has a small partial-thickness burn to the anterior portion of the foot. When completing the pre hospital care report, which mechanism of injury would you document?

Contact burn

An unhelmeted rider, who was thrown from his motorcycle when he wrecked, has a large scalp avulsion with obvious skull depression to the top of his head with venous bleeding. These findings suggest which type of injury?

Open head injury

On follow-up, the ED physician informs you that a patient whom you transported earlier was found to have a hemothorax. In this condition

Blood collects in the chest cavity, collapsing the lung

Which statement regarding contusions is true?

Blood vessels in the dermal layer have ruptured

A woman has called you to help her middle-aged friend, who has stopped eating and talking and has frequent crying spells. Your history reveals that the woman lost her husband a few months ago, and has been withdrawn and despondent ever since. Throughout the assessment, the alert and oriented woman continually states that she wants to be left alone. When you bring the stretcher into the room, the woman states, "I do not want to go to the hospital. Let me sign whatever I need to sign." What is your best response to this situation?

Contact medical direction and ask for advice

An unrestrained female patient, who was involved in a minor motor vehicle collision, refuses EMS care or transport. She is seven months pregnant and is alert and oriented. When asked, she denies any complaint. Your best action would be to

Contact medical direction for instructions

An 18-month-old child has a temperature of 105.2° F. Without treatment, what will most likely occur?

Cellular dysfunction or seizures

At the scene of a motor vehicle accident, a car has extensive front-end damage and a deformed steering wheel. The restrained driver, who was self-extricated, is in the care of EMRs, who have applied a cervical collar and continue to maintain spine motion restriction precautions. The patient's airway is patent and respirations adequate. His radial pulse is rapid and weak, and his skin is cool and diaphoretic. The EMRs report the following vital signs: pulse, 136 beats/min; respirations, 20 breaths/min; blood pressure, 100/68 mmHg; and SpO2, 95% with supplemental 02 being administered. The right forearm is deformed, and the head, chest, and upper back are uninjured according to your assessment. Your next action would be to:

Check the abdomen for injury

An awake but confused patient with a history of diabetes accidentally spilled a pot of hot water on himself while cooking. He has a partial-thickness burn on his right anterior leg. Following an unremarkable primary assessment, which action seems most appropriate

Check the patient's blood glucose level

A 27-year-old male was involved in an altercation and was struck in the ribs with a baseball bat. Assessment reveals intact skin with bruising to the right lateral chest. When palpating this area, pain, instability, and crepitus are all observed. The patient also complains of difficult and painful breathing. The EMT would recognize:

Chest injury caused by blunt trauma

A young adult male is in police custody after he crashed his car into a utility pole. There is minor front-end damage to the vehicle, and the air bags deployed. The patient was not wearing a seat belt and has an abrasion to his forehead. He is awake and oriented to person, time, and place. His speech is slurred and he has an odor resembling that of beer on his breath. The arresting officer states that he just wants you to check the patient before he transports him to jail. Given this scenario, which action seems most appropriate for the EMT to take?

Contact medical direction with assessment findings

After you obtain a refusal at a residence in a neighborhood, a couple approaches the ambulance and inform you that they have concerns that a 6-year-old child who lives in a home two doors away may be physically abused. They state that they heard the child crying and what sounded like physical abuse occurring. In this situation, you would:

Contact the police and wait in the ambulance until they arrive

A 92-year-old female informs you that she has diabetic retinopathy. You recognize that this condition involves

Damage to blood vessels in the eye

When cooling a patient with possible heat stroke, which of these findings would be of most concern to the EMT?

Determining that the patient is now shivering after having cold packs applied to the neck

A female patient is 34 weeks pregnant, and you are delivering the baby. As soon as the baby is completely delivered, you should immediately:

Dry the baby with a warm towel

A 32-year-old female complains of the sudden onset of sharp pain to the left side of her abdomen. She denies any medical history and when asked, admits to being sexually active. Her last menstrual period was two months ago. As an EMT, you should immediately suspect

Ectopic pregnancy

The use of Class 2 or 3 ANSI high-visibility vests when operating on a roadway is a requirement of

Federal law

One of the four essential features of post-traumatic stress disorder is:

Feelings of anger

The EMT is correctly assessing the perfusion status of a 10-month-old when she

Feels for a brachial pulse

When assessing a central pulse of a young child, the EMT would palpate the:

Femoral pulse

A patient who has been exposed to an encephalitis-like agent most likely will complain of:

Fever and confusion

You have been called for a female patient with a history of bipolar disease who is severely depressed and stated suicidal intentions. Which type of affect would you expect this patient to display?

Flat

The function of the lens of the eye is to:

Focus light onto the retina

A young female patient was weeding her garden when she was bitten on her left hand by an unidentified snake. The primary assessment does not reveal any life threats and vital signs are stable. In caring for this patient and her injury, it is critical that the EMT:

Have the patient remove her wedding ring from her left hand

A patient who is completely reliant on his gastric tube for feeding has accidentally pulled it from his body. Which statement is most accurate about this patient?

He is unable to receive his oral medications

After the EMT observes that secretions are interfering with the baby's breathing, which action indicates that the EMT is properly suctioning the newborn?

He squeezes the bulb syringe prior to placing it in the baby's nose

A young male golfer has been struck in the right eye with a golf club. The upper and lower lids are swollen shut and ecchymotic. There are also blood clots between the two eyelids. What would be an indication that the EMT is providing proper care to this patient?

He wipes any remaining blood from the face but not the eye

A patient with severe head and neck pain states that he was properly wearing his seat belt when another car struck him from behind. As a knowledgeable EMT, you would realize that the complaint of

Head and neck pain could occur even when the seat belt is properly worn

When it is necessary to secure a patient to a long spine board during a spine motion restriction process, which of these regions is typically secured last to the spine board?

Head and neck region

Which assessment finding best indicates that the newborn is significantly distressed?

Heart rate of 92 beats per minute

Which finding indicates that a patient who received a blow to the head is suffering from something other than a simple concussion?

His pupils are noticeably unequal when assessed

If a patient is exposed to a nerve agent, which assessment finding would the EMT expect

Increased salivation

You have been called for a 44-year-old male who was burned on the right hand and arm after falling against the hot door of an industrial incinerator. When you arrive on the scene, a nurse employed by the company informs you that he sustained a partial-thickness burn. Based on this information, the EMT should expect which characteristics at the burn site

Intense pain and blister formation

A 57-year-old man tripped and fell on the sidewalk, hitting his head on the concrete. According to witnesses, the patient was unresponsive for several minutes following the fall. Which bit of information would be most critical to relay to the physician in the emergency department?

Known history of alcoholism

A female patient has acute onset of right-sided abdominal pain, describes it as sharp, and rates it as 9/10. The primary survey reveals no life threats, but her skin is cool and diaphoretic. She states that she has been going to the bathroom more and is worried because her father has diabetes. Her last period was nine weeks ago, and she has a history of irregular periods. Her heart rate is 122 beats/min, respirations are 20 breaths/min, blood pressure is 138/68 mmHg, and SaO2 is 97%. In this scenario, which bit of information is of most concern to the EMT

Last period nine weeks ago

How can an EMT help prevent the stress and anxiety associated with treating critically injured children

Learn and practice the skills and equipment used when treating children

During transport, a female patient who is restrained begins to cry and states that she is sorry for becoming violent and will not behave in that manner anymore. She requests that the restraints be removed because they are hurting her legs and she does not wish to be restrained any longer. After ensuring that the restraints are not too tight and not harming the extremities, you would:

Leave the restraints in place until patient care is transferred to the hospital

A football player was struck in the head during a hard tackle. and is reportedly demonstrating retrograde amnesia to the event. Currently the patient complains of a headache and nausea and reports that he has pain to his upper back and tingling in his left hand. The EMT is providing proper care for the patient when he:

Leaves the helmet in place but removes the face mask first

You are transporting a 19-year-old male with a gunshot wound to the chest. On scene, you covered the wound with an appropriate dressing. Now, as you reassess the patient, you note that the patient is tachypneic and complains of increased difficulty breathing. Breath sounds on the side of the injury are diminished. Your immediate action would be to:

Lift the dressing from the wound briefly during exhalation

When an EMS unit is alerted to an emergency call from dispatch, which piece of information is it most essential that the EMT obtain first

Location of the call

What does the "M" in the mnemonic TRACEM stand for

Mechanical

Which thermometer should be utilized by an EMT when assessing the temperature of a patient with hypothermia?

Medical-grade thermometer in the patient's rectum

When planning for a terrorist attack. which concept makes the best use of resources that are locally available

Mutual assistance

A 6-year-old male has a decreased level of consciousness. His mother states that the family does not have health insurance, so they did not take him to the hospital last week when he started vomiting and had copious amounts of diarrhea. She reports that he has not had anything to eat or drink since then. The patient has snoring respirations that are rapid and inadequate. His radial pulse cannot be located, and his carotid pulse is rapid and weak. His capillary refill is 5 seconds and his skin is cool to the touch. What would your first intervention in caring for this child be

Open the airway using the head-tilt, chin-lift maneuver

Which description best fits an oxygen concentrator that may be part of a patient's home medical equipment?

Oxygen device that extracts oxygen from the air

A 40-year-old homeless male is found lying in the street. The physical examination reveals that the patient has a stab wound to the mid-chest. He is unresponsive and his skin is cool to the touch. His respirations are snoring at 40 breaths/min and his pulse is 120 beats/min. Which care should the EMT provide firs

Perform a jaw-thrust maneuver

A minivan has struck a utility pole. The driver is unresponsive and has life-threatening injuries. Which option would be most appropriate when extricating the patient from the vehicle?

Perform rapid extrication with a cervical collar applied

After assessing a 67-year-old male, your partner informs you that it appears the patient had a stroke and has dysarthria. In relation to the dysarthria, you would anticipate the patient being unable to

Pronounce words correctly

When working in the "warm" zone, the EMT must remember that:

Protective gear must be worn in this zone

An 84-year-old male complains of generalized weakness and an ache in his left shoulder. He also reports mild shortness of breath, but has clear lung sounds and an SpO2 of 93%. When asked, he reports that the discomfort started after he got out of bed 2 hours ago. He denies any known injury to his shoulder or back. He has a history of coronary artery disease, hypertension, TIA, and irritable bowel syndrome. Family members state that the patient has early dementia. There are no obvious life threats, and his vital signs are normal for his age. The appropriate treatment for this patient would include:

Provide oxygen and treat the patient as a cardiac patient

You are accompanying a team of hikers climbing a high mountain. In the morning, you are summoned to a tent and find one of the climbers confused and complaining of a headache. His airway is patent and respirations adequate. He has no medical history and was in good health until found this morning by his friend. You are suspicious of high-altitude cerebral edema (HACE). What should you do to help the patient?

Provide supplemental oxygen

You are delivering a baby in the patient's house. As the baby's head appears at the opening of the vagina, you notice that the amniotic membrane is still intact. You would

Rip open the amniotic sac with your fingers

A vehicle has rolled onto its passenger side after running a red light at an intersection and striking another car. The restrained driver remains in the car, screaming in pain. To manage this scene, what should you do first

Stabilize the vehicle

A fisherman fell through the ice and was submerged in a lake for approximately 4 minutes before being pulled out by friends. You find the man to be unresponsive, not breathing, and without a pulse. What should you do immediately?

Start cardiopulmonary resuscitation

Assessment of a patient who complains of the sudden onset of chest discomfort reveals a central intravenous catheter to his right upper chest. While assessing this piece of medical equipment, which finding related to the catheter should be of greatest concern to the EMT?

The line is unclamped and without a cap

A young female driver, who was involved in a motor vehicle collision, complains of cervical pain resulting from a lateral-type mechanism of neck injury. Based on this information, which type of collision most likely took place

The patient's car was struck from the side

At an industrial complex, a young male was struck in the head by a large piece of steel thrown from a grinding machine. Assessment reveals a soft, painful depression underlying a tear in his scalp, which is covered with dried blood. Clear fluid is draining from his right ear. The patient is conscious but confused and exhibits the following vital signs: pulse, 84 beats/min; respirations, 16 breaths/min and adequate; blood pressure, 142/76 mmHg; and SpO2, 90%. Emergency Medical Responders are on scene and are maintaining manual spine motion restriction precautions. Which instruction indicates the next step needed in this patient's care?

Will someone administer supplemental oxygen to the patient?

For routine disinfection of a stethoscope after each call, the EMT should

Wipe it with a solution of 1 part water and 10 parts bleach.

Which statement shows that the EMT understands the use of a tourniquet in controlling bleeding

"A tourniquet should be applied tightly enough that arterial blood flow distal to the tourniquet is completely stopped

What is the level of training for those who respond to the release of hazardous materials so as to protect people, property, and the environment

First Responder Operations

Which statement is true regarding tactical EMS

Competency in core skills is typically required before responders can participate in a TEMS course

When at the scene where a patient has attempted suicide, what is the most immediate priority

Crew safety

A splint applied too loosely can lead to which complication

Laceration of previously intact blood vessels

During an in-service program, your medical director asks if anyone can tell her the cause of heat cramps. What response would be most appropriate?

"They are caused when the patient's electrolytes such as salt and calcium become deranged and are too low in the body.

The EMT shows that he understands care of the post-delivery mother when he states:

"Allowing the baby to nurse immediately after delivery can help to control uterine bleeding."

You have been called for a young male patient who has been acting withdrawn. As a precaution, dispatch also alerted police, who are now on scene with you. The patient appears depressed and when you ask him if he is thinking about hurting himself, he answers, "Yes." The next question asked by the EMT should be:

"Do you have a plan?

During an in-service program focusing on care of the trauma patient, the medical director asks if anyone can correctly describe the "platinum 1 O minutes." What statement by a participant is the best response?

"EMS should initiate transport of the critically injured trauma patient to the hospital within 10 minutes of arriving on scene."

How many newborns will require a level of care that falls below the top tier of the neonatal resuscitation pyramid?

20 percent

The greatest benefit of applying a traction splint in the field to a deformed femur and thigh is that ii can:

Decrease the pain

The EMT is performing CPR on a neonate. What is the correct compression-to-ventilation ratio?

3 compressions to every 1 ventilation

By the time a fetus reaches full term, the tidal volume of the female has increased by

40 percent

A 54-year-old male patient has been involved in a car crash. Window glass has caused an open wound to his upper neck. Which dressing is best for this injury?

Occlusive dressing

When applying a cervical collar to a young boy who fell out of a tree, the EMT realizes the main purpose of the collar is to

Remind the patient not to move his head or neck

Assessment of a newborn indicates that his heart rate is 120 beats per minute. How many points should he be awarded according to the Apgar scoring system?

2

Which of these patients is losing body heat by the mechanism of convection?

A 30-year-old male in a cool temperature and exposed to a light breeze

An alert and oriented 22-year-old male fell off a tractor and now complains of pain to his right leg. As you quickly scan the patient, you note deformity to the tibia-fibula area of the lower extremity. After conducting the primary assessment and manually stabilizing the leg, which action should the EMT take?

Check for a pedal pulse

As you approach a motorcyclist who was thrown from his bike, you hear him saying that he cannot feel or move his legs. You note obvious deformity to both femurs as well as to his left wrist and forearm. Emergency Medical Responders are holding manual cervical spine motion restriction and have already assessed the PMS in the extremities. Given this information, which assessment should be done next?

Check the patient's rate and effort of breathing

An 89-year-old female with dementia, who is being cared for by her daughter, fell and now exhibits deformity to her left upper arm. As you attempt to splint her arm, she continually screams and pulls away from you. After several attempts, your best course of action should be to

Coach the daughter on how to help apply the splint

When assessing a patient, which sign or symptom is most indicative of a pneumothorax?

Decreased breath sounds to the right lung

You are responding to a call for a 4-year-old child hit by a car. When assessing this child, which injury patterns would you recognize as typical based on the child's age and mechanism of injury?

Deformed femurs, bruises to the chest and abdomen, and head injury

Which assessment finding, in a 32-year-old female involved in a motor vehicle collision, should the EMT recognize as potentially life threatening?

Deformity of the left femur

Which assessment finding would contraindicate the use of the traction splint?

Deformity to the hip

You have been called to a local daycare facility for a sick child. At the scene, panicked daycare workers inform you that the 4-year-old child was lethargic all morning and wanted to sleep. After resting for several minutes, she began to "shake all over." This lasted for approximately 30 seconds. Your assessment reveals the patient to be responsive to painful stimuli and breathing adequately. Her airway is open and her radial pulse is strong and bounding. Her skin is hot and moist to the touch, with no signs of cyanosis. You are told that the patient has no medical history. En route to the hospital, the patient's mental status improves. Based on this presentation and information, you assume that the seizure occurred secondary to:

Fever

A young girl complains of eye pain after having sand thrown in her face by another girl at the park. When assessing her eyes, you note large particles of sand in the right eye. The sand appears to be on the conjunctiva, which also seems to be scratched and irritated. Your initial management of this injury would be:

Flush the eye with clean water or sterile water from the ambulance

You are managing a patient who has been consuming large amounts of water while playing a sport outside for several hours. The patient presents with general malaise, has a severe headache, has vomited once, and complains of photophobia. The patient's core temperature is just slightly elevated, and you see bloating in his hands. During transport, in which position should this patient be placed?

Fowler's position

On scene, you must lead a patient's visually impaired wife from the house to the ambulance. This is best accomplished by

Placing her hand on your shoulder as you walk to the ambulance

How can the EMT best keep a psychiatric patient calm and avoid provoking a violent outburst?

Inform the patient of all that is going on and use a calm voice throughout care

A patient has been bitten by what appears to be a black widow spider. The bite to her left hand is dull and achy, and the patient states that the muscles in er shoulders and back are starting to hurt, as if they are cramping up. After ensuring that no life-threatening conditions are present and administering oxygen, the EMT should:

Keep the hand below the level of the heart

Assessment of a 79-year-old female with a long history of medical complaints reveals tenderness and deformity to her left hip. Staff at the personal care home state that she slipped in some water and fell onto her buttocks. Currently, she is alert and oriented and describes the pain as 10/10. Regarding care of her leg, what is most appropriate care

Maintain the leg in the position found and transfer the patient to a long spine board

A young boy has an open chest wound to the left lateral thorax after falling off his bike onto a metal spike protruding from the ground. His family is present; they are very upset and crying. As an EMT, you recognize that in this situation, your primary focus is:

Maintaining oxygenation

A 15-year-old male with a history of seizures presents as oriented. He takes the anticonvulsant Keppra and has no other medical history according to his mother. The mother called EMS because the patient missed taking his medicine and she wants you to "check him out." Which action would be most appropriate for the EMT to take at this time?

Obtain a full set of vital signs

You are transporting a young and healthy female patient who was involved in a motor vehicle collision, during which she struck her chest on the steering column. Although the primary assessment reveals no life threats, the patient does have redness to her sternum, as well as pain and tenderness. Breath sounds are clear and present bilaterally, and vital signs are within normal limits. As you transport and reassess this patient, which sign or symptom would indicate the immediate need for ALS?

Onset of an irregular heartbeat

To find an implanted medication port on a patient, the EMT should:

Palpate the anterior chest

A 48-year-old male is in shock from injuries sustained in a motor vehicle collision. Which assessment finding(s) is (are) most suggestive that the patient may have internal bleeding

Pelvic instability with a BP of 78/48 mm Hg

Because of the improvements in life span and medical technology:

People with certain deficits or medical conditions can remain at home

A female patient has been involved in a motorcycle crash. Your scene size-up reveals her to be unresponsive and lying in the roadway. It also appears that her left leg has been amputated at the knee. What should the EMT do immediately?

Perform a jaw-thrust maneuver

An unresponsive 10-year-old male was struck by a car while riding his bike. He has blood on his face and presents with snoring respirations. Respirations are approximately 10 breaths per minute, with cyanosis noted to his extremities. Manual spine motion restriction to the neck is being held by Emergency Medical Responders. What should you do next

Perform a jaw-thrust maneuver

You have just arrived on the scene of a very serious motor vehicle collision. Emergency Medical Responders (EMRs) have rapidly extricated an unresponsive female from the driver's seat. The EMRs report that she was unrestrained and struck the steering wheel with her chest and abdomen. As you start the primary assessment, you note that the patient has snoring respirations and demonstrates shallow breathing at a rate of 24 breaths per minute. What should you do next?

Perform a jaw-thrust maneuver

You have been called to a hotel swimming pool for a confused adult male. Patrons state that the man was drinking most of the afternoon and was in and out of the pool. The man is now very confused and noncompliant with your requests. The patrons moved the patient so that he is lying on the ground, away from the water's edge, when you arrive. His airway is open and his breathing is adequate. His radial pulse is strong, and his skin is wet with water from the pool. While conducting the secondary assessment, you note a large hematoma to the back of his head. What should you do next

Perform manual spine motion restriction of his head and neck

An unrestrained middle-aged female was thrown from her car as it rolled in the median of an interstate. She is supine, appears unresponsive, has blood on her face, and has snoring respirations. Your first action upon reaching her side, after taking spine motion restriction precautions, should be to:

Perform the jaw-thrust maneuver

You have been called to a park for a teenage boy who is "sick." On scene, you find a 16-year-old male patient who knows his name but is confused about place and time. Friends state that they have been playing basketball most of the morning and afternoon. The temperature is in the 90s and the humidity is high. The patient's airway is patent and his breathing is fast but adequate. His pulse is weak and rapid. Observation of his skin reveals it to be pale, cool, and moist. Oxygen is being administered via nonrebreather face mask by Emergency First Responders. Which other step would be appropriate care for this patient

Place him supine on the stretcher with feet elevated

While cleaning the ambulance following an emergency call, you find a syringe and a needle that were used by the paramedic to give a patient an injection. What is the proper way to dispose of used syringes and needles

Place the syringe and needle in a special puncture-resistant container

If the placenta attaches over the cervix, the patient and fetus would be at risk for which condition

Placenta previa

An EMT is transporting a restrained patient who is trying to spit on him. Which action would the EMT be justified in doing?

Placing a surgical mask over the patient's mouth while being careful not to impede his breathing

A window washer fell 20 feet from scaffolding while washing the windows on an office building. He is alert and oriented, but states that he cannot move or feel his legs. Additionally, because his blood pressure is 72/48 mmHg, you suspect spinal shock. Which other assessment finding reinforces your suspicion of spinal shock?

Warm and dry skin

When involved with a pediatric patient, the EMT recognizes that patient assessment begins when

You first lay eyes on the patient

Which statement concerning a ventricular assist device is accurate?

You will likely feel no pulse and cannot obtain a blood pressure in patients with VADs

What is the best description of a topical hemostatic agent?

dressing that is specifically designed to stop bleeding

Which of these statements made by your EMT partner would require immediate correction when treating a patient with hypothermia and a deep freezing cold injury to his left foot, ankle, and lower leg?

"We will need to gradually rewarm the patient at a rate of 2-3° F per hour."

You are transporting a patient who was bitten on her hand by a spider approximately 20 minutes ago. On scene, the primary assessment revealed no life threats and vital signs were stable. While conducting the reassessment, which of these findings would be of most concern to the EMT

Hives to the chest and abdomen

A 21-year-old male has a stab wound to the left side of his neck. Emergency Medical Responders are holding manual spine motion restriction. As you approach the patient, you note blood on his shirt and pale-looking skin color. As team leader, which direction would you give first?

"I need someone to place a gloved hand over that stab wound."

You are caring for a crying 30-month-old male who fell 16 feet from a second-story window, landing on a patch of soft dirt and leaves. He is alert and oriented and seems to have pain to his left shoulder and arm. Which instruction would you give to the team of EMS providers caring for this patient

"Let's make sure his SpO2 is greater than 95%."

You are first on the scene of a two-car motor vehicle collision. Two patients are entrapped in one vehicle. In the other vehicle, there is a male driver complaining of chest pain. During this time, your primary concern is:

Ensuring personal safety

What is an advantage of the Lund and Browder system of BSA estimation?

It uses an age-related chart for more accurate determination

While riding a bike, a young boy fell onto a stick protruding from the ground. The stick impaled him in the neck. The boy immediately pulled the stick out and r--- ran home, where his mother called 911. Assessment reveals a gaping wound to the right neck. All hemorrhage has clotted off and manual cervical spine motion restriction is being maintained. Which instructions should you provide to the other EMTs on scene?

"Place an occlusive dressing over the wound and tape it on all four sides.

You have been called to a football practice field on a very hot day for a heat-related emergency. On scene, you find a young male patient panting and lying under a tree. His skin is flushed, and teammates are continually wiping the sweat off his face, arms, and chest with towels. Which of these statements to his teammates would be most appropriate?

"Please let him sweat. It is helping his body cool down."

An Emergency Medical Responder asks you why you should not overextend the airway on a pediatric patient when performing a head-tilt, chin-lift maneuver. Your reply would be

"The cartilage of the trachea is very soft and can 'kink' if the neck is extended too far."

An 83-year-old male fell down five stairs to the floor below. He hit his head and now complains of a headache. When getting a history from this patient, which question should be asked before all others

"What caused you to fall?"

You are by the side of a male patient who participated in deep-sea diving earlier in the day. The 32-year-old patient informs you that he thinks he pulled a muscle in his shoulder, which has been hurting more as the day goes on. When questioned, he admits to having a headache and blurred vision. You are suspicious of decompression sickness. Which statement made by the patient would be most significant given the situation?

"While diving, I saw a shark and came up as fast as I could."

On the scene of a motor vehicle collision, the driver is sitting inside the car with all windows up. When you attempt to open the doors, you find that they are locked. Given this situation, which statement made to the patient is most appropriate

"Without moving your head or neck, please try to unlock the door."

You have been dispatched to a motor vehicle collision. On scene, you find that a minivan has completely sheared off a wooden utility pole. The electrical wires are draped over the hood of the minivan and the driver, a woman in her 20s, is screaming that she is not hurt and wants to get out of the vehicle. The wires are not moving, arcing, or making any noise. Which instruction is most appropriate at this time?

"You need to remain still and calm in your seat; do not touch anything in the car."

Which actions by the EMS crew have met the platinum ten minutes criteria?

Assessed, provided spinal motion restriction precautions, and initiated transport within 10 minutes of arriving on scene

An 86-year-old male has experienced a seizure and is now lethargic and confused. His family states that the patient has a history of seizures and was sleeping in bed when the seizure was witnessed by his wife. He remains in bed with an intact airway, adequate breathing, and a rapid radial pulse. You also note him to be incontinent of urine. Appropriate care of this patient would include

Considering oxygen and transport with him positioned on his left side

A confused and lethargic 22-year-old male patient with no medical history came up from a deep-sea dive and is now complaining of shortness of breath and sharp pain, which he localizes to the right side of his chest. He denies nausea, vomiting, or dizziness. You have applied oxygen and are getting ready for transport. How will you transport this patient on the stretcher?

Lateral recumbent position

You have arrived at the scene where a tractor-trailer overturned into a small ravine adjacent to the freeway. As you exit the ambulance and are donning your gloves, the driver of the truck approaches and informs you that he was hauling radioactive medical wastes and warns you that there is a car underneath the overturned trailer. Given this situation, what should you do immediately

Move the ambulance upwind and uphill of the scene

Which action demonstrates that the EMT is correctly performing uterine massage

One hand cups the top of the uterus, with one of the hands positioned just above the symphysis pubis

The mother of a 4-year-old boy states that he fell off the top bunk bed and hit his head. After falling, he appeared to seize for approximately 30 seconds. Currently, he has nonpurposeful movement to painful stimuli, with a patent airway and slow respirations. His radial pulse is intact, and his skin is warm and dry. You note urinary incontinence. After taking manual spine motion restriction precautions, what should you do next

Perform a jaw-thrust maneuver and start positive pressure ventilation

An intoxicated teenage male with a history of diabetes fell down five stairs and is now responding to a shoulder pinch with garbled speech. Manual spine motion restriction precautions are being maintained by fire department EMRs. The primary assessment reveals an open airway, adequate breathing, and strong radial pulse. Vital signs are pulse 122 beats/min, respirations 18 breaths/min, blood pressure 108/60 mmHg, and SpO2 97% on room air. The patient's skin is warm and dry. Which action would be performed next?

Perform a secondary exam

An 85-year-old male with a history of heart attacks has called 911 for chest discomfort. He states that the pain started 2 hours ago when he was sitting on his porch. When asked, he states that it feels similar to his last heart attack, only this time he is also dizzy. The primary assessment indicates no life threats, and the Emergency Medical Responder on scene reports the following vital signs: pulse, 92 beats/min; respirations, 18 breaths/min; blood pressure, 88/52 mmHg; and SpO2, 92% on room air. The patient has nitroglycerin, but has not taken it yet (you have a standing order to administer nitroglycerin without calling medical direction). After applying oxygen, you should:

Perform the secondary exam

You have determined that you will need to perform a field delivery. Which instructions to the patient show that the EMT is properly positioning her?

Please lie on your back, draw your knees upward, and spread your legs apart."

A patient has just been log rolled and positioned onto the long backboard. Which care measure should be performed next, assuming it was not previously completed?

Secure the patient's torso with straps

Which mnemonic can be used to help remember the types of damage that can be caused by hazardous materials

TRACEM

Which statement regarding the role and function of the cervix during pregnancy is true?

The cervix becomes plugged with mucus to prevent contamination of the uterus during gestation

An alert and oriented auto mechanic had his right thigh pinned between the bumpers of two cars. Assessment findings include deformity and swelling to the right hip area and mid-thigh, along with ecchymosis and swelling to the right knee. The patient complains of excruciating pain to the right leg. Which set of instructions given to other EMTs on the scene would be appropriate

"Let's provide spine motion restriction precautions now, get the patient in the ambulance, and then provide more care to the leg en route."

On scene at a school cafeteria, you are presented with a 11-year-old boy who attempted to eat a small plum whole. He is conscious with stridorous respirations and unable to cough when instructed to do so. His skin color is gray and cyanotic. What would your immediate action be?

Deliver five abdominal thrusts

A patient has been stabbed in the right upper quadrant of the abdomen. Which solid organ should the EMT be most concerned with being injured?

Liver

A midsize car has been struck by a small delivery truck and sustained moderate damage to the front passenger area. The car is on its tires, and wood chocks have been placed under the undercarriage of the car. The air bags did not deploy. What is the next step in this rescue?

Release air from the tires

The EMT would recognize which response when a combat veteran patient states, "I still see them burning but cannot do anything about it"

Reliving

54. A patient at a mountain ski resort presents to you with notable shortness of breath. She denies any past medical history and takes no medications. Your assessment reveals crackles to the bases of both lungs. After applying oxygen, you realize that the key to helping this patient improve is to:

Remove her to a lower altitude

A motorcyclist wearing a full helmet was thrown from his motorcycle after hitting a patch of oil on the road. The bike had been traveling at a high rate of speed. Manual spine motion restriction is being maintained by an off-duty EMT. The primary assessment reveals the patient to be responsive to painful stimuli and breathing rapidly with slight sonorous airway sounds. His radial pulse is weak and fast. As you quickly scan his body, you note deformity to the left femur and lower leg. Your immediate action should be to:

Remove his helmet

You have been called by a family for their mother, who is "not acting right." On scene, you find the 69-year-old woman to be confused and seemingly slow in talking. Her airway is open and respirations are 12 breaths/min. Her pulse is 58 beats/min, and her skin is cool to the touch, despite the thermostat keeping the house at normal room temperature. The family states that she has been this way for about three days, but before then seemed to be doing well. Your partner obtains a blood pressure reading of 104/52 mm Hg. When getting a medical history, which of these statements made by the family would make sense, given the presentation of the patient

"Mom suffers from a low thyroid."

You are first to arrive at the side of a pond where a boy playing hockey has fallen through the ice. The child is holding onto the ice at the edge of the break and has a distressed look on his face. Your immediate action would be to:

Instruct the patient to remain still until the rescue team arrives

You are treating a patient with suspected heat cramps. Your medical director has issued standing orders for such treatment given the high number of heat-related emergencies to which your service routinely responds in the summer months. Regarding these standing orders, which of these instructions would seem most appropriate?

Mix one teaspoon salt in one quart of water and administer half of a glass every 15 minutes

A patient is confused and combative after playing football in the open sun on an extremely hot day. On the field, you find his airway open, respirations tachypneic but adequate, and radial pulse rapid. The patient's skin is hot and dry. After deciding whether to provide spine motion restriction precautions, your next action would be to:

Move the patient to the air-conditioned ambulance

You are called for an elderly woman who has slipped on the ice. On arrival, you find her supine on an icy sidewalk and responsive to painful stimuli with decorticate posturing (flexion). She has blood coming from a laceration on the back of her head and is breathing agonally at 4 times a minute. What should you do immediately?

Perform the manual jaw-thrust maneuver

You have been called for a 78-year-old female patient who slipped while getting out of her bathtub. She is confused and cold to the touch. You learn that the patient slipped 24 hours ago and has been on the bathroom's tile floor ever since. Since it is summer, she also had her air conditioner running on high. The EMT would best categorize this emergency as:

Urban hypothermia with heat loss caused by conduction

You have been called to a hunting camp for a patient with a severe freezing cold injury to his hand and fingers. The patient is suffering no life-threatening conditions and you elect to rewarm the affected areas. Which of these warming techniques would be most appropriate for this patient?

Thaw and rewarm the tissue as quickly as possible

After ensuring that there are no life-threatening conditions to the airway, breathing, and circulation, the priority in caring for a patient with hypothermia is:

Preventing additional heat loss

During the summer months, you work as an EMT at a first aid station that helps people who hike trails in the high mountains of the western United States. One afternoon, you are presented with a 42-year-old male patient who was just at an altitude of 14,000 feet while hiking. The patient states that while at this altitude he experienced a severe headache, became very uncoordinated and stumbled often, and felt very nauseous. He became scared, so he left the trail to be

High-altitude cerebral edema

A person is in a hot environment and his body is successfully compensating to off-load excess heat. Which assessment findings best illustrate this process

Warm and diaphoretic skin, elevated heart rate, and increased respirations

You are treating a male patient who is hypothermic. What should you do to decrease heat loss caused by radiation

Wrap a towel around the patient's head

You have been chosen by the medical director to give a 15-minute presentation on cold-related emergencies to a local volunteer fire department. In preparing the talk, which teaching point should you emphasize?

Emergency responders play a critical role in caring for hypothermic patients since the most important phase of care occurs during the first 30 minutes

Medical direction has ordered you to begin rewarming an unresponsive patient with a core temperature of 93° F. Which of these measures would most benefit this patient

Wrap the patient in several warm blankets

You are called to a construction site on a hot and humid day for a male patient with a heat emergency. On scene, you find a 49-year-old man who responsive to verbal stimuli. Coworkers state that the patient had been moving concrete blocks all day in the hot weather and had not rested, eaten, or had anything to drink in the past two hours. During the primary assessment, you note the patient's airway to be open and breathing rapid but adequate. His radial pulse is rapid and bounding, and his skin is hot and dry. Which of these findings makes this patient a priority for immediate and rapid transport?

Hot and dry skin

You are by the side of a young male patient who hit his head after diving from a hillside into a shallow lake. The patient is unresponsive and has a large hematoma to his forehead with abrasions all over his face. A paramedic asks you to apply painful stimuli to his arms and legs. You recognize that the paramedic wants you to assess for which type of injury or condition

Spinal injury

A young man has been struck by lightning on a golf course and is in cardiac arrest. Other golfers say that he was struck 8-1 O minutes ago. What should the EMT do immediately

Start cardiopulmonary resuscitation

Which classifications of WMD are combined in the same category when using the mnemonic B-NICE?

Radiological and nuclear

Regarding snakebites, which of these statements is true?

The majority of snakebites involve nonpoisonous snakes

You have arrived on the scene of a large structure fire. Emergency Medical Responders (EMRs) are by the side of a patient who is responsive to verbal stimuli and has significant burns to his body. They report that the patient fell down a flight of stairs while trying to escape and was trapped for several minutes. Their assessment findings include partial- and full-thickness burns to his right arm, right leg, and right side of the face. The EMRs also report that the patient's unburned skin is cool, clammy, and pale, with a delayed capillary refill. His pulse is 144 beats/min, respirations are 22 breaths/min, and blood pressure is 88/50 mmHg. Which instructions would be most appropriate?

"After reassessing the ABCs, let's do a secondary assessment and look for signs of injuries that are causing shock."

You arrive at a hunting camp for a hunter who stumbled over a rock and caught himself on his outstretched arm. The patient denies hitting his head or neck. There are no threats to the airway, breathing, or circulation. The patient has swelling and ecchymosis to his left wrist. A radial pulse is readily palpated and the skin in the hand is warm to the touch. Which instructions given by the EMT to other rescuers indicates proper care of this patient?

"After we splint his wrist, we will need to elevate it during transport to decrease the opportunity for swelling

You have arrived on the scene of an industrial plant where a female worker had a chemical splashed into her eye. She now complains of severe pain in the eye and is flushing it with tap water over a sink. Which question is it most important that the EMT ask first?.

"Are you wearing contact lenses?"

A woman informs you that she is eight months pregnant and fatigues easily. She is apprehensive because when she lies down, she gets dizzy and feels as though she is going to vomit. What is the EMT's best response?

"Check with your obstetrician, but you may want to rest and sleep on your left or right side."

86. What would be the most appropriate instructions to give a new EMT with whom you are working regarding the ventilation of an unresponsive and apneic 3-year-old boy?

"Deliver one breath every 3 seconds."

A young female was thrown from a horse and is now confused. Assessment findings include an open airway, adequate breathing, and a strong radial pulse. Her vital signs are normal. Which question would be the most important to ask the patient's mother, who was riding with the patient, as soon as possible

"Did she lose consciousness?"

You are responding to a home for injuries resulting from a domestic altercation. In preparation for patient contact, what would you tell your partner

"Do not let the patient get between you and the door."

A 31-year-old unrestrained pregnant female was involved in a motor vehicle collision in which she struck the steering wheel. She now complains of chest and abdominal pain. She is conscious, alert, and oriented with no deficits noted to the aiiway, breathing, or circulation. When asking her questions regarding her status, which questions should you ask first?

"Do you feel as though you are having contractions?"

You are assessing an 86-year-old male who fell down a flight of stairs after getting out of bed to use the bathroom. He complains of severe head, neck, and .back pain. Which question is most appropriate to ask this patient

"Do you know what caused you to fall?"

You are part of your EMS system's QA/QI program. While reading PCRs, you come across one that pertained to a psychiatric emergency in which the patient was restrained. As you review the PCR, which statement would cause you to suspect that the EMS crew needs remedial education?

"Following extremity restraint, the patient was positioned and secured prone on the cot, with his head turned to the side to allow airway monitoring."

You are assessing a patient with schizophrenia. Friends called 911 because he was experiencing hallucinations this morning. He is prescribed Thorazine and risperidone for his illness. Which question is most pertinent to the current situation and should be asked first?

"Have you been taking your medications as prescribed?"

A male patient with a history of behavioral problems continually smacks his lips, sticks his tongue out, and turns his head to the left. His arm also jerks upward regularly. Which statement, made by a caregiver, would an EMT recognize as most important and relating to this presentation?

"He has been on psych medications for many years.•

You suspect a very sick child to be in shock. His skin is dry, radial pulse weak, and blood pressure low. Which statement made by the parents would reinforce this suspicion?

"He has had diarrhea for three days.

A patient is lying under a tree after being involved in a motor vehicle collision. She states that immediately after the crash, she felt okay, but now her legs are numb and tingling. She also has lower back discomfort. Which question is it most important for the EMT to ask next

"How did you get out of the car?"

Friends pulled an unresponsive 15-year-old male from a lake. They were swimming when they lost track of him. Assessment reveals labored breathing and a weak pulse. Which question would give the EMT the most information related to the prognosis of the patient?

"How long do you think he under the surface of the water?"

Which statement made by the EMT shows an understanding of dressing and bandaging a wound?

"I always leave the tips of the fingers or toes exposed when bandaging the arm or leg so future assessment of circulation is possible."

You are assessing and treating a young girl who superficially cut her wrists in a self-stated suicide attempt. Bleeding was not significant and has clotted. While transporting the patient, which statement from the EMT is most appropriate

"I am going to clean these cuts up and then wrap them.

You have been called for a 16-year-old female with vaginal bleeding. On scene, you determine the patient to be pregnant and bleeding for 3 hours. Which statement would indicate proper care of the patient

"I am going to put this sanitary napkin between your legs to collect the blood."

You are concerned that a sick 5-year-old patient with lower abdominal pain and a temperature of 100.3° F has appendicitis. He describes the pain as generalized and crampy. Although he complains of nausea, he is not vomiting. His pulse, respirations, blood pressure, and SpO2 are normal, and there are no immediate life-threatening conditions. Based on these assessment findings, his father informs you that he does not wish his son to be transported, and will call the pediatrician in the morning. How would you respond

"I have to tell you, if he has appendicitis, it will just get worse and can become a major medical emergency for him."

You are assessing a 5-year-old boy who complains of arm pain after falling down three steps. As you physically assess the patient, he tries to bite your hands. His mother is at his side. Which statement by you is appropriate to make regarding the behavior of biting

"I know that you hurt, but biting is not okay."

A female who is 32 weeks pregnant was involved in a motor vehicle collision. She is currently secured to the long spine board, with other spinal motion restriction precautions taken. Which patient statement should concern the EMT most

"I think that I just had a contraction."

17. Which statement made by your new EMT partner best describes the way to determine the severity of a patient's blood loss?

"I use the patient's signs and symptoms to judge the severity of the blood loss."

You are preparing a 24-year-old male for transport to the hospital after he stated that he wanted to kill himself. The patient's father pulls you aside and tells you that his son repeatedly has been in the hospital for suicidal statements but is then released and comes home and the whole cycle repeats itself. He is worried that one day his son will actually commit suicide and wonders if there is anything he can do to decrease the chance of this happening. In regard to the most common means by which people commit suicide, you should respond by saying:

"I would suggest gathering up all guns and weapons and removing them from the house.

Which statement about compartment syndrome is true

"It describes swelling in the tissues that can result in the loss of an extremity."

You have been asked to speak to a group of new mothers regarding the emergency treatment of a fever. During your presentation, one of the mothers asks you which temperature would cause a child to seize. Your response would be

"It is not the specific temperature that is of concern. but the rate at which the body temperature rises

The wife of a military veteran with PTSD states that her husband developed type 2 diabetes 1 O years after returning from combat. There is no history of this disease in either side of the family, and the patient is not overweight and does not have any other risk factors for the disease. Consequently, the wife believes the diabetes results from her husband's experience in the military. How would you best respond to her assertion?

"It is not uncommon for combat veterans to unexpectedly develop such conditions like this.''

A 48-year-old female patient with a history of depression has called for help because she feels suicidal. During transport, the patient refuses to talk to you despite repeated attempts to obtain her past medical history and learn about the events leading up to her call for help. Which statement is most appropriate at this time?

"It's okay if you do not want to talk, I will be right here if you change your mind.

Which of the instructions from one EMT to another shows the correct application of a cervical collar

"Keep his head in neutral position while I apply the cervical collar

You pull up to the scene of a single-car motor vehicle collision. Emergency Medical Responders (EMRs) are maintaining inline manual cervical spine motion restriction of the 56-year-old driver, who was unrestrained when she struck a tree at a high rate of speed. As you approach the vehicle, you note that the patient appears unresponsive, with blood coming from her nose and ears. Which instruction to the EMRs is most appropriate

"Let me conduct the primary assessment and place a cervical collar on the patient, and then we can start to get her out of this car.

Which instruction would you provide to your team immediately after securing a patient with head, neck, and back pain to the long backboard with appropriate spine motion restriction equipment?

"Let's check for extremity PMS before we move the patient to the stretcher

A 2-year-old boy fell down a flight of stairs, is crying loudly, and is very difficult to communicate with. Because you are unable to clear the spine, you elect to initiate spine motion restriction precautions. Which instruction would you provide to other EMTs who are initiating this care for the patient?

"Let's place a folded towel under his shoulders and back to help maintain head alignment."

A 28-year-old male was cutting limbs from a tree when he lost his footing and fell 20 feet. He is unresponsive and breathing very poorly, at 28 times per minute. His radial pulse is weak and thready, and his skin is cool to the touch. Emergency Medical Responders (EMRs) have placed the patient on a nonrebreather face mask and are holding manual cervical spine motion restriction. A blanket has also been applied and covers the man. Based on these assessment findings, which instruction would you give to the EMRs?

"Let's take off the oxygen mask and start positive pressure ventilation to assist his breathing

Which statement about traction splints is true

"Mechanical traction should be applied until the mechanical traction is equal to the manual traction and the patient experiences a reduction in pain.

Which statement made by an EMT shows that he understands the major cause of death in the prehospital setting for a burn patient

"Most burn patients who die before getting to the hospital die of airway or bleeding problems, not the burn itself."

An unrestrained driver of a car that has struck a tree at 45 mph has suffered a contusion to his heart. The EMT would recognize that this injury occurred during which impact of the collision?

Organ collision

At a pool party, a 48-year-old female was pulled from the water after being submerged for 1 O seconds according to bystanders. The patient responds to verbal stimuli by moaning and has an intact airway with adequate respirations. Her heart rate is 116 beats/min and SpO2 is 97%. Which statement made by friends would be considered most important at this time?

"She takes insulin for her sugar condition."

An 89-year-old female is short of breath and has a fever. Her SpO2 is low, and she has crackles and coarse breath sounds to both lungs. When getting the patient's history from her family caregivers, which statement would make you suspicious that the patient has aspirated something

"She was fine until just after eating lunch."

A female who had hot coffee thrown at her has a superficial burn to the right side of her face. After transferring the patient to the hospital emergency department for continued care, a new EMT asks you if the patient will have a permanent scar on her face. Which is your best reply?

"Since only the outer layer of skin was burned, there is little chance of scarring."

After taking a patient to the hospital for an unsuccessful suicide attempt, your partner states that this is the fifth suicide call he has had this month. Four of them have been women who were unsuccessful; the fifth was a man who shot himself in the head and was dead on the scene. How should you respond to your partner?

"That seems to make sense, given that more women than men attempt suicide, but men are generally more successful."

Which statement made by an EMT indicates an understanding of facial anatomy and/or injuries

"The only movable bone making up the face is the mandible."

You are on scene with a patient who has a freezing cold injury. The patient stated that he had no sensation in his hand or fingers prior to the initiation of a rewarming process. You have subsequently begun rewarming his hand and fingers. Which of these statements made by the patient following the rewarming process is of greatest concern?

"There is no pain when I rub my fingers with my other hand."

The EMT shows she has a proper understanding of Braxton-Hicks contractions when he says:

"They are often irregular and less painful than labor contractions, but they should still be evaluated by the hospital

The EMT would recognize potential geriatric neglect when the patient makes which statement?

"They do not give me my high blood pressure medications daily as I think they are supposed to

A patient has suffered a superficial burn to the chest, arms, and face after pouring gasoline on a smoldering campfire and accidentally inhaling the plume of smoke and fire that erupted suddenly. He is now coughing. Which statement indicates that the EMT is properly caring for him?

"Throughout transport, I am going to listen to your lungs quite frequently."

A 43-year-old male patient has removed a tick from his arm prior to your arrival. He is refusing all care at this time. Which of these statements made by the EMT would be most appropriate?

"Tick bites can result in bacterial infections, which can be very serious.

At the scene of an MVC, you must remove an injured child from his car seat and provide full spine motion restriction precautions prior to transport to the hospital. Which statement made by the lead EMT indicates that this procedure is being performed correctly

"To get her on the long board, let's tilt the car seat backward, and carefully slide her out head-first

You are assessing a 12-year-old female in respiratory distress and with a history of asthma. Which statement, made by her mother, would be most concerning

"Two years ago she needed to have a breathing tube inserted into her throat.

Which statement demonstrates an understanding of shock caused by internal bleeding?

"You should always suspect internal bleeding in a patient with the signs and symptoms of shock but no external bleeding."

In most EMS systems, a multiple-casualty incident is said to occur when the number of patients is

3 or more

A coworker is telling you about a recent EMS call in which he provided care to a preschooler. Based on this description, you know that age of the patient would have been:

3-5 years

Which patient should the EMT be most suspicious of having a spontaneous abortion?

A 21-year-old woman, 6 weeks pregnant, passing vaginal blood clots

The EMT would recognize which patient as suffering from multisystem trauma?

A 43-year-old male with extremity trauma and shock from a motor vehicle collision

The National Incident Management System (NIMS) provides for:

A consistent approach to managing a disaster by multiple responding departments and agencies

Based on the structure of the nervous system, which statement is true?

A deep laceration to the arm can sever peripheral nerves

Regarding the field triage of injured patients, which of these patients is NOT a member of a special patient population that would necessitate transport to a trauma center?

A female patient who is 12 weeks pregnant and who fell on the sidewalk, injuring her hip and wrist

Where is at EMT at highest risk for exposure to phosphine gas

A methamphetamine lab

Your partner informs you that the 3-year-old boy you have been called to care for is breathing 26 limes per minute. As a knowledgeable EMT, you would recognize this to be

A normal rate

At an elderly extended care facility, a ventilator-dependent patient is going to be transported to the emergency department for a suspected urinary tract infection. The ventilator settings are rate, 10; tidal volume, 425; and oxygen concentration, 50%. Given this information, which statement is true

A volume of 4250 ml is delivered every minute

A patient presents with an arrow impaled in the right lower quadrant of his abdomen. He is in excruciating pain and states he was accidentally shot by a ,friend while hunting. While there is no external bleeding coming from the injury, the surrounding skin is ecchymotic other assessment findings such as, the vital signs, indicate the patient is in early shock. Which action should the EMT perform first?

A. Administer supplemental oxygen

A patient states that she cannot leave the house to go to the hospital "because I am scared I will do something idiotic and lose it and then everyone will look at me and laugh." She states she rarely leaves the house and the last time she did, she began to hyperventilate. This caused her to return home immediately. Based on this information, the EMT should recognize the patient as having

Agoraphobia

A 15-year-old female was struck in the mouth with a baseball bat. She has lost her front teeth and is spitting blood. The EMT's priority concern when caring for this patient would be:

Airway compromise

An infant is short of breath and has rhonchi in both lungs. He is alert with adequate respirations at a rate of 38 breaths per minute. His skin color is pink but cool to the touch. Due to the patient's movement, it is difficult to get a reliable pulse oximeter reading. Additionally, when EMRs place a pediatric mask on his face, he becomes very upset and physically struggles to remove it. In this situation you would:

Allow the mother to hold the infant and provide blow-by oxygen

When assessing a 32-year-old male with a history of developmental challenges, you note the protrusion of a clamped tube from his back in the area of his right kidney. The best method of determining what the tube is and why it is in place would be to

Ask family members

A young female was involved in an altercation and stabbed in the abdomen with an ice pick. After being stabbed, she quickly removed the ice pick. She states that there was not much bleeding but does say it "stings" quite a bit. Assessment reveals a small puncture wound to the lower right quadrant with some dried blood around the site. Which best describes the primary focus of the EMT in caring for this patient?

Assessing for internal blood loss or shock

You are on the scene of a motor vehicle collision and must look for an area to land a medical helicopter. Which option would be the best choice

An open field that is level and preferably 150 feet from the accident

You are teaching an EMT class about the role of the EMS response to a terror attack involving weapons of mass destruction (WMD). A student asks you what constitutes a WMD. How would define a WMD

Any agent used to cause indiscriminate death and destruction

19. Which statement is true regarding care of the multisystem trauma patient?

Any threat to life should be managed as it is identified

You are called for an alert and oriented female who had a cesarean section several days ago. Today, the patient bent over and tore the surgical incision open. There is a moderate amount of bleeding coming from the site, but otherwise the patient is stable. What should the EMT do to control the bleeding?

Apply a multitrauma dressing over the incision

A 37-year-old woman, who is four months pregnant, complains of abdominal and pelvic pain after being assaulted in her home by a live-in boyfriend. During the assault, he struck her in the head, chest, and abdomen with his fists and a chair as she lay on the floor. There are no obvious threats to the airway, breathing, or circulation. Her vital signs are pulse, 124 beats/min; respiration, 22 breaths/min; blood pressure, 114/68 mmHg; and SpO2, 98% on room air. Which intervention is needed for this patient

Apply a nonrebreather mask with high-concentration oxygen

The EMT recognizes that the division between the thoracic and abdominal cavities lies approximately:

At the fifth intercostal space

You are caring for an 8-month-old boy who is unresponsive after choking on a piece of hard candy. As you look into his mouth, you can see the candy in the back of his oropharynx. What would your next action be

Attempt to retrieve and remove the candy

While delivering a baby, you note that the umbilical cord is wrapped around the baby's neck. You would:

Attempt to slip the cord over the baby's shoulders or head

A 40-year-old male fell 20 feet from a ledge while hiking. The park ranger is now maintaining manual spine motion restriction for the patient. The patient is combative and confused, but his airway is open and his breathing is adequate. The patient has a radial pulse of 112 beats/min, blood pressure of 96/76 mmHg, and SpO2 of 91 % on room air. What should the EMT do first?

Begin administering supplemental oxygen

A 24-year-old female was an unrestrained driver of a car that struck a guardrail, causing her to sustain blunt trauma to the chest and abdomen. Vital signs obtained by Emergency Medical Responders are pulse 120 beats/min, respirations 22 breaths/min, blood pressure 100/78 mmHg, and SpO2 96%. As you transport her to the hospital, which finding indicates that the patient is deteriorating?

Blood pressure 86/50 mm Hg

You are transporting a male patient with a deep stab wound to the groin, with damage also noted to the genitalia following an assault by his girlfriend. The patient is alert and oriented, albeit restless. Vital signs on scene were pulse, 128 beats/min; respirations, 20 breaths/min; blood pressure, 108/64 mmHg; and SpO2, 92%. Oxygen is being administered via nasal cannula. Upon reassessment, which sign would be most concerning to the EMT?

Blood pressure is now 106/88 mmHg

When assessing a patient with a head injury, which finding is most indicative of increasing pressure within the skull

Blood pressure of 192/106 mm Hg

A skilled nursing facility has called you for a patient who has a problem with his intraventricular shunt. The patient is a 21-year-old man who was born with hydrocephalus and had the shunt placed shortly after birth. When assessing the patient, which sign would raise your suspicion that the shunt is occluded?

Blood pressure of 260/126 mm Hg

A 2-year-old patient is awake and in respiratory distress with stridorous respirations. His airway is patent and pulses rapid. Vital signs are pulse, 144 beats/min; respirations, 28 breaths/min; SpO2, 93% on room air; and temperature, 101 °F. The patient has no medical history, although diabetes runs in the family. Appropriate care for this patient would include:

Blow-by oxygen that is humidified

A middle-aged male died immediately after a self-inflicted gunshot wound to the head. Which portion of the central nervous system was most likely damaged to cause such a rapid death?

Brainstem

While en route to a domestic violence incident with possible injuries, the police contact you and report that a male patient has arterial bleeding from a stab wound to the arm. Given this information, which bleeding characteristics do you expect?

Bright red blood that is spurting with each beat of the heart

You have been called to the local community hospital for a 48-year-old male who was involved in a motor vehicle collision 2 hours prior. He has been diagnosed with a cerebral contusion and must be transported to a trauma center for specialty care. As a knowledgeable EMT, you recognize that a cerebral contusion is:

Bruising and swelling of the brain tissue

You have been called for a patient who had a large bag of lime powder tear as he was unloading it from a truck. On your arrival, the patient complains of burning to both legs. You note that, aside from the shorts he is wearing, lime dust has covered both legs. Your immediate action would be to

Brush as much of the lime off his legs as possible

You are assessing a patient who was involved in a motorcycle accident. He states that he saw an oncoming car and "laid the bike down" to avoid an outright collision. He was wearing a helmet. Which injury pattern would make sense given the action of the motorcyclist?

Burn to the inside of the leg from the motorcycle exhaust pipe

The type of injury most commonly associated with an incendiary device is

Burns

13. A 27-year-old male fell down a full flight of stairs while escaping his burning apartment. Which assessment finding in this patient should demand your immediate attention?

Burns on the face

You are working the scene of a collision involving a tractor-trailer that is hauling an unknown chemical. After it has been determined that no material has been spilled, the rescue chief asks you to get the truck's shipping papers. Where would these papers most likely be located?

Cab of the truck

Torrential rains have been falling over the past few days and your service area is experiencing major flooding. While returning to the disaster command post in the ambulance, you are flagged down by a group of people who inform you that a woman is clinging to a tree in the middle of a rapidly moving and swollen stream. She is screaming for help and states that she does not know how much longer she can hang on. What should you do immediately?

Call for a water rescue team and boat

A middle-aged male patient has been stabbed once in the right anterior chest. His airway is patent, respirations tachypneic. pulse weak and rapid, and skin cool and diaphoretic. Breath sounds are clear on the left and severely diminished on the right. JVD and tracheal deviation are present. His SpO2 is 93% on supplemental oxygen via BVM. The vital signs are pulse, 140 beats/min; respirations, 28 breaths/min: and blood pressure, 100/78 mmHg. Your next action should be to:

Call for an ALS intercept en route to the ED and lift the dressing to see if air escapes during exhalation

You have been called to a residence for a 21-year-old female in labor. On arrival, you immediately see that the head of the baby is out of the vagina. While delivering the head, the mother tells you that she is having twins. The EMT should immediately

Call for another EMS crew

When getting a medical history from a patient, you refer to him as Mr. Wrask. He stops you and asks you to call him Derek. You should:

Call him Derek during the rest of your interactions

An unrestrained female driver hit a utility pole with her vehicle at a moderate rate of speed. During the impact, she struck the steering wheel with her chest The patient's airway is open, and she states that it is painful to breathe. Her pulse is moderate in strength, irregular, and tachycardic. Breath sounds are equal bilaterally, and no jugular venous distention is noted. Assessment of her chest reveals bruising and instability to the sternum. When asked, she denies any past medical history. Based on this mechanism and the assessment findings, the EMT should be suspicious of which condition

Cardiac contusion

Of the actions described, which indicates proper care of a patient who has experienced a partial-thickness burn to the hand

Carefully remove rings prior to applying a dry sterile dressing

A pediatric patient has an intraventricular shunt, according to the parents. Based on this information, you recognize that there is a problem with which system in the body?

Central nervous

While performing the primary assessment on a patient with an isolated spinal cord injury, you note that he is in severe respiratory distress and struggling to breathe. Where should you suspect the spinal cord injury has occurred?

Cervical spine

A 66-year-old female patient has been struck by a car. Your assessment reveals gurgling respirations, rapid breathing, and cool, diaphoretic skin. You also observe bruising to the chest and abdomen. What should you do most immediately?

Suction the airway

A patient has a burn that completely encircles his right forearm and hand. After performing the primary assessment, the EMT should:

Check for motor status, sensory status, and a pulse in the right upper extremity

You have been called to an assisted living facility for a 43-year-old male patient who has a history of anxiety and schizophrenia and who, according to staff, is more confused today. The staff also states that the patient has refused to take his medications for the past three days. Your assessment reveals him to have no primary assessment life threats, but his speech is slurred and he does not make sense when he talks. As you review the paperwork provided to you by staff, you note that the patient has a history of thyroid problems, diabetes, and hypertension. His pulse is 124 beats/min, respirations 16 breaths/min, and blood pressure 154n0 mmHg. At this point, which action is most important?

Check his blood glucose level

While assessing a patient on a home mechanical ventilator, the low pressure alarm sounds. What would your immediate action be?

Check the connections in the ventilator tubing and between the tubing and the tracheostomy for leaks

You are in the process of performing a field delivery and have just delivered the baby's head. What should you do next

Check the neck for the umbilical cord

Firefighters are extricating the driver of a small car that was struck broadside by a delivery truck at a moderate rate of speed. The patient was unrestrained, and the car sustained severe damage to the driver's side. As the male patient is pulled from the car, you note severe deformity to his left thigh area where an open femur fracture has been bleeding severely. Your primary assessment shows the patient to be responsive to painful stimuli, with an open airway and adequate breathing at 20 breaths per minute. His radial pulse is present but rapid at a rate of 124. After applying oxygen and stopping the leg bleed with direct pressure, the EMT should immediately

Conduct a rapid secondary assessment

You are assessing a patient who has sustained blunt trauma to the head. Which sign would be most indicative of the loss of cerebrospinal fluid?

Clear fluid coming from the nose

You suspect that an unhelmeted male patient who was thrown from a motorcycle may have a basilar skull fracture. As you perform the secondary assessment, which finding would reinforce this suspicion?

Clear fluid coming from the right ear and the left nostril

You are at a scene where a radioactive material is emitting alpha rays. As a knowledgeable EMT, you recognize that the proper shielding from these rays can be achieved by:

Clothing

You have been called to a residence for a patient who is sick and not feeling well. The patient has multiple medical problems and is confined to bed, with care provided by family members. The patient has a gastric tube and an indwelling urinary catheter. Which assessment finding should raise your suspicion that the patient has a urinary tract infection?

Cloudy urine in the bag of the urinary catheter

A baseball player was struck in the side of the face by a baseball. Assessment findings include swelling and numbness to the left side of the face and left eyelid, as well as crepitus in the same area. Assessment of the eye reveals no obvious injury. The patient denies any problem with his eyesight, but is having difficulty visually tracking your finger upward. What would be appropriate care for this injury?

Cold packs to the face and eyelid

You arrive at the side of a pregnant patient in her third trimester who has had vaginal bleeding for several hours. Observation reveals several blood-soaked towels on the bed. She is alert and oriented with a patent airway and adequate breathing. Her pulse is 88 beats per minute and her blood pressure is 104/66 mmHg. In caring for this patient you would

Collect all towels with blood and transport them with the patient

Assessment of an alert and oriented 9-year-old child with a history of asthma reveals him to be breathing 20 times per minute with adequate chest rise and fall. You also note that he exhibits nasal flaring and has slight retractions of the intercostal muscles. His heart rate is 100 beats per minute and his blood pressure is 102/64 mmHg. On room air, he has a pulse oximeter reading of 98%. The EMT would recognize:

Compensated respiratory distress

You have been summoned for a patient with diabetes who exhibits altered mental status. Which sign would cause you to suspect that the patient has hyperosmolar hyperglycemic syndrome (HHS)

Complaint of thirst and dry mouth

A 7-year-old child was riding his bike downhill and struck a parked car. He was thrown from the bike, impacting his head and back on the roadway Presently, the patient is alert, oriented, and complaining of a severe headache. His airway is open and his breathing is adequate. A radial pulse is rapid and easily palpated, and his skin is cool and moist. After taking manual spine motion restriction of the head and neck, the EMT should:

Consider the need for supplemental oxygen

An alert and oriented 18-year-old female patient has threatened to hurt herself, but is refusing transport to the hospital. She is calm and states that she knows her rights and taking her would constitute false imprisonment. Her family is present and wants her to go to the hospital for help, as she has attempted suicide in the past. Given the situation and the patient's threat, what is your best course of action?

Contact law enforcement for assistance

A family has called you after the apnea alarm for their 1-month-old baby sounded. They state that the settings seem too sensitive and the alarm goes off frequently. They would like the EMS squad to readjust the settings, so that this scenario can be avoided in the future. They do not want the patient transported to the hospital and are prepared to sign a refusal-of-care form. Assessment reveals no life threats to the patient's airway, breathing, or circulation. Given the situation, you would

Contact medical direction

The goal of an attack using a "dirty bomb" most likely will be

Contamination of an area with radioactivity

At a drill involving the rollover of a tourist bus, you have been assigned to the transport area. Which activities will you be performing within this section

Coordinate the transfer of patients to ambulances for transport to the hospital

You have been called for a male patient who was shot with a rifle. Assessment reveals an entrance wound to the left lower quadrant of the abdomen. A hemorrhaging exit wound to the left lower back is also located. What should you do immediately

Cover the entrance wound with a gloved hand

During an altercation, an intoxicated male was cut with a sharp knife. Assessment reveals a laceration, oozing dark red blood, across the right eyelid down to the right cheek. It also appears that the patient's eye was cut with the knife, a suspicion reinforced by the patient's complaint of eye pain and collection of blood in the sclera. The EMT would:

Cover the eye with a sterile dressing soaked in saline

A patient was struck in the right upper arm with a baseball bat. Which sign or symptom indicates the highest probability that the humerus has been fractured

Crepitus felt on palpation

The "EMS Management of Patients with Potential Spinal Injury" document published by the American College of Emergency Physicians in 2015 discusses which principle regarding the immobilization of patients?

Current out-of-hospital management of potential spinal injury lacks evidentiary scientific support

patient who was involved in a motor vehicle collision is complaining of neck pain. Which piece of information would most contribute to the EMT's high index of suspicion that the patient may have suffered more significant injuries beyond isolated neck pain?

Death of the car's driver

A 20-year-old waitress spilled hot coffee and burned her right forearm. When assessing the burn, you note a full-thickness burn surrounded by partial-thickness burns. The patient complains of pain in the surrounding partial-thickness burn, but not in the center where the full-thickness burn is located. As an EMT, you recognize that the patient does not localize pain to the center of the burn because of:

Destruction of pain receptors located in the dermis

You have been called for a 42-year-old male who complains of back pain. At the scene, the man tells you that he was lying on the floor when his 5-year-old son unexpectedly jumped feet first onto his back. He gives a medical history of high blood pressure and asthma, for which he takes medications. As you assess the patient, which finding provides the greatest indication that the patient has sustained an abdominal injury

Discoloration around the umbilicus

Upon your arrival on at a residential address for a patient with a previous traumatic brain injury, you are taken to the patient, who is ventilator dependent. The patient presents as unresponsive, cyanotic, and listless, and has a weak carotid pulse. Ventilator alarms are sounding. What is your immediate action?

Disconnect the ventilator and start positive pressure ventilation

Which injury is possible based on the anatomy of the spine?

Disk injury between thoracic vertebrae 11 and 12

Sheltering of patients from radiation can be accomplished with

Distance

Your assessment of a patient who was involved in a motor vehicle collision reveals a deformity to the left side of the head underneath the hair. On further inspection, you note that the skin overlying the deformity is still intact. Based on these assessment findings, what should be the EMT's greatest concern?

Possible brain injury

A chemical agent used by terrorists is described as having a high volatility. You should recognize this descriptor as meaning that the chemical is

Easily evaporated

When assessing a patient, you note a bruise to his chest. On the prehospital care report, this injury is properly documented as

Ecchymosis

A patient is struck by lightning while running from her car to her house. What would be the primary burn cause?

Electrical

The family of an 87-year-old female has called 911. The patient is confused and exhibiting improper and out-of-character behavior. The family informs you hat the patient has diabetes, but takes pills and not insulin. They also state that she has been complaining of dizziness and has been drinking water nonstop as well as urinating. Assessment reveals adequate breathing, skin that is cool, and a radial pulse that is rapid and weak. The glucometer is malfunctioning and not providing a glucose reading. Based on her history, you should be suspicious of and treat the patient for:

Elevated blood sugar

In an EMS system, the first assessment of the call and the patient's condition is most often made by the

Emergency Medical Dispatcher

A large bus with 52 passengers overturned on a busy highway. At the scene, 12 people are dead and 23 others were taken to various hospitals, many with life-threatening injuries. As commander of the incident, which action is most appropriate for you to take after the last patient has left the scene

Encourage rescuers to talk among themselves about any feelings or thoughts they may need to share

You have been called to a geriatric extended care facility for a 91-year-old female with pneumonia. The patient is to be transported to a local hospital for further treatment. When asked, the staff confirms and then produces the patient's Do Not Resuscitate (DNR) order. The patient's airway is patent, respirations labored, and skin warm, with an intact radial pulse. The vital signs are pulse, 124 beats/min; respirations, 20 breaths/min; blood pressure, 104/58 mmHg; SpO2, 91 %; and temperature, 101.2° F. The patient also complains of chest discomfort and has a cough. Given this situation, the focus of your care should be

Ensuring adequate oxygenation

You have just arrived by the side of an elderly patient who accidentally overdosed with his daily medications after becoming confused about what to take and when. Given this scenario, what would be a priority in caring for this patient

Evaluate and support the patient's airway and breathing

A patient has suffered an open crush injury to his elbow. As you approach him, you note the patient is sitting upright and holding a towel to his elbow. Although the towel is soaked with blood, the elbow does not appear to be actively bleeding. Your first action in caring for this patient should be to:

Evaluate the patient's airway

A triage officer at the scene of a suicide bombing assigns you to a patient who has been injured from the secondary effects of the blast. Which type of injuries would you expect in this patient

Facial laceration caused by flying glass and debris

On scene, you encounter a restless patient who has been stabbed in the right upper quadrant of the abdomen and now complains of abdominal pain and a dry mouth. Assessment reveals no immediate life threats to the airway or breathing. The radial pulse is rapid and weak, and the skin cool and diaphoretic. Vital signs are pulse, 124 beats/min; respirations, 20 breaths/min; blood pressure, 122/88 mmHg; and SpO2, 90% on room air. The injury is isolated, with no involvement of the head, neck, or back. In managing the scene and the patient, which observation would cause you to immediately intervene

Family giving the patient water to drink since he is thirsty

On entering an apartment for a call related to a confused elderly female patient, you note that the air conditioner is running on high and the room is extremely cold. Which other clue obtained during the scene size-up would best indicate that this patient may be suffering from urban hypothermia

Fan blowing directly onto the patient

Almost all disaster victims experience

Fear

You have been called for a 77-year-old male with chest discomfort. On scene, the patient denies ever having chest discomfort and requests to sign a refusal-of-care form. Before having the form signed, the EMT must consider that many geriatric patients may deny symptoms because of:

Fear of losing independence and dignity

Which statement about the care and treatment of a patient with a spinal injury in the prehospital setting is true

Prehospital care for the patient with a spine injury involves spine motion restriction precautions and the identification of life-threatening conditions

You have arrived on the scene of a chemical leak at an industrial park. The placard on the leaking container has a "1" in the blue diamond and a "4" in the red diamond, which means the material is

Flammable but represents a low health hazard

A patient with diagnosed osteoporosis is more likely to sustain which type of bone injury

Fracture

You are responding to a patient with a psychiatric problem. While you are en route, dispatch contacts you and states that a mental health provider is present at the scene and you will be transporting the patient to the local psychiatric hospital for treatment of her bipolar disorder. At the scene, which behavior should help confirm that the patient has bipolar disorder?

Happy, excessively cheerful, and elated behavior

A patient experiencing command hallucinations would most likely tell you:

He was told by Zeus that he must amputate his arm so he will not die

A patient is confused and anxious after falling 20 feet from a ladder onto the grass below. Which clinical finding described is most suggestive of shock

Heart rate of 112 beats per minute

Which single assessment finding is most consistent with a stable newborn?

Heart rate of 140 beats per minute

A 43-year-old male fell from a roof and cannot move or feel his arms or legs. When assessing the patient, which sign would lead the EMT to suspect the patient is developing spinal shock?

Heart rate of 62 beats/min

When treating a patient whom you believe has an ectopic pregnancy, you know that the primary threat to the patient's life is:

Hemorrhage

A diver has summoned 911 after he came to the surface and became short of breath and "itchy" all over. Additionally, he complains of a headache. Assessment reveals a patent airway, adequate breathing, clear breath sounds, and the following vital signs: pulse 92 beats/min, respirations 20 breaths/min, blood pressure 156/62 mm Hg, and SpO2 96%. Proper care of this patient should include

High-concentration oxygen through a nonrebreather face mask

You are transporting an alert and oriented male who sustained severe abdominal trauma in an assault. The primary survey indicates a patent airway with respirations of 24 breaths per minute. Breath sounds are present bilaterally. The patient has a rapid and weak radial pulse of 144 beats per minute, and his skin is pale and cool. The blood pressure is 92/64 mmHg and SpO2 is 92%. Best management of the respiratory and oxygenation parameters would include

High-concentration oxygen through a nonrebreather mask

A patient has been stabbed with a knife at the fifth intercostal space on the right side of the chest in an attempted suicide. Assessment reveals him to be short of breath and coughing. Which question is ii most important for the EMT to ask regarding the weapon?

How long was the knife blade?"

While transporting a 38-year-old female who fell from a second-story window, you note that she is becoming confused and her pulse rate is increasing. Additionally, her blood pressure has dropped and her skin is now cool and clammy. Based on these assessment findings, you would recognize:

Hypovolemic shock

For the EMT, what is the best means of protection against acquiring a disease such as smallpox prior to any sort of exposure

Immunizations

A young male patient was running through a park shortly before dawn and struck his head on a low-hanging branch of a tree. Emergency Medical Responders have already provided spine motion restriction precautions and are providing supplemental oxygen since the patient was initially unresponsive. During transport, which finding obtained during your reassessment would best indicate that the patient has experienced a concussion

Improving memory

At a scene where a vehicle was involved in a single-car accident, you find the driver walking around at the scene. As you approach, the driver states that he felt fine when he exited the vehicle but is now concerned that his legs feel like they are "going to sleep." What is your first step in treating the driver

Instruct him to hold his head in a neutral inline position

When evaluating capillary refill time in a pediatric patient, the EMT must remember that:

It is considered more reliable than in an adult

Which statement about abuse is true

It is found in all socioeconomic levels

A 43-year-old male is entrapped in a car that struck a tree at a high rate of speed. He suffered blunt trauma to the chest and has a decreased level of consciousness. In this situation, what is the best reason to transfer the patient by helicopter to the hospital

Prolonged extrication was necessary

You are transporting a male patient who was stabbed in the left upper quadrant of the abdomen. On scene, the patient was alert and oriented, with a patent airway and adequate respirations. There was minimal hemorrhage from the stab wound, which was covered with a sterile dressing. Oxygen was applied and transport was initiated. During the reassessment, which assessment finding should the EMT consider to be of the greatest concern?

Increased heart rate and unexplained restlessness

You arrive on the scene of a motor vehicle collision. Walking toward you is the unrestrained driver of the vehicle that sustained moderate front-end damage. The patient complains of some back pain, but is walking around after the crash and does not appear to be suffering from any neurologic deficits. Although he answers your questions appropriately, he keeps asking you, "What happened?" What is your initial action in caring for this patient?

Initiate spine motion restriction precautions

While providing a continuing education seminar for your EMS system about geriatric patients, which point about geriatric trauma would you make

Injuries in elderly patients are most commonly caused by falls

You have been called for an elderly female who was found by family members on the floor of her bathroom. She is unresponsive and has a bruise to the right side of her face. Her respirations are snoring and shallow. Her skin is cool and dry, and she is incontinent of urine. Her daughter tells you that the patient has a past medical history of stroke, COPD, and seizures. Which intervention would be most appropriate when managing this patient's airway

Jaw-thrust maneuver

When relocating local residents away from a disaster, you should

Keep residents in their social groupings as much as possible

A 62-year-old male was the restrained driver of a car that was hit on the driver's side. Emergency Medical Responders have extricated the patient and provided spinal motion restriction precautions, including a cervical collar, long board, and straps. When asked, the patient complains of dizziness as well as left leg and left arm pain. Assessment reveals multiple contusions and deformity to his left forearm and a laceration with minor bleeding to the left side of his face. The patient's airway is open and his breathing is labored. A rapid radial pulse is felt. Skin is cool and dry. Which assessment finding would the EMT address first?

Labored breathing

You suspect that a patient with an open pneumothorax may be developing a tension pneumothorax. Which action is critical given this situation?

Lift one side of the dressing during exhalation

A patient has been shot in the abdomen. Assessment reveals that the bullet entered the body in the right upper quadrant and exited the lower right back. Given this finding, the EMT should assume which solid organs may have been injured?

Liver, right kidney, and pancreas

You have been called for a conscious and alert 83-year-old male complaining of a fever for three days. He has a history of peripheral vascular disease. When assessing the patient's circulation, you have a difficult time feeling his radial pulse. What would your next action be?

Locate the patient's carotid pulse

The EMT recognizes the underlying pathophysiology associated with the most common type of ectopic pregnancy is a fertilized egg that is:

Lodged in a fallopian tube

A patient has been shot in the chest with a rifle. Assessment reveals a deteriorating level of consciousness and inadequate breathing. Positive pressure ventilation is being administered, and the entrance wound, which is located midclavicular at the second intercostal space on the left side of the chest, has been covered with an occlusive dressing. Your next priority action would be to:

Look for a bullet exit wound posteriorly

A patient was ejected from a car when it overturned at a high rate of speed. The patient is unresponsive. The primary assessment is complete and you are providing positive pressure ventilation with supplemental oxygen. The patient has a significant deformity to the right side of his head, and both pupils are equal but slow to react to light. The patient also has deformity to the forearms. Vital signs are pulse, 144 beats/min; respirations, 24 breaths/min and inadequate; blood pressure, 90/50 mmHg; and SpO2, 97%. Based on this presentation, the EMT should:

Look for evidence of bleeding in an area of the body other than the brain

A 57-year-old male fell 20 feet while setting up holiday lights on his roof. He impacted the ground feet first and has suffered open fractures to both tibias, with both bones protruding through the skin. Your assessment reveals him to be responsive to painful stimuli. His airway is open, his breathing is rapid, and his radial pulse is weak. The skin is cool and diaphoretic. Vital signs are pulse 132 beats/min, respirations 24 breaths/min, and blood pressure 106/90 mmHg. According to family, the patient has no pertinent medical history. Your partner is providing positive pressure ventilation. At this point in the patient's care, it is a priority for you to:

Look for other injuries

Prior to your arrival on the scene of a motorcycle crash, an Emergency Medical Responder contacts you via radio and reports that there is one patient who has an avulsion to her left arm. As a knowledgeable EMT, which kind of injury should you anticipate?

Loose flap of skin torn on the patient's left arm

A patient was ejected from a car during a rollover collision at a moderate to high rate of speed. Which presentation in this patient is most characteristic of the incomplete spinal cord injury referred to as a "central cord syndrome"?

Loss of motor function to the arms with intact motor function to the legs

Which statement best describes the technique to be used when hyperventilating a patient with a head injury who shows signs of brain herniation

Provide 1 ventilation every 3 seconds

You have been asked to deliver a talk about EMS and its interaction with the geriatric population. Which point would you emphasize in your presentation?

Many elderly patients have a combination of different diseases in various stages

A frantic mother has contacted EMS because she believes that her 4-year-old daughter got into her blood pressure medications and ate three or four pills. The patient presents as awake and oriented, with pale skin that is cool and diaphoretic. Her pulse is rapid and respirations adequate. Vital signs are pulse, 116 beats/min; respirations, 22 breaths/min; blood pressure, 82/44 mmHg; and SpO2, 99%. After completing the primary assessment, what would your immediate action be

Obtain the name and dosage of the medication, and then call the poison control center

A patient who fell out of a tree has an open skull injury. What would be an appropriate way to describe and document this injury on the PCR?

Obvious deformity and instability to the right side of the skull with a laceration to the overlying scalp

An alert and oriented young male fell 5 feet from a stage, impacting a metal railing with the right side of his chest, just under his armpit. The primary assessment is negative for life threats, although he does complain of very painful breathing and has remarkable tenderness and crepitus over the fifth rib laterally in this area. Throughout care, which action is essential to perform

Monitor breath sounds

Multiple EMS agencies are involved in a WMD incident involving hundreds of patients. Which statement regarding the different protocols and medical direction used by those different agencies is most accurate

Most protocols involving on-line medical direction should convert to standing orders during the event

Which of the statements concerning trauma is most accurate?

Multisystem trauma has a higher mortality rate than single-system injuries

On scene, you are presented with a nonverbal 21-year-old female with multiple disfigurements from birth defects. According to her home health nurse, the patient has a fever, which requires transport to the emergency department. Your partner is familiar with the patient because he transported her last week for the same condition. When obtaining the patient's past medical history, which source should be tried first?

Nurse

Yours is the third ambulance to arrive at the scene of a nighttime motor vehicle collision on a busy road. As you approach the scene, which observation should cause you to alert the incident commander of a problem

One ambulance has been positioned with its lights facing oncoming traffic

You have just arrived by the side of a nonverbal patient with snoring respirations. The caregiver states that they were eating lunch when the patient suddenly collapsed face first onto the table. She also states that the patient has a history of a stroke, high blood pressure, and colon cancer. Your priority in caring for this patient at this time would be

Open and look into the patient's mouth

The secondary assessment of a patient who complains of right leg pain after falling down several stairs reveals a break in the skin where a fractured tibia bone broke through and then recessed back into the leg. The EMT should recognize this as which type of injury?

Open fracture

You have arrived at a residence to find a 6-month-old girl in her crib in cardiac arrest. Quick assessment reveals no rigor mortis or lividity. The mother states that she put the baby to bed at 9 p.m. last night and awoke at 3 a.m. to find her in her present state. What would your first action be

Open the airway and provide 2 ventilations

A male driver was ejected from his vehicle after it rolled several times at a high rate of speed. As you approach the patient, you note that he is unresponsive and struggling to breathe. He also has a laceration to the left side of his face and multiple contusions to his legs. After assigning another EMT to take manual cervical spine motion restriction, what should you do?

Open the airway using the jaw-thrust maneuver

When assessing and treating a traumatically injured female in her third trimester of pregnancy, the EMT must remember which important point?

Optimal care for the baby will come from caring for the mother

A female patient who is eight and a half months pregnant states that she does not feel well and is weak with clammy skin. She denies chest pain, shortness of breath, and dizziness. She states that she did vomit earlier in the morning and still feels somewhat nauseated. She is alert and oriented and has a patent airway. She is breathing 18 times per minute and has a heart rate of 88 beats per minute. Her blood pressure is 116/70 mmHg and her SpO2 reading is 95% on room air. The most appropriate care for this patient should include:

Oxygen at 15 1pm via a non re breather face mask

You believe that a patient who has been shot in the lower abdomen is bleeding internally and is in an early stage of shock. The patient's mental status is decreasing. What would be appropriate prehospital care of this patient?

Oxygen therapy and rapid transport to the hospital

You have been called to a geriatric extended care facility to transport a patient with fever and pneumonia to the hospital for further care and treatment. The primary focus of your care during this call will be

Oxygen therapy and safe transport

You are transporting a patient who was shocked by an electrical current while repairing a residential electrical service panel. He has partial-thickness burns to the hand and foot where the electricity entered and exited his body, respectively. When reassessing this patient, which finding would be of most concern

Palpation of an irregular heartbeat

You have been called to the post office to help with a disorderly male, on scene you find a middle-aged male patient who states that he needs to be protected because the CIA is out to get him for something he did not do. As a knowledgeable EMT, you would recognize the patient's behavior as

Paranoia

A patient dove into a shallow pool and struck his head on the bottom. Your assessment findings indicate that he has no motor or sensation in his legs, but can move his arms. The EMT should recognize this condition to be

Paraplegia

A 21-year-old male sustained a severe burn to his left leg while working on an industrial furnace. Coworkers immediately doused his flaming pants and removed them. Your primary assessment reveals no threats to the airway, breathing, or circulation. While performing the secondary assessment on his leg, which finding would be most critical?

Partial-thickness burn completely encircling the lower leg

While en route for a call involving a pregnant patient, an Emergency Medical Responder (EMR) on scene contacts you and reports that the patient is actively seizing. The EMR reports that she is seven months pregnant and has not been feeling well for the past three days (per family). Which instruction is critical for you to relay to the EMR?

Provide oxygen at 15 liters per minute

The family of a patient in hospice care for the final stage of lung cancer has summoned 911 because their father is short of breath. The patient is responsive to verbal stimuli and has an open airway with adequate respiration of 18 to 20 breaths per minute. His skin is cool and diaphoretic with diminished breath sounds bilaterally. Your partner reports a heart rate of 112 beats/min, blood pressure of 96/44 mmHg, and SpO2 of 82% on room air. The patient has a valid DNR order, but family is scared and wants him transported to the ED for evaluation. Your next action would be to

Provide supplemental oxygen, monitor vital signs, and transport the patient to the ED

A middle-aged male has been stabbed once in the left anterior chest. His airway is patent, respirations tachypneic, pulse weak and rapid, and skin cool and diaphoretic. Breath sounds are clear and equal bilaterally. The vital signs are pulse, 140 beats/min; respirations, 24 breaths/min; blood pressure, 100/78 mmHg; and SpO2, 96% on supplemental oxygen. JVD is present. Given this presentation, you would have a high index of suspicion for:

Pericardia! tamponade

In the cycle of domestic violence, in which phase does the violence occur on a regular basis?

Phase II

A 16-year-old male has been struck in the genitalia by a baseball. Assessment indicates a tremendous amount of edema as well as hematoma formation and bruising to his scrotum. The primary assessment reveals no life-threatening conditions; however, the patient is crying and rates the pain as 10/1 0. Which action is most appropriate for this patient

Place cool compresses to the testicles

The EMT recognizes that alcohol or drug dependence often occurs in individuals with post-traumatic stress disorder (PTSD) because

The alcohol or drug is used to blunt the sympathetic nervous system and emotional mental anguish

Assessment findings for a patient who was thrown from a motorcycle indicate that he has a flail chest wall segment to his right anterior chest. He exhibits labored breathing and an Sp02 of 92%. Breath sounds are clear and equal bilaterally. The segment has been stabilized, and you are prepared to start positive pressure ventilation. Given these assessment findings, which type of injury underlying the flail segment is your primary concern

Pulmonary contusion

Assessment of a young girl who was hit by a car while riding her bike reveals her to be responsive to painful stimuli with flexion of the extremities; she is also in respiratory distress. There is marked deformity to her thoracic spine and bruising noted to her anterior chest and abdomen. She does not move her legs when a noxious stimulus is applied to the lower extremities. Manual cervical spine motion restriction is being maintained and a cervical collar has been applied by fire department EMRs. Given the critical nature of this patient, which action is most appropriate for her care?

Quickly but carefully provide full spine motion restriction precautions on scene prior to rapid transport to the hospital

A "whiplash"-type neck injury is most often observed with which type of collision?

Rear-end impact

A young male patient has self-extricated from his vehicle after hitting a telephone pole and rolling his car several times at a high rate of speed. He is spitting blood and has a large area of tenderness and soft tissue injury to the left side of his chest. On scene, you did not observe any paradoxical motion of the chest wall. The patient remains alert and oriented with stable vital signs, but continually complains of painful breathing. During transport, what is it essential that you do

Reassess for paradoxical motion of the chest wall

When approaching a combat veteran who is highly agitated, the EMT must remember that:

Reassurance with a firm, calm voice may be necessary and should be attempted first

A 23-year-old female patient has been stung in the right arm by some unidentified marine life. Her arm and hand are swollen and red. The primary assessment is unremarkable and she is complaining of pain to the site of injury. You would

Remove rings from the patient's right hand

The mother of a 2-year-old has called EMS because her son has an axillary temperature of 103.2° F. On scene, your assessment reveals the boy to be confused and lethargic, with a rectal temperature of 104.1 ° F. When caring for this child, you would:

Remove the child's clothes and sponge his body with tepid water

The rescue captain has asked that you disconnect a car's battery to shut down all power to the vehicle completely at the scene of a one-car MVC. How should you do this safely

Remove the negative battery cable first

Poor positioning of an obese patient on the stretcher would most likely result in which immediate danger?

Respiratory distress

You suspect a trauma patient has a hemothorax to the left lung. Which assessment finding would reinforce this suspicion?

Respiratory distress and the signs and symptoms of shock

A mother asks you what causes her son to get bronchiolitis. You would inform her that this infection is caused by

Respiratory syncytial virus

The cerebellum is described as the area of the brain that:

Responding only to painful stimuli

You are transporting a patient who experienced a right-sided head injury after a large refrigerator fell and pinned his head between the appliance and the floor. On scene, the patient was responsive to verbal stimuli with the following vital signs: pulse, 96 beats/min; respirations, 16 breaths/min; blood pressure, 134/66 mmHg; and SpO2, 95%. During transport, as you reassess the patient, what would be the clearest sign that the head injury is worsening?

Responding only to painful stimuli

An 81-year-old female called 911 when she suddenly lost vision in her right eye. Based on this description, the EMT should be suspicious of:

Retinal detachment

When assessing a 3-year-old who is in respiratory distress, which assessment finding would be most concerning

Retractions observed above the clavicles

A man has been bitten in his arm by his dog. He states that the bite occurred several hours earlier, when he accidentally stepped on the dog's paw. When asked, he tells you that the dog is up-to-date on all her shots, including the rabies vaccine. Assessment reveals two small puncture wounds to the hand with some bruising in the surrounding tissue. The patient wants to refuse treatment and transport. Given these assessment findings, what is the primary reason this patient should be seen in the emergency department

Risk for infection

A patient suffered abrasions to the legs after falling on a moving treadmill. In relation to the function of the skin, the EMT should be most concerned about:

Risk for infection

A male patient fell 20 feet from a cliff to a trail below while hiking with his girlfriend. The primary assessment shows him to be confused, with an open airway and shallow breathing. His pulse is 72 beats/min and his blood pressure is 78/50 mmHg. The skin is warm and flushed. The patient has no motor ability or sensation in his legs. What is the most likely cause of this patient's presentation?

Spinal shock

A young male who was drinking heavily has been involved in a motor vehicle accident. As an unrestrained driver, his head went through the driver's-side window; in the process, a portion of his ear was severed. Blood is now coming from the remaining portion of his left ear. In overseeing the care of this patient, which action by another EMT would require corrective action?

Rolled gauze packed into the ear canal to slow bleeding

You are by the side of a 19-year-old female who is 36 weeks pregnant. When assessing this patient, which finding best indicates that she is in preterm labor

Rupture of the amniotic sac

The EMT recognizes that preventive mechanical maintenance performed regularly on the ambulance results in

Safer patient transports

When you are first on the scene of a terrorist incident involving WMD, which role is of the utmost importance

Scene size-up

A woman fell from a second-story window onto the concrete sidewalk below. She is unresponsive and has a large depression to the back and top of her skull. Additional findings include abdominal bruising and an angulated left ankle. Your partner reports that the patient's vital signs are pulse, 68 beats/min; respirations, 14 breaths/min and irregular; blood pressure, 198/112 mm Hg; and SpO2, 91 % on room air. Based on these assessment findings, what should the EMT suspect is occurring

Severe head injury with increasing pressure within the skull

An 8-year-old child was chasing his dog when the boy ran out into the street and was struck by a car. He is responsive to verbal stimuli and has an unstable pelvis and a bruise to the side of his face. His airway is open and his breathing is rapid and shallow. His radial pulse is rapid and weak and his skin is cool, moist, and diaphoretic. The patient also has abrasions to his back after being thrown by the car onto the pavement. Based on these assessment findings, the EMT would treat the patient for which life-threatening condition

Shock

A patient who was involved in a diving accident is found with his head positioned so that his left cheek is touching his left shoulder. He states that he has severe pain in his neck and it hurts to move his head. He denies numbness, tingling, or decreased strength in his arms or legs. What is the EMT's best course of action

Stabilize and maintain the head in the position in which the patient is holding it

As you pull up to a motor vehicle collision, you quickly scan the scene for clues as to the type and severity of injuries. Which observation would best indicate that a patient may have a head injury?

Starburst on the windshield

After placing a small rock in her mouth, a 7-month-old girl begins to choke. You are on scene within minutes and find her unresponsive in her mother's arms. You attempt to provide ventilations with the bag-valve mask, but are unsuccessful after two attempts. What would your next action be

Start cardiopulmonary resuscitation

A teenage boy was found by friends as he attempted to hang himself in a garage using chains suspended from the ceiling. As you approach, you note that the patient is conscious, struggling significantly to breathe, is ashen in color around the mouth and to the hands, and has contusions and swelling that encircle his neck from the chains. After taking manual cervical spine motion restriction and opening the airway, your next action would be to:

Start positive pressure ventilation

An unrestrained 32-year-old female motorist hit a utility pole head-on at 45 mph. The car sustained extensive damage. The patient has an open airway, is breathing poorly at 32 breaths per minute with absent alveolar breath sounds, and is responsive to painful stimuli. Her radial pulse is rapid and weak, and her skin is cool and cyanotic. Breath sounds are decreased on the right side. Emergency Medical Responders are initiating spine motion restriction precautions. Your next action would be to:

Start positive pressure ventilation

Assessment findings for a driver who was ejected from his vehicle in a rollover-type collision include unresponsiveness and bruising to the abdominal and pelvic areas, along with an open femur fracture. The patient has an open airway and is breathing at 32 times per minute with absent alveolar breath sounds. His skin is cool and diaphoretic; the radial pulses are weak. Manual cervical spine motion restriction is being maintained. The EMT's next action would be to

Start positive pressure ventilation

You are by the side of a 2-month-old baby who was born prematurely and is on an apnea monitor. While the patient is in your care, the apnea alarm emits a loud alarm. Quick assessment of the baby reveals no respiratory activity. What would your immediate action be?

Start positive pressure ventilation

You have arrived on the scene of an assault involving a knife. Assessment of the unresponsive 21-year-old male patient reveals him to have sustained multiple lacerations to the arms and abdomen, and a section of his intestine is now protruding through a large laceration in the area around the umbilicus. He has lost a significant amount of blood. His airway is open and he is breathing poorly at a rate of 28 breaths per minute. His radial pulse is weak, and his skin is cool and pale. What should the EMT do first?

Start positive pressure ventilation

A young female patient involved in a motor vehicle collision responds to painful stimuli by extending her arms and legs. Her airway is open, and her breathing is shallow and irregular. A radial pulse that is moderate in strength is palpated. Her pulse is 64 beats/min, respirations are 8 breaths/min and irregular, and blood pressure is 210/110 mm Hg, with an SpO2 of 90% on room air. The patient has an obvious deformity to her forehead and a left pupil that is dilated and does not respond to light. Manual spine motion restriction is being maintained by Emergency Medical Responders. At this time, the EMT's priority action is to

Start positive pressure ventilation at 20 breaths/min

Assessment indicates that a term newborn's respiratory rate is 38 breaths per minute and his heart rate is 80 beats per minute after administration of supplemental blow-by oxygen. The EMT should

Start positive pressure ventilation with supplemental oxygen

A young male has been pulled from a lake by his friends after being submerged for several minutes. Assessment reveals his airway to be patent, respirations absent, and a weak carotid pulse of 12 beats/min. Which care should you provide at this time?

Start positive pressure ventilation. place the patient on a long board, and transport to the hospital

A patient has been involved in a motorcycle crash and has a flail segment to the right lateral chest. His airway is open and he is breathing poorly at 24 breaths per minute with a decreasing SpO2. The EMT shows that he is appropriately caring for this injury when he

Starts positive pressure ventilation

Pathologic narrowing of the valves within the heart is referred to as:

Stenosis

A female patient struck her face on the steering wheel in a head-on motor vehicle collision and has deformity and crepitus to the bridge of her nose. Further assessment reveals bleeding deep within the nasal passages. Prior to your arrival, fire department EMRs have secured the patient to a long spine board and applied a cervical collar and straps. To best address the hemorrhage in the nasopharynx, the EMT would:

Suction blood from the airway as needed

You are caring for a male patient who was ejected from the passenger side of a vehicle that was traveling at a high rate of speed when it left the roadway and overturned several times. During your primary assessment, you note the patient to have blood in the airway, rapid and shallow breathing, an absent radial pulse, and a weak and rapid carotid pulse. You also visualize that his left leg is severely injured, with gross visible angulations from suspected multiple fractures. Your partner is triaging the other occupants of the vehicle and you can hear the wail of the arriving fire department vehicles. Given this situation, what should you do first

Suction the airway with a hand-operated suction machine while trying to minimize cervical spine motion

A 42-year-old male was involved in a fight and was stabbed in the right lateral chest. The knife is still impaled, and the patient now complains of shortness of breath. When he speaks, he gurgles and blood drains from his mouth. His respirations are labored and his radial pulse is weak. Which intervention should you perform first?

Suction the blood from the airway

You are assessing a young child who was briefly submerged after falling approximately 20 feet from an observation platform into the shallow portion of a lake. Family members jumped into the lake immediately and pulled the girl to safety. She is confused and begins to vomit profuse amounts of water. What should you do immediately?

Suction using a rigid-tip catheter, while maintaining manual spine motion restriction precautions

A teenage female has been removed from a pool. Your assessment reveals her to be responsive to painful stimuli with decorticate posturing. Her airway has water in it, and she appears to be vomiting water. Breathing is agonal at 8 breaths per minute; you hear a gurgling noise with each attempt at breathing. The patient's pulse is weak and slow. What should be the priority treatment you would render next?

Suction water from the airway

A construction worker has been shot with a nail gun, resulting in a long nail going through his cheek and becoming firmly embedded in the lower gum and jaw. There is considerable blood in his mouth, and the patient is in excruciating pain. Given this scenario, what would be your first priority?

Suctioning the airway of blood

A wife of a returning combat veteran called EMS because her husband is complaining of constant headaches. When you ask about post-traumatic stress disorder (PTSD) or traumatic brain injury (TBI), she states that the patient was injured by an improvised explosive device in Iraq, but cannot remember the event. She explains that the patient has undergone numerous CTs and MRls of head and has been told by his family doctor that there is no sign of injury. Regarding these medical conditions, you realize that:

TBI may exist despite the negative findings of the CT and MRI tests

A 41-year-old male patient was struck in the head with a metal pipe during a fight with an angry neighbor. The scene is safe, and as you approach the patient, you note that he is combative and has blood on the left side of his head and face, and on his shirt. His breathing appears to be labored and he is incontinent of urine. Which care should you perform immediately?

Take manual cervical spine motion restriction

You are called for a 78-year-old female who fell earlier in the day. As you enter her kitchen, you observe her sitting in a chair with ecchymotic areas to her left cheek and left arm. There also appears to be blood on her pants. She is on home oxygen for COPD and is breathing in a moderately labored manner. The patient states that she fell a few hours ago, and now the pain and swelling in her arm are unbearable. She has also developed significant lower neck pain. What should you do first?

Take manual spinal motion restriction precautions

A car has struck a utility pole at a high rate of speed. The male driver appears to be unresponsive and has snoring respirations. The vehicle has just been stabilized, and you have donned the appropriate personal protective equipment. What should you do next?

Take manual spine motion restriction

A 77-year-old male got out of bed, became dizzy, and fell onto the floor, hitting the right side of his face. You find the patient sitting on the floor with his wife next to him. His wife states that he did not lose consciousness and that this has happened several times over the past few days, ever since the doctor put him on a new blood pressure medication. Presently, the patient is alert and oriented and complains of dizziness, headache, and neck pain. He has a history of coronary artery disease, emphysema, and hypertension. Which action should the EMT perform first

Take manual spine motion restriction precautions and begin the primary assessment

How does the EMT's role in an "active shooter• incident vary from any other unsecured "shots fired" scene

The EMT's role is the same in both types of scenes

You are on scene of an MVC in which a 3-year-old female was in a car seat in the backseat. The child appears not to be injured, but the mother would like her taken to the hospital for an evaluation. Since the patient will need a car seat for transport, which criteria would indicate that the car seat she is currently sitting in cannot be used to safely transport the patient

The car seat appears to have a crack in the side when impacted by the door handle

In the human body, which condition is present when a person is in shock?

The cells do not get enough oxygen and waste products accumulate

An intoxicated patient has suffered a burn to his left lateral thigh after passing out with his leg touching the side of a kerosene heater. Close examination of the burn reveals tough leathery tissue in the center of the burn, with red skin with blisters surrounding it. Regarding the burn, what would the EMT agree to be true?

The central portion of the burn is a full-thickness burn and the surrounding portion is a partial-thickness burn

A 6-month-old male was outside with his parents and was stung on the tongue after putting a bottle in his mouth that had a bee on the nipple. In comparison to the same injury in an adult, why would the EMT be more concerned with this child?

The child's tongue is proportionally larger, increasing the chance of airway occlusion from minor swelling

A soldier patrolling the streets in a foreign country is suddenly fired on by a sniper. Which response would you expect to occur within his body?

The pupils dilate

An alert and oriented 29-year-old male diver surfaced and called 911 for severe right ear pain. When asked, he specifically denies nausea, dizziness, or pain other than in his ear. Assessment reveals no threats to the airway, breathing, or circulation. The ear shows no obvious injury or discharge. His vital signs are pulse 96 beats/min, respirations 16 breaths/min, blood pressure 146/76 mmHg, and SpO2 97%. What is the proper care for this patient?

Transport to the emergency department 15 minutes away

You are preparing to transport an 86-year-old female with an altered mental status who complains of nausea and vomiting. During transport, the occurrence of aspiration pneumonia would be best prevented by:

Transporting the patient in a semi-Fowler's position

Research by the American College of Surgeons Committee on Trauma has shown a higher death rate for patients who were immobilized to a backboard when:

The injury was the result of penetrating trauma to the head or torso

A car, driven by an unrestrained male, strikes a utility pole at 35 mph. Given this scenario, which would be true?

The internal organs struck the inside of the body at a speed of 35 mph

You have been selected to represent your EMS system on a disaster planning committee involving many different agencies. Regarding preparation for a possible disaster, which item is most beneficial to stress in the meeting

The lay community should become involved in disaster planning by learning basic first aid and CPR

You are assessing the pupils of a patient who hit his head after falling from the top of a tractor trailer. Which pupillary finding suggests a closed head injury?

The left pupil constricts to light but the right pupil does not

You have been called to a crisis shelter for a patient possibly having a baby. On scene, you find an older female patient who is developmentally challenged and cannot communicate normally. Staff states that the patient just came to the shelter and they know nothing about her, other than that she is pregnant. Assessment reveals crowning, and you proceed to deliver what appears to be a healthy baby boy. When assessing the mother after delivery, which finding should increase your suspicion that the woman may be carrying twins?

The mother's abdomen remains large after delivery of the first baby

You have been called by the caregiver of a 91-year-old female with dementia. The patient complains of pain and deformity to her upper left arm. As you assess the patient, you note various bruises to the patient's body, and become suspicious of elder abuse. Which action is most appropriate at this time?

Treat the patient for a possible broken arm and initiate transport

You have been assigned to the triage unit at a multiple-casualty incident. A patient with a yellow triage tag arrives. Your immediate action should be to

Triage the patient again

At the scene of a hazardous materials emergency, you have been assigned to the cold zone. In this zone, which type of activities will you be performing

Triage, vital signs, and medical histories

A patient was struck in the back by a heavy piece of wood that was being bent to make a form for a concrete arch. He is responsive, but cannot feel or move his legs. He has bruising to his back and is incontinent of urine. The skin below the injury site is red and warm. As a knowledgeable EMT, you should realize that:

The paralysis may resolve

Which description best explains the term "silent heart attack

The patient had little to no chest discomfort

You note a 5-year-old patient seizing as you approach him. Which observation or piece of information would be most alarming?

The patient has been seizing for 12 minutes

Another crew has called for your assistance in extricating an obese male with a nosebleed from a third-floor bedroom. As you enter the room, which observation indicates proper management of the patient's condition

The patient is sitting upright, leaning forward, with nostrils pinched shut

You are assessing a patient with COPD who uses an oxygen concentrator. In this case, you recognize:

The patient requires less than 6 lpm of oxygen

A 92-year-old male states that he "passed out" and fell. He regained consciousness a few moments later, but had shortness of breath and sharp chest discomfort. Which element of the patient's history is of greatest concern?

The patient took nitroglycerin and still has pain

You have been called to a residence for a patient who is sick. On scene, the family tells you that the patient is being treated for "stomach flu" and has had a fever of 102°F for the past two days. She has been taking all of the medications prescribed by her doctor, but this evening became very "sweaty." Assessment shows her to be stable with a pulse of 88, respirations of 18 breaths/min, adequate blood pressure of 128/68 mmHg, and an oral temperature of 100.2°F. Based on these data, the EMT should recognize that:

The patient's body is cooling itself by transferring heat into the sweat, which is then evaporated into the air

Which statement regarding the placenta, made by your EMT partner while reviewing the OB/GYN protocols, is most accurate?

The placenta typically delivers itself within 20 minutes of the birth of the baby."

You have been given an order to cut through the "B" posts of a four-door car with a trapped passenger inside. As an EMT with knowledge of vehicle extrication, which posts do you cut

The posts that the front doors close against

A woman strikes her forehead against the dashboard of her car as it strikes a tree. Immediately following the impact, her brain shifts back and forth within her skull. Based on the anatomy and physiology of the skull, which statement is true?

The ridges of the basilar skull can damage the brain as it moves back and forth

What forms the physical barrier against the entrance of undesirable microorganisms, found in the environment, into the body

The skin

What is the best description of the "platinum 10 minutes" as it applies to care of the trauma patient?

The target maximum scene time for EMS as it applies to the seriously injured trauma patient

A 19-year-old female was assaulted by her intoxicated boyfriend. After punching her in the chest, he slashed her face with a knife. Assessment reveals the patient to be stable with a large laceration from beneath her right eye to her right ear. She is very upset and crying and repeatedly asks you if this injury will leave a permanent scar on her face. Your best response would be:

When we get to the hospital, you can discuss this more thoroughly with the doctor

You and several other EMTs are on location where a young female lost control of her car and ended up in a shallow lake. The car is approximately 20 feet from shore and upright, with the wheels touching the bottom of the lake. The woman is on the car's roof; she states that she is not hurt but is scared. What should the EMT do to rescue the patient?

Toss a rope in a throw bag to the patient

The EMT is correctly using a rope in a throw bag when he:

Tosses the throw bag using an underhand motion toward the patient

Family members tell you that 3 hours ago, their 76-year-old mother suddenly became confused and had great difficulty speaking. However, within 15 minutes she returned to normal. Based on this description, the EMT should be suspicious of:

Transient ischemic attack

Bystanders called 911 for a female patient who was screaming obscenities in a store parking lot. On arrival, you find the patient to be sitting in the middle of the parking lot muttering incomprehensible phrases. After conducting the primary assessment and finding no immediate life-threatening conditions, you move her to the ambulance. Additional assessment findings include abrasions to her arm and urinary and fecal incontinence. She also has a bottle of Prozac in her pocket. You reference Prozac in your medication book and find it to be used to treat depression. With this information, what will be a component of how you should treat and/or transport this patient

Transport the patient on her left side

A 62-year-old female with severe kyphosis fell and hit the back of her head, which is now oozing blood from a small laceration. Family members state that she suddenly complained of a "very bad headache" before collapsing to the kitchen floor. She has snoring respirations and cyanosis in the extremities. Which intervention should you perform first?

Use a jaw-thrust maneuver

Which finding is of greatest concern when assessing a 33-year-old female who is eight months pregnant

Vaginal bleeding not associated with pain

A 44-year-old electrician has been shocked. He now responds to verbal stimuli with garbled speech. His airway is open and he is breathing poorly at a rate of 8 times per minute. His pulse is slow and irregular. Which action would the EMT perform next

Ventilate with a bag-valve mask at 1 O breaths/min

How would you tell a pregnant woman to restrain herself when traveling in a car?

Wear all seat belts normally, as you would if not pregnant

While attempting resuscitation of a baby with possible SIDS, which question would be appropriate for the EMT to ask the mother

Were there any problems with the pregnancy or his birth?"

A patient with a closed fracture to the forearm has been properly splinted when which other structures are properly immobilized

Wrist and elbow

A 49_year-old man was fishing in a stream of cold water when he became submerged after getting tangled in some hidden roots. The patient was pulled from the stream by friends after approximately 5 minutes of submersion. Your assessment reveals that he is responsive to painful stimuli and breathing 5 times per minute. His pulse is weak and rapid. Which set of instructions would be appropriate to relay to the team of rescuers?

"Let's move him into the warm ambulance as quickly as possible, and then cut off his clothes and cover him

A 44-year-old female patient was hiking in a state park when she was bitten by a snake in her left calf. To get help, she had to hike 4 miles to the first-aid station where you are located. The hike took her 1 ½ hours. Your assessment reveals a red and swollen area to her leg, and the patient complains of discomfort to that area. Closer inspection reveals two puncture wounds in the center of the affected area. The patient asks you if she is going to die from the snake bite. What would be your most appropriate reply?

"While you still need to get checked out, if venom was injected, the signs and symptoms of generalized reaction typically appear very quickly.


Kaugnay na mga set ng pag-aaral

Macroeconomics 102 final study guide

View Set

Pharm Exam 3: Ch. 49 - Antidysrhythmic Drugs (Key Bank & Elsevier Q's)

View Set

Chapter 10 Food Selection and Preparation (Exam 2)

View Set

Chemistry Chapter 25 (Nuclear Radiation)

View Set

L2, R41: Backtesting and Simulation

View Set